所在位置: 查字典英语网 >高中英语 > 高考英语 > 高考高考英语 > 高考高考复习指南 > 3年高考2年模拟1年原创备战2017高考精品系列之英语:专题18 人物故事、人物传记类(原卷版)

3年高考2年模拟1年原创备战2017高考精品系列之英语:专题18 人物故事、人物传记类(原卷版)

发布时间:2017-03-03  编辑:查字典英语网小编

  3年高考2年模拟1年原创精品系列

  专题18人物故事、人物传记类阅读

  【2017年高考命题预测】

  预测一 高考阅读理解所选材料通常都是国外网站上的地道的英语文章,然后经过命题专家整合、改编而成。文章主要介绍国内外的文化风俗、历史名胜、人文地理、著名人物等。

  预测二 文章的篇幅较短,词数多为280~330,兼顾多种体裁,通常有记叙文、说明文、议论文、应用文等,要求考生充分利用所给信息,回答文章提出的问题。通常文章难度不大,关键是考查考生高效处理信息的能力,还要求考生对文章有更深层次的理解。但是题目并不容易,很容易出错。全国卷的命题逐渐向地方自主命题卷,尤其是新课标地区的试卷看齐,不断增加词汇量和文章长度,向新课标过渡。

  预测三 题目设置灵活多样,通常以细节理解题为主,推理判断题为辅,兼顾猜测词义、句意题及目的意图题和主旨大意题。这种命题特点要求考生更好地了解文章中的有用信息,提高阅读速度。

  2017考纲解读和近几年考点分布

  人物传记主要向读者介绍科学家、文学家、思想家、政治家、企业家和影视、体育明星等杰出人物在各自领域所取得的重大成就,尤其注重介绍这些杰出人物如何克服身体残缺、自然环境和社会环境所造成的各种困难。为人类做出巨大贡献,也为自己的事业谱写精彩篇章。这些杰出人物不仅推动了社会的进步,而且他们对事业的追求、对人民的厚爱成为时代最重要的精神财富。通过阅读人物的小传记或摘录该人物一些典型事件,如人生的起伏、克服困难取得成功等,考生可以得到激励或受到启发,同时也可以了解这些人物所生活的社会文化背景。

  故事类阅读理解选材来源于日常生活,给人以似曾相识的感觉。体裁一般是记叙文,包含记叙文的时间、地点、人物、事件等要素。写作手法多采用时间、空间或逻辑线索贯穿文章始终。命题以细节为主,推理为辅。应特别注意对人物的外貌、语言、动作和思想的描写,从而把握人物性格特点和作者的情感态度。若是叙述性的文章,要读懂起因、经过、结果。若是故事性文章应渎懂故事的发生、发展、高潮、结局。

  1.熟悉人物介绍文特点

  人物传记是高考英语阅读理解题重要题材,它主要向读者介绍科学家、文学家、思想家、政治家和企业家等杰出人物在各行各业所取得的重大成就,并特别着重介绍这些杰出人物如何克服自身身体残缺或自然环境和社会环境所造成的各种困难,用强烈的事业心和责任感、用汗水和泪水、用鲜血和青春为人类做出巨大贡献,为生命谱写精彩乐章。当然这些杰出人物留给社会的不仅仅是和平发展和经济效益,更重要的是,他们对事业的追求和对人民的厚爱成为我们这个时代最重要的精神财富。

  2.熟悉人物介绍文命题规律

  (1)人物时段介绍判断题

  这种试题往往以What is the article mainly about? What life part of the hero is mentioned in the text? 为设问方式考查对人物时段介绍的判断。因为人物传记往往介绍杰出人物一生学习、工作和生活情况,因此这种试题往往以The life of the hero.或The whole life of the hero is mentioned in our text.为答案。这种试题也可转换形式,以How old is the hero now? How old was the hero when he died? How many years did he live?为设问方式。因为人物传记往往要从人物出生介绍到现在/人物死亡,因此推算这些试题答案时,应仔细分析思考人物的起止年龄。

  (2)童年生活质量判断题

  人物传记常出现What do we know about his childhood?之类的试题。杰出人物为什么有超强的毅力和惊人的吃苦精神?因为他们从小身处逆境饱尝生活艰辛,从此发奋努力,全力实现心中理想,因此这种试题答案多为:He lived a hard life.或He had an unhappy childhood.或He didn抰 live as happy as the other children.

  (3)人物简历正确顺序判断题

  人物传记的最大特点就是记叙人物一生的经历,因此人物简历正确顺序判断题是常考试题,这种试题往往从人物全部活动中挑选4~6个重要活动并打乱其顺序,要求学生重新排序,并选择相应正确答案。解题时应先阅读试题中所列4~6个重大事件,力争对事件有一定印象,然后回到原文中重新阅读原文,并从事件的发生、发展、高潮及事件发生的前因后果等角度仔细排列事件发生的前后顺序。解题时切忌使用一一代入法,因为这么做看似可行,其实费时又费力,而且这么做难度加大,极易产焦虑心理,陷入恶性循环。

  (4)人物名言理解判断题

  杰出人物经过长期默默无闻的努力终获成功后,一定会引起人们强烈关注,因此媒体采访其先进事迹并加以褒扬和宣传是理所当然的事。而最成功的人往往又是最谦虚的人,尤其是杰出人物平凡甚至卑微的出生让他们觉得自己的成功算不上什么,不值得大竖特竖,因此这些杰出人物谦虚的语言往往成为资讯媒体更为关注的内容,这也成了杰出人物最能打动人们的地方。因此人物传记阅读理解常出现人物名言理解判断题。这些试题往往以“… ”means_________.为设问方式考查对人物名言的正确理解。其答案往往为I am ordinary或I抦 nothing. 或I抦 just so-so等。

  (5)人物成功原因判断题

  杰出人物的成功为社会创造了巨大的精神财富和物质财富,人们不禁会探究其成功秘诀,使之转化为新的生产力,因此人物传记阅读理解常出现人物成功原因判断题。这些试题往往以Why did the hero finally succeed?或What is the secret of the hero抯 success?或What can we learn from the hero抯 success?为设问方式考查对人物成功因素的判断。其答案往往为“He/She is not easy to bow before difficulties.或He /She never gives in to all kinds of hardships. 或He/She never gives up/loses heart in face of failure.或He/She has strong perseverance.或He/She has strong self-belief等。”

  (6)人物性格判断题

  人们常所说有成就的人往往是有个性的人,可以说正是杰出人物的独特个性铸就了他的成功,因此探究人物个性也成了人物传记阅读理解重要题型,不过这种试题与人物成功原因判断题有异曲同工之处,其回答也大同小异,这里不再赘述。

  (7)人物重要成果判断题

  所有人物介绍文均会介绍杰出人物重要成果,因此常出现人物重要成果判断题,这种试题常以What was the use of...?...would solve....为设问方式,解题时应认真阅读原文对人物重要成果的介绍及其主要作用和现实意义,从而准确解答试题。

  (8)人物困难判断题

  杰出人物的成功不是一帆风顺的,都会出现重大困难,因此人物介绍文常出现人物困难判断题,这种试题常以What was the biggest difficulty that...faced? ...failed to continue his research work because...,解题时应认真阅读原文对人物困难的描绘,将所遇困难进行排列,并弄清其最大困难是什么,以便准确做题。

  (9)人物地位判断题

  杰出人物可能逝世多年,但人们是不会忘记他们的,人们是会公平评价他们的历史地位的,因此人物介绍文常出现人物地位判断题,这种试题常以From the passage, we can infer that....或According to the passage,...was...为设问方式,解题时应认真阅读原文对人物重大贡献及社会影响进行介绍的内容,从而准确解答试题。

  阅读这类文章,应特别注意对任务的外貌、语言、动作、思想的描写,进而把握人

  物性格特点和作者的情感态度。命题以细节为主,推理为辅。那么怎样准备理解细节呢?首先,从问题中找到关键词,然后以此为线索,运用略读和查阅的技巧在文中寻找细节,找到后再把这一部分内容仔细阅读,迅速比较选项和文中细节的区别。

  若是叙述性的文章,要读懂起因、经过、结果。若是故事性文章应读懂故事的发生、发展、高潮、结局。它们是我们读懂故事所必需的。

  若是幽默故事,应体会风趣的对话、特别的动作,它们是幽默的精髓。

  【考点pk】

  名师考点透析

  考点一、细节事实理解。

  一篇文章必然是由许多具体的细节、具体的内容构成的,所有这些信息以某种顺序(时间、空间、情感变化等)排列起来,来进一步解释或阐释主题,体现文章的主旨大意。

  细节理解题目主要考查学生对文章具体事实和细节的理解能力,属于表层理解题范畴,难度较小,但在整个阅读理解题中所占比例最大,细节理解题多从文章的某个具体事实或细节出发来设计题目。

  此类题通常根据文章具体内容来设问,也常使用下列方式来提问

  1.Which of the following statements is(not) true/correct?

  2.Which of the following is not mentioned?

  3.All of the following are true except...

  4.According to the passage,when/where/why/how/how/what/which,etc...?

  1.直接事实题

  在解答这类问题时要求学生抓住题干文字信息,采用针对性方法进行阅读,因为这类题的答案在文章中可以直接找到。 如:

  A giant dam was built many years ago to control the Colorado River in the U.S.This dam was built to protect the land and houses around the river.

  This huge dam is in the Black Canyon.It is possible to drive a car from one side of the river to the other on a road,which is on the top of the dam.This dam is so big that there is an elevator inside.The elevator goes down forty-four stories from the road to the bottom.There is enough concrete in this dam to build a highway from New York to San Francisco.Thousands of people worked on this dam for five years.

  This huge dam was called Boulder Dam when it was finished in 1936.Later it was renamed Hoover Dam in honor of a president of the United States.Hoover Dam,one of the highest dams in the world,is situated between the states of Arizona and Nevada.

  Q:Hoover Dam lies______ .

  A.between Arizona and Nevada

  B.in the Black Canyon

  C.between New York and San Francisco

  D.both A and B

  【解析】由第二段的第一句话和最后一段的最后一句话可得出正确答案是D项。

  2.间接事实题

  解答此类题,需要结合上下文提供的语境和信息进行简单的概括和判断或者要进行简单的计算。

  Invited by Mr.Ye Huixian,host of the well-received TV programme“Stars Tonight”,Miss Luo Lin,Miss Asia of 1991,appeared as the guest hostess on the Shanghai TV screen last Sunday.

  Born in Shanghai and taken to Hongkong when she was only six years old,Luo Lin has never dreamed of being Miss Asia.Her childhood dream was to be an air hostess.Before she took part in the competition,she had been an airhostess in Cathay Airline for seven years.However, it still took her three months to learn the art of walking on the stage,dancing,singing,making-up and other proper manners,designed by the Asia TV Station.

  “It’s really a hard job for me.I won’t enter for such competition any more.Anyhow,I am quite lucky.I am also glad to have had more chance to work for the social welfare since I won the title.This time,in Shanghai,I’d love to make a deep impression on my TV audience,”said Luo Lin with a sweet smile.

  Q:Which of the following is NOT true?

  A.Luo Lin is a native of Shanghai.

  B.Luo Lin moved to Hongkong with her parents.

  C.Luo Lin won the title of Miss Asia in 1991.

  D.Asia TV Station helped Luo Lin to become Miss Asia.

  【解析】A、C和D项都可在原文找到答案,而B项原文所给的是:taken to Hongkong可判断不是moved to Hongkong with her parents,因此选B。

  3、数据推算。

  If you register both the VIEW qualification course and the Forum at the same time,you will save_______ .

  A.$100 

  B.$300 

  C.$350 

  D.$400

  这类题目要求学生就文章提供的数据,以及数据与文中其他信息的关系做简单计算和推断。在做此类题时:

  1.要抓住并正确理解与数据有关的信息含义。

  2.弄清众多信息中那些属于有用信息,那些属于干扰信息。

  3.不要孤立看待数字信息,而要抓住一些关键用语的意义。

  考点二、主题理解或写作意图推断。

  一、主旨阅读理解题考查的内容

  1.短文的标题(title,headline);

  2.短文或段落的主题(subject);

  3.中心思想(main idea);

  4.作者的写作目的(purpose)。

  二、此类题的设问方式

  1.What would be the best title for the text?

  2.What does the second paragraph mainly discuss?

  3.What is the passage mainly about?

  4.The main purpose of announcing the above events is__________.

  考点三、推理推断。

  推理题要求考生根据文章提供的事实和线索进行逻辑推理,推断出作者没有提到或者没有明说的事实或者可能发生的事实。这类题旨在考查学生透过词语的字面意义去理解作者的言外之意或弦外之音的能力,属于深层理解题。

  此类题的设问常常包括infer,imply,suggest,conclude 等词,这类题的设问方式主要有:

  1.We can infer from the Passage that__________.

  2.What can be inferred from the Passage?

  3.Which of the following can be inferred from the Passage?

  4.It can be inferred from Paragraph 3 that__________.

  5.The author suggests in this paragraph that__________.

  6.The writer implies that__________.

  7.It can be inferred that__________.

  8.It can be concluded from the Passage

  that__________.

  9.On the whole,we can conclude that__________.

  10.From the text we can conclude that__________.

  11.After reading the Passage

  we may conclude that__________.

  12.What conclusion can be drawn from the Passage?

  13.The author is inclined to think that__________.

  14.When the writer talks about,what he really means is that__________.

  15.What’s the writer’s attitude/feeling towards...?

  16.In the writer’s opinion,...

  近年来,高考加大了对学生判断推理能力的考查。判断推理题要求在理解原文表面 文字信息的基础上,作出一定判断和唯论,从而得到文章的隐含意义和深层意义。推理题所涉及的内容可能是文中某一句话,也可某几句话,但做题的指导思想都是以文字信息为依据,旣不能做出在原文中找不到文字根据的推理,也不能根据表面文字信息做多步推理。做题时要注意题干的语言形式,如According to the passage .It can be inferred from the passage that..; It can be concluded from the passage that...等,虽然从表面上看是问有关全文的题,但实际上不用看全篇,仍然只需要根据选项中的线索找到原文中与之相关的一句话或几句话,然后得出答案。针对推理题的不同情形,可以采取以下做法:1. 假如题干中有具体线索,根据具体线索找到原文相关句(一句或几句话),然卮做出推理; 2.假如题干中无线索,如 It can be inferred from the passage that...; It can be concluded from the passage that...等,先浏览一下4个选项,排除不太可能的选项,然后根据最可能的选项中的关键词找到原文相关句,作出推理;3.如果一篇文章中其他题都未涉及文章主旨, 那么推理题,如infer,conclude题型,可能与文章主旨有关,考生应该定位到文章主题所在位置(如主题句出现处);假如其他题己经涉及文章主旨,那么要求推断出来的内容可能与段落主题有关,如果如此,应该找段落主题所在处;如果不与段落主题有关,有时与全文或段落的重要结论有关,这时可以寻找与这结论相关的原文叙述。

  考点四、词义、句义猜测。

  词义猜测阅读理解题考查对文中关键词语的理解。在阅读理解题中,所考查的词或短语的意义往往不停留在字面上,要根据语境来判断。

  此类题的设问方式主要有

  1.The word “...”in Line...means/can be replaced by...

  2.As used in the passage,the phrase “...”suggests...

  3.From the passage,we can infer that the word/phrase “...” is/referred to...

  4.The word “...” is closest in meaning to...猜词是应用英语的重要能力,也是高考中常用的题型。它不但需要准确无误地理解上下文,而且要有较大的泛读量,掌握或认识较多的课外词汇。我们要学会“顺藤摸瓜”,通过构词、语法、定义、同位、对比、因果、常识、上下文等线索确定词义。

  1.定义法。如:

  Annealing is a way of making metal softer by heating it and then letting it cool very slowly.

  句子给予annealing 以明确的定义,即 “退火”。

  It will be very hard but also very brittle—that is,it will break easily.

  从后面的解释中我们可以了解到brittle 是“脆”的意思。

  The herdsman,who looks after sheep,earns about 650 yuan a year.

  定语从句中 looks after sheep 就表明了 herdsman 的词义为“牧人”。

  2.同位法。如:

  They traveled a long way and at last got to a castle,a large building in old times.

  同位语部分a large building in old times 给出了 castle 的确切词义,即“城堡”。

  We are on the night shift—from midnight to 8 a.m.—this week.

  两个破折号之间的短语很清楚地表明night shift 是“夜班”的意思。

  3.对比法。如:

  She is usually prompt for all her class,but today she arrived in the middle of her first class.

  but一词表转折,因此but 前后的意思正相反。后半句的意思是“她今天第一节上了一半才来”,因此反向推理,可得出她平时一向“准时”的结论。

  4.构词法(前缀、后缀、复合、派生等)。如:

  Perhaps,we can see some possibilities for next fifty years.But the next hundred?

  possibility 是 possible 的同根名词,据此可以判断定 possibility 意思是“可能性”。

  5.因果法。如:

  The lack of movement caused the muscles to weaken.Sometimes the weakness was permanent.So the player could never play the sport again.

  从后面的结果“永远不能再运动”中,可以推测 permanent 的意思为“永远的,永久”。

  【三年高考】

  1高考试题及其解析

  1.【2016·全国新课标I】C

  I am peter Hodes ,a volunteer stem courier. Since

  March 2017, I've done 89 trips of those , 51 have been abroad, I have 42 hours to carry stem cells(干细胞)in my little box because I've got two ice packs and that's how long they last, in all, from the time the stem cells are harvested from a donor(捐献者) to the time they can be implanted in the patient, we’ve got 72 hours at most, So I am always conscious of time.

  I had one trip last year where I was caught by a hurricane in America. I picked up the stem cells in Providence, Rhode Island, and was meant to fly to Washington then back to London. But when I arrived at the check-in desk at Providence, the lady on the desk said: “Well, I’m really sorry, I’ve got some bad news for you-there are no fights from Washington.” So I took my box and put it on the desk and I said: “In this box are some stem cells that are urgently needed for a patient-please, please, you’ve got to get me back to the United Kingdom.” She just dropped everything. She arranged for a flight on a small

  plane to be held for mere-routed(改道)me through Newark and got me back to the UK even earlier than originally scheduled.

  For this courier job, you’re consciously aware than that box you’re got something that is potentially going to save somebody’s life.

  29.Which of the following can replace the underlined word “courier” in Paragraph17

  A provider

  B delivery man

  C collector

  D medical doctor

  30.Why does Peter have to complete his trip within 42hours? A. He cannot stay away from his job too long. B. The donor can only wait for that long. C. The operation needs that very much. D. The ice won't last any longer. 31.Which flight did the woman put Peter on first? A. To London        B. To  Newark

  C. To Providence       D. To Washington

  2.【2016·北京】B

  Surviving Hurricane Sandy(飓风桑迪)

  Natalie Doan,14, has always felt lucky to live in Rockaway, New York. Living just a few blocks from the beach, Natalie can see the ocean and hear the wave from her house. 揑t抯 the ocean that makes Rockaway so special,?she says.

  On October 29, 2017,put that ocean turned fierce. That night, Hurricane Sandy attacked the East Coast, and Rockaway was hit especially hard. Fortunately, Natalie抯 family escaped to Brooklyn shortly before the city抯 bridge closed.

  When they returned to Rockaway the next day, ,pthey found their neighborhood in ruins. Many of Natalie抯 friends had lost their homes and were living far away. All around her, people were suffering, especially the elderly. Natalie抯 school was so damaged that she had to temporarily attend a school in B ,rooklyn.

  In the following few days, the men and women helping Rockaway recover inspired Natalie. Volunteers came with carloads of donated clothing and toys. Neighbors devoted their spare time to helping others rebuild. Teenagers climbed dozens of flights of stairs to deliver water and food to elderly people trapped in powerless high-rise buildings.

  揗y mom tells me that I can抰 control what happens to me,?Natalie says. 揵ut I can always choose how I deal with it.?

  Natalie抯 choice was to help.

  She flightcreated a website page matching survivors in need with donors who wanted to help. Natalie posted introduction about a boy named Patrick, who lost his baseball card collecting when his house burned down. Within days, Patrick抯 collection was replaced.

  In gthe coming months, her website page helped lots of kids: Christopher, who received a new basketball; Charlie, who got a new keyboard. Natalie also worked with other organizations to bring much-need supplies to Rockaway. Her efforts made her a famous person. Last April, she was invited to the White House and honored as a Hurricane Sandy Champion of Change.

  Today, the scars(创痕)of destruction are still seen in Rockaway, but hope is in the air. The streets are clear, and many homes have been rebuilt. 揑 can抰 imagine living anywhere but Rockaway,?Natalie declares. 揗y neighborhood will be back, even stronger than before.?59.Whe012,pn Natalie returned to Rockaway after the hurricane ,she found______.

  A.some friends had lost their lives

  B.her neighborhood was destroyed

  C.her school had moved to Brooklyn

  D.the elderly were free from suffering

  60.According to paragraph4,who inspired Natalie most?

  A.The people helping Rockaway rebuild

  B.The people trapped in high_rise building

  C.The volunteers donating money to survivors

  D.Local teenagers bringing clothing to elderly people

  61.How did Natalie help the survivors?

  A.She gave her toys to the kids

  B.She took care of younger children

  C.She called on the White House to help

  D.She built an information sharing platform

  62.What does the story intend to tell us?

  A.Little people can make a big difference

  B.A friend in need is a friend indeed

  C.East or West,home is best

  D.Technology is power

  3.【2016·天津】B

  Every man wants his son to be somewhat of a clone, not in features but in footsteps. As he grows you also age, and your ambitions become more unachievable. You begin to realize that your boy, in your footsteps, could probably accomplish what you hoped for. But footsteps can be muddied and they can go off in different directions.

  My son Jody has hated school since day one in kindergarten. Science projects waited until the last moment. Book reports weren’t written until the final threat.

  I’ve been a newspaperman all my adult life. My daughter is a university graduate working toward her master’s degree in English. But Jody? When he entered the tenth grade he became a “vo-tech” student(技校学生). They’re called “motorheads” by the rest of the student body.

  When a secretary in my office first called him “motorhead”, I was shocked. “Hey, he’s a good kid,” I wanted to say. “And smart, really.”

  I learned later that motorheads are, indeed, different. They usually have dirty hands and wear dirty work clothes. And they don’t often make school honor rolls(光荣榜).

  But being the parent of a motorhead is itself an experience in education. We who labor in clean shirts in offices don’t have the abilities that motorheads have. I began to learn this when I had my car crashed. The cost to repair it was estimated at $800. “Hey, I can fix it,” said Jody. I doubted it , but let him go ahead, for I had nothing to lose.

  My son ,with other motorheads, fixed the car. They got parts(零件)from a junkyard, non-toasting toaster have been fixed. Neighbours and co-workers trust their car repair to him.

  Since that first repair job, a broken air-conditioner, a non-functioning washer and a non-toasting toaster have been fixed. Neighbors and co-workers trust their car repairs to him.

  These kids are happiest when doing repairs. They joke and laugh and are living in their own relaxed world. And their minds are bright despite their dirty hands and clothes.

  I have learned a lot from my motorhead: publishers need printers, engineers need mechanics, and architects need builders. Most important, I have learned that fathers don’t need clones in footsteps or anywhere else.

  My son may never make the school honor roll. But he made mine.

  41. What used to be the author’s hope for his son?

  A. To avoid becoming his clone.

  B. To resemble him in appearance.

  C. To develop in a different direction.

  D. To reach the author’s unachieved goals.

  42. What can we learn about the author’s children?

  A. His daughter does better in school.

  B. His daughter has got a master’s degree.

  C. His son tried hard to finish homework.

  D. His son couldn’t write his book reports.

  43. The author let his son repair the car because he believed that_______.

  A. His son had the ability to fix it.

  B. it would save him much time.

  C. it wouldn’t cause him any more loss

  D. other motorheads would come to help.

  44. In the author’s eyes, motorheads are _______.

  A. tidy and hardworking

  B. cheerful and smart

  C. lazy but bright

  D. relaxed but rude

  45. What did the author realize in the end?

  A. It is unwise to expect your child to follow your path.

  B. It is important for one to make the honor roll.

  C. Architects play a more important role than builders.

  D. Motorheads have greater ability than office workers.

  4.【2016·浙江】D

  Two things changed my life:

  my mother and a white plastic bike basket. I have thought long and hard about it and it’s true. I would be a different person if my mom hadn’t turned a silly bicycle accessory into a life lesson I carry with me today.

  My mother and father were united in their way of raising children, but it mostly fell to my mother to actually carry it out. Looking back, I honestly don’t know how she did it. Managing the family budget must have been a very hard task., but she made it look effortless. If we complained about not having what another kid did, we’d hear something like, “I don’t care what so –and –so got for his birthday, you are not getting a TV in your room a car for your birthday a lsvish sweet 16 party.” We had to earn our allowance by doing chores around the house. I can still remember how long it took to polish the legs of our coffee table.My brothers can no doubt remember hours spent cleaning the house .Like the two little girls growing up at the White House,we made our own beds (no one left the house until that was done)and picked up after ourselves.We had to keep track of our belongings ,and if something was lost ,it was not replaced.

  It was summer and ,one day ,my mother drove me to the bike shop to get a tire fixed---and there it was in the window, White, shiny, plastic and decorated with flowers ,the basket winked at me and I knew ----I knew---I had to have it.

  “It’s beautiful,” my mother said when I pointed it out to her,”What a neat basket.”

  I tried to hold off at first ,I played it cool for a short while. But then I guess I couldn’t at and it any longer:“Mom, please can I please ,please get it? I ‘ll do extra chores for as long as you say, I’ll do anything ,but I need that basket,I love that basket.Please ,Mom .Please?”

  I was desperate.

  “You know,” she said ,gently rubbing my back while we both stared at what I believes was the coolest thing ever,” If you save up you could buy this yourself.”

  “By the time I make enough it’ll bu gone!”

  “Maybe Roger here could hold it for you,” she smiled at Roger ,the bike guy.

  “He can’t hold it for that long ,Mom .Someone else will buy it .Please, Mom,Please?”

  “There might be another way,” she said.

  And so our paying plan unfolded. My mother bought the beautiful basket and put it safely in some hiding place I couldn’t find. Each week I eagerly counted my growing saving increased by extra work here and there (washing the car ,helping my mother make dinner, delivering or collecting things on my bike that already looked naked

  without the basket in front).And then ,weeks later ,I counted ,re-counted and jumped for joy. Oh ,happy day ! I made it! I finally had the exact amount we’d agreed upon….

  Days later the unthinkable happened. A neighborhood girl I’d played with millions of times appeared with the exact same basket fixed to her shiny ,new bike that already had all the bells and whistles. I rode hard and fast home to tell my mother about this disaster. This horrible turn of events.

  And then came the lesson . I’ve taken with me through my life:”Honey, Your basket is extra-special,” Mom said, gently wiping away my hot tears.”Your basket is special because you paid for it yourself.”

  55.What can we learn from the first two paragraphs?

  A. The children enjoyed doing housework.

  B.The author came from s well-off family

  C. The mother raised her children in an unusual way

  D.The children were fond of the US president’s daughters.

  56.When the author saw the basket in the window,she

  .

  A. fell in love with it

  B. stared at her mother

  C. recognized it at once

  D.went up to the bike guy

  57.Why did the author say many “please” to her mother?

  A. She longed to do extra work.

  B. She was eager to have the basket.

  C.She felt tired after standing too long.

  D.She wanted to be polite to her mother.

  58.By using“naked” (Paragraph 12),the author seems to stress that the basket was

  A..something she could afford

  B.something important to her

  C.something impossible to get

  D.something she could do without

  59.To the author, it seemed to be a horrible turn of events that

  A. something spoiled her paying plan

  B. the basket cost more than she had saved

  C.a neighborhood girl had bought a new bike

  D.someone else had got a basket of the same kind

  60.What is the life lesson the author learned from her mother?

  A. Save money for a rainy day

  B. Good advice is beyond all price.

  C. Earn your bread with your sweat

  D. God helps those who help themselves

  5.【2016·江苏】D

  Not so long ago, most people didn’t know who Shelly Ann Francis Pryce was going to become. She was just an average high school athlete. There was every indication that she was just another American teenager without much of a future. However, one person wants to change this. Stephen Francis observed then eighteen-year-old Shelly Ann as a track meet and was convinced that he had seen the beginning of true greatness. Her time were not exactly impressive, but even so, he seemed there was something trying to get out, something the other coaches had overlooked when they had assessed her and found her lacking. He decided to offer Shelly Ann a place in his very strict training seasons. Their cooperation quickly produced results, and a few year later at Jamaica’s Olympic games in early 2008, Shelly Ann, who at that time only ranked number 70 in the world, beat Jamaica’s unchallenged queen of the sprint(短跑).

  “Where did she come from?” asked an astonished sprinting world, before concluding that she must be one of those one-hit wonders that spring up from time to time, only to disappear again without signs. But Shelly Ann was to prove that she was anything but a one-hit wonder. At the Beijing Olympic she swept away any doubts about her ability to perform consistently by becoming the first Jamaican woman ever to win the 100 meters Olympic gold. She did it again one year on at the World Championship in Briton, becoming world champion with a time of 10.73--- the fourth record ever.

  Shelly-Ann is a little woman with a big smile. She has a mental toughness that did not come about by chance. Her journey to becoming the fastest woman on earth has been anything but smooth and effortless. She grew up in one of Jamaica’s toughest inner-city communities known as Waterhouse, where she lived in a one-room apartment, sleeping four in a bed with her mother and two brothers. Waterhouse, one of the poorest communities in Jamaica, is a really violent and overpopulated place. Several of Shelly-Ann's friends and family were caught up in the killings; one of her cousins was shot dead only a few streets away from where she lived. Sometimes her family didn’t have enough to eat. She ran at the school championships barefooted because she couldn’t afford shoes. Her mother Maxime, one of a family of fourteen, had been an athlete herself as a young girl but, like so many other girls in Waterhouse, had to stop after she had her first baby. Maxime’s early entry into the adult world with its responsibilities gave her the determination to ensure that her kids would not end up in Waterhouse's roundabout of poverty. One of the first things Maxime used to do with Shelly-Ann was taking her to the track, and she was ready to sacrifice everything.

  It didn't take long for Shelly-Ann to realize that sports could be her way out of Waterhouse. On a summer evening in Beijing in 2008, all those long, hard hours of work and commitment finally bore fruit. The barefoot kid who just a few years previously had been living in poverty, surrounded by criminals and violence, had written a new chapter in the history of sports.

  But Shelly-Ann’s victory was far greater than that. The night she won Olympic gold in Beijing, the routine murders in Waterhouse and the drug wars in the neighbouring streets stopped. The dark cloud above one of the world’s toughest criminal neighbourhoods simply disappeared for a few days. “ I have so much fire burning for my country,”Shelly said. She plans to start a foundation for homeless children and wants to build a community centre in Waterhouse. She hopes to inspire the Jamaicans to lay down their weapons. She intends to fight to make it a woman’s as well as a man’s world.

  As Muhammad Ali puts it, “ Champions aren't made in gyms. Champions are made from something they have deep inside them. A desire, a dream, a vision.” One of the things Shelly-Ann can be proud of is her understanding of this truth.

  65. Why did Stephen Francis decide to coach Shelly-Ann?

  A. He had a strong desire to free her family from trouble.

  B. He sensed a great potential in her despite her weaknesses.

  C. She had big problems maintaining her performance.

  D. She suffered a lot of defeats at the previous track meets.

  66. What did the sprinting world think of Shelly-Ann before the 2008 Olympic Games?

  A. She would become a promising star.

  B. She badly needed to set higher goals.

  C. Her sprinting career would not last long.

  D. Her talent for sprinting was known to all.

  67. What made Maxime decide to train her daughter on the track?

  A. Her success and lessons in her career.

  B. Her interest in Shelly-Ann’s quick profit.

  C. Her wish to get Shelly-Ann out of poverty.

  D. Her early entrance into the sprinting world.

  68. What can we infer from Shelly-Ann's statement underlined in Paragraph 5?

  A. She was highly rewarded for her efforts.

  B. She was eager to do more for her country.

  C. She became an athletic star in her country.

  D. She was the envy of the whole community.

  69. By mentioning Muhammad Ali’s words, the author intends to tell us that .

  A. players should be highly inspired by coaches

  B. great athletes need to concentrate on patience

  C. hard work is necessary in one’s achievements

  D. motivation allows great athletes to be on the top

  70. What is the best title for the passage?

  A. The Making of a Great Athlete

  B. The Dream for Championship

  C. The Key to High Performance

  D. The Power of Full Responsibility

  2015年高考试题

  专题13人物传记、故事类

  1.【2015·湖北卷】B

  What Theresa Loe is doing proves that a large farm isn’t prerequisite for a modern grow-your-own lifestyle. On a mere 1/10 of an acre in Los Angeles, Loe and her family grow, can(装罐)and preserve much of the food they consume.

   Loe is a master food preserver, gardener and canning expert. She also operates a website, where she shares her tips and recipes, with the goal of demonstrating that every has the ability to control what’s on their plate.

   Loe initially went to school to become an engineer, but she quickly learned that her enthusiasm was mainly about growing and preparing her own food. “I got into cooking my own food and started growing my own herbs (香草) and foods for that fresh flavor,”she said. Engineer by day, Loe learned cooking at night school. She ultimately purchased a small piece of land with her husband and began growing their own foods.

  “I teach people how to live farm-fresh without a farm,” Loe said. Through her website Loe emphasizes that “anybody can do this anywhere.” Got an apartment with a balcony (阳台)? Plant some herbs. A window? Perfect spot for growing. Start with herbs, she recommends, because “they’re very forgiving.” Just a little of the herbs “can take your regular cooking to a whole new level,” she added. “I think it’s a great place to start.” “Then? Try growing something from a seed, she said, like a tomato or some tea.”

  Canning is a natural extension of the planting she does. With every planted food. Loe noted, there’s a moment when it’s bursting with its absolute peak flavor. “I try and keep it in a time capsule in a canning jar,” Loe said. “Canning for me is about knowing what’s in your food, knowing where it comes from.”

  In addition to being more in touch with the food she’s eating, another joy comes from passing this knowledge and this desire for good food to her children: “Influencing them and telling them your opinion on not only being careful what we eat but understanding the bigger picture,” she said, “that if we don’t take care of the earth, no will.”

  55.The underlined word 損rerequisite?(Pare. 1) is closest in meaning to 揰_____? her

  A. recipe

  B. substitute

  C. requirement

  D. challenge

  56.Why does Loe suggest starting with herbs?

  A. They are used daily.

  B. They are easy to grow.

  C. They can grow very tall

  D. They can be eaten uncooked

  57.According to Loe, what is the benefit of canning her planted foods?

  A. It can preserve their best flavor

  B. It can promote her online sales

  C. It can better her cooking skills

  D. It can improve their nutrition

  58.What is the搕he bigger picture?(Para. 6) that Loe wishes her children to understand?s

  A. The knowledge about good food

  B. The way to live a grow-our-own life

  C. The joy of getting in touch with foods 

  D. The responsibility to protect our earth

  2.【2015·北京卷】A

  The Boy Made It!

  One Sunday, Nicholas, a teenager, went skiing at Sugarloaf Mountain in Maine. In the early afternoon, when he was planning to go home, a fierce snowstorm swept into the area. Unable to see far, he accidentally turned off the path. Before he knew it, Nicholas was lost, all alone! He didn’t have food, water, a phone, or other supplies. He was getting colder by the minute.

  Nicholas had no idea where he was. He tried not to panic. He thought about all the survival shows he had watched on TV. It was time to put the tips he had learned to use.

  He decided to stop skiing. There was a better chance of someone finding him if he stayed put. The first thing he did was to find shelter form the freezing wind and snow. If he didn’t, his body temperature would get very low, which could quickly kill him.

  Using his skis, Nicholas built a snow cave. He gathered a huge mass of snow and dug out a hole in the middle. Then he piled branches on top of himself, like a blanket, to stay as warm as he could.

  By that evening, Nicholas was really hungry. He ate snow and drank water from a nearby stream so that his body wouldn’t lose too much water. Not knowing how much longer he could last, Nicholas did the only thing he could- he huddled(蜷缩) in his cave and slept.

  The next day, Nicholas went out to look for help, but he couldn’t find anyone. He followed his tracks and returned to the snow cave, because without shelter, he could die that night. On Tuesday, Nicholas went out to find help. He had walked for about a mile when a volunteer searcher found him. After two days stuck in the snow, Nicholas was saved.

  Nicholas might not have survived this snowstorm had it not been for TV. He had often watched Grylls’ survival show. Man vs. Wild. That’s where he learned the tips that saved his life, In each episode(一期节目)of Man vs. Wild, Grylls is abandoned in a wild area and has to find his way out.

  When Grylls heard about Nicholas’ amazing deeds, he was super impressed that Nicholas had made it since he knew better than anyone how hard Nicholas had to work to stay alive.

  56. What happened to Nicholas one Sunday afternoon?

  A. He got lost.

  B. He broke his skis.

  C. He hurt his eyes

  D. He caught a cold

  57. How did Nicholas keep himself warm?

  A. He found a shelter.

  B. He lighted some branches.

  C. He kept on skiing.

  D. He built a snow cave.

  58. On Tuesday, Nicholas _____.

  A. returned to his shelter safely

  B. was saved by a searcher

  C. got stuck in the snow

  D. staved where he was

  59. Nicholas left Grylls a very deep impression because he _____.

  A. did the right things in the dangerous situation

  B. watched Grylls’ TV program regularly

  C. created some tips for survival

  D. was very hard-working

  3.【2015·广东】A

  Peter loved to shop used articles. Almost a month ago, he bought popular word game that used little pieces of wood with different letters on them. As he was purchasing it, the salesgirl said, “Uh, look, the game box haven’t even been opened yet. That might be worth some money. ”

  Peter examined the box, and, sure enough, it was completely covered in factory-sealed plastic. And he saw a date of 1973 on the back of the box.

  “You should put that up for auction (拍卖) on the Internet, and see what happens.”the salesgirl said.

  “Yes, you’re right. People like something rare.” Peter agreed, “I can’t imagine there being very many unopened boxes of this game still around 40 years later.”

  “Don’t forget to tell me if you sell it.” the salesgirl smiled.

  “No problem.” Peter said.

  After he got home, Peter went online to several auction websites looking for his game. But he couldn’t find it. Then he typed in the name of the word game and hit Search. The search result was 543 websites containing information about the changes of the game. Over the years, the game had been produced using letters in different sizes and game boards in different colors. He also found some lists of game fans looking for various versions of the game. Peter emailed some of them, telling them what he had.

  Two weeks later, Peter went back to the shop.

  “Hello. Do you still remember the unopened word game?”

  The salesgirl looked at him for a second, then recognized him and said, “Oh, hi!”

  “I’ve got something for you,” Peter said. “I sold the game and made $1,000. Thank you for your suggestion.” He handed her three $ 100 bills.

  “Wow!” the salesgirl cried out. “Thank you, I never expected it.”

  26. Which of the following best describes Peter’s word game?

  A. It was made around 40 years ago.

  B. It had game boards in different sizes.

  C. It was kept in a plastic bag with a seal.

  D. It had little pieces of wood in different colors.

  27.

  What did the salesgirl probably think of Peter’s word game?

  A. Old and handy.

  B. Rare and valuable.

  C. Classic and attractive.

  D. Colorful and interesting

  28.

  Peter got the names of the game fans from _________.

  A. an auction

  B. the Internet

  C. a game shop

  D. the second-hand shop

  29. What happened at the end of the story?

  A. Peter gave the girl $300 as a reward.

  B. The salesgirl became Peter’s friend.

  C. Peter returned the word game for $ 1,000.

  D. The salesgirl felt confused to see Peter again.

  30.

  What is the main theme of the story?

  A. It’s important to keep a promise.

  B. It’s great to share in other people’s happiness.

  C. We should be grateful for the help from others.

  D. Something rare is worth a large amount of money.

  4.【2015·陕西】B

  When the dog named Judy spotted the first sheep in her life, she did what comes naturally. The four-year-old dog set off racing after the sheep across several fields and, being a city animal, lost both her sheep and her sense of direction. Then she ran along the edge of cliff( 悬崖) and fell 100 feet, bouncing off a rock into the sea.

  Her owner Mike Holden panicked and celled the coastguard of Cornwall, who turned up in seconds . Six volunteers slid down the cliff with the help of a rope but gave up all hope of finding her alive after a 90-minute search.

  Three days later, a hurricane hit the coast near Cornwall. Mr. Holden returned home from his holiday upset and convinced his pet was dead. He comforted himself with the thought she had died in the most beautiful part of the country.

  For the next two weeks, the Holdens were heartbroken . Then, one day, the phone rang and Steve Tregear, the coastguard of Cornwall, asked Holder if he would like his dog bark.

  A birdwatcher, armed with a telescope, found the pet sitting desperately on a rock. While he sounded the alarm, a student from Leeds climbed down the cliff to collect Judy.

  The dog had initially been knocked unconscious(失去知觉的)but had survived by drinking water from a fresh scream at the base of the cliff. She may have fed on the body of a sheep which had also fallen over the edge. “The dog was very thin and hungry,” Steve Tregear said , “It was a very dog. She survived because of a plentiful supply of fresh water,” he added.

  It was ,as Mr. Holden admitted, “a minor miracle(奇迹)”.

  50.

  The dog Jody fell down the cliff when she was

  A. rescuing her owner

  B. caught in a hurricane

  C. blocked by a rock

  D. running after a sheep

  51. Who spotted Judy after the accident?

  A. A birdwatcher

  B. A student from Leeds

  C. Six volunteers

  D. The coastguard of Cornwall

  52. What can we infer from the text?

  A. People like to travel with their pets.

  B. Judy was taken to the fields for hunting.

  C. Luck plays a vital role in Judy's survival.

  D. Holden cared little where Judy was buried.

  53. Which of the following can be the best title for the text?

  A. Miracle of the Coastguard.

  B. Surviving a Hurricane.

  C. Dangers in the Wild

  D. Coming Back from the Dead.

  5.【2015·四川】B

  Nothing could stop Dad. After he was put on disability for a bad back, he bought a small farm in the country, just enough to grow food for the family. He planted vegetables, fruit trees and even kept bees for honey.

  And every week he cleaned Old Man McColgin's chicken house in exchange for manure(肥料). The Smell really burned the inside of your nose. When we complained about the terrible smell, Dad said the stronger the manure, the healthier the crops, and he was right. For example, just one of his cantaloupes filled the entire house with its sweet smell, and the taste was even sweeter.

  As the vegetables started coming in, Dad threw himself into cooking. One day, armed with a basket of vegetables, he announced he was going to make stew(炖菜).Dad pulled out a pressure cooker and filled it up with cabbages, eggplants, potatoes, corns, onions and carrots. For about half an hour. the pressure built and the vegetables cooked. Finally, Dad turned off the stove, the pot began to cool and the pressure relief valve sprayed out a cloud of steam. If we thought Dad's pile of chicken manure was bad, this was 10 times worse. When Dad took off the lid, the smell nearly knocked us out.

  Dad carried the pot out and we opened doors and windows to air out the house. Just how bad was it? The neighbors came out of their houses to see if we had a gas leak!

  Determined, Dad filled our plates with steaming stew and passed them around. It didn’t look that bad, and after the first wave had shut down my ability to smell, it didn’t offend the nose so much, either. I took a taste. It would never win a prize in a cooking competition, but it was surprisingly edible and we drank up every last drop of soup.

  34.Why did Dad clean Old Man Mocolgin’s chicken house regularly?

  A. To earn some money for the family.

  B. To collect manure for his crops.

  C. To get rid of the terrible smell.

  D. To set a good example to us.

  35.What can we infer about Dad抯 stew?g

  A. It is popular among the neighbors.

  B. It contains honey and vegetables.

  C. It looks very wonderful.

  D. It tastes quite delicious.

  36.What does the underlined word 搊ffend?in the last paragraph mean?ri

  A. To attract

  B. To upset

  C. To air

  D. To shut

  37.What can we learn about Dad form the text?

  A. He is an experienced cook.

  B. He is a troublesome father.

  C. He has a positive attitude to life.

  D. He suffers a lot from his disability.

  6.【2015·天津】C

  One day when I was 12, my mother gave me an order: I was to walk to the public library, and borrow at least one book for the summer. This was one more weapon for her to defeat my strange problem — inability to read.

  In the library, I found my way into the “Children’s Room.” I sat down on the floor and pulled a few books off the shelf at random. The cover of a book caught my eye. It presented a picture of a beagle. I had recently had a beagle, the first and only animal companion I ever had as a child. He was my secret sharer, but one morning, he was gone, given away to someone who had the space and the money to care for him. I never forgot my beagle.

  There on the book’s cover was a beagle which looked identical(相同的) to my dog. I ran my fingers over the picture of the dog on the cover. My eyes ran across the title, Amos, the Beagle with a Plan. Unknowingly, I had read the title. Without opening the book, I borrowed it from the library for the summer.

  Under the shade of a bush, I started to read about Amos. I read very, very slowly with difficulty. Though pages were turned slowly, I got the main idea of the story about a dog who, like mine, had been separated from his family and who finally found his way back home. That dog was my dog, and I was the little boy in the book. At the end of the story, my mind continued the final scene of reunion, on and on, until my own lost dog and I were, in my mind, running together.

  My mother’s call returned me to the real world. I suddenly realized something: I had read a book, and I had loved reading that book. Everyone knew I could not read. But I had read it. Books could be incredibly wonderful and I was going to read them.

  I never told my mother about my “miraculous” (奇迹般地) experience that summer, but she saw a slow but remarkable improvement in my classroom performance during the next year. And years later, she was proud that her son had read thousands of books, was awarded a PhD in literature, and authored his own books, articles, poetry and fiction. The power of the words has held.

  46.

  The author’s mother told him to borrow a book in order to_____.

  A. encourage him to do more walking

  B. let him spend a meaningful summer

  C. help cure him of his reading problem

  D. make him learn more about weapons

  47. The book caught the author’s eye because_____.

  A. it contained pretty pictures of animals

  B. it reminded him of his own dog

  C. he found its title easy to understand

  D. he liked children’s stories very much

  48.

  Why could the author manage to read the book through?

  A. He was forced by his mother to read it.

  B. He identified with the story in the book.

  C. The book told the story of his pet dog.

  D. The happy ending of the story attracted him.

  49. What can be inferred from the last paragraph?

  A. The author has become a successful writer.

  B. The author’s mother read the same book.

  C. The author’s mother rewarded him with books.

  D. The author has had happy summers ever since.

  50.

  Which one could be the best title of the passage?

  A. The Charm of a Book

  B. Mum’s Strict Order

  C. Reunion with My Beagle

  D. My Passion for Reading

  7.【2015·重庆】A

  At thirteen, I was diagnosed(诊所) with

  kind of attention disorder. It made school difficult for me. When everyone else in the class was focusing on tasks, I could not.

  In my first literature class, Mrs.Smith asked us to read a story and then write on it, all within 45 minutes. I raised my hand right away and said,“Mrs.Smith, you see, the doctor said I have attention problems. I might not be able to do it.”

  She glanced down at me through her glasses, “you are no different from your classmates, young man.”

  I tried, but I didn’t finish the reading when the bell rang. I had to take it home.

  In the quietness of my bedroom, the story suddenly all became clear to me. It was about a blind person, Louis Braille. He lived in a time when the blind couldn’t get much education. But Louis didn’t give up. Instead, he invented a reading system of raised dots(点), which opened up a whole new world of knowledge to the blind.

  Wasn’t I the “blind” in my class, being made to learn like the “sighted” students? My thoughts spilled out and my pen started to dance. I completed the task within 40 minutes. Indeed, I was no different from others; Ijust needed a quieter place. If Louis could find his way out of his problems, why should I ever give up?

  I didn’t expect anything when I handled in my paper to Mrs.Smith, so it was quite a surprise when it came back to me the next day- with an“A” on it. At the bottom of the paper were these words:“ See what you can do when you keep trying?”

  36. The author didn’t finish the reading in class because.

  A. He was new to the class

  B. He was tried of literature

  C. He had an attention disorder

  D. He wanted to take the task home

  37. What do we know about Louis Braille from the passage?

  A. He had good sight

  B. He made a great invention.

  C. He gave up reading

  D. He learned a lot from school

  38. What was Mrs.Smith ‘s attitude to the author at the end of the story?

  A. Angry

  B. Impatient

  C. Sympathetic

  D. Encouraging

  38. What is the main idea of the passage?

  A.The disabled should be treated with respect.

  B.A teacher can open up a new world to students.

  C. One can find his way out of difficulties with efforts.

  D. Everyone needs a hand when faced with challenges.

  8.【2015·安徽】B

  When her five daughters were young, Helene An always told them that there was strength in unity (团结). To show this, she held up one chopstick, representing one person. Then she easily broke it into two pieces. Next, she tied several chopsticks together, representing a family. She showed the girls it was hard to break the tied chopsticks. This lesson about family unity stayed with the daughters as they grew up.

  Helene An and her family own a large restaurant business in California. However, when Helene and her husband Danny left their home in Vietnam in 1975, they didn't have much money. They moved their family to San Francisco. There they joined Danny's mother, Diana, who owned a small Italian sandwich shop. Soon afterwards, Helene and Diana changed the sandwich shop into a small Vietnamese restaurant. The five daughters helped in the restaurant when they were young. However, Helene did not want her daughters to always work in the family business because she thought it was too hard.

  Eventually the girls all graduated from college and went away to work for themselves, but one by one, the daughters returned to work in the family business. They opened new restaurants in San Francisco and Los Angeles. Even though family members sometimes disagreed with each other, they worked together to make the business successful. Daughter Elisabeth explains, "Our mother taught us that to succeed we must have unity, and to have unity we must have peace. Without the strength of the family, there is no business."

  Their expanding business became a large corporation in 1996, with three generations of Ans working together. Now the Ans' corporation makes more than $20 million each year. Although they began with a small restaurant, they had big dreams, and they worked together. Now they are a big success.

  60. Helene tied several chopsticks together to show ______.

  A. the strength of family unity

  B. the difficulty of growing up

  C. the advantage of chopsticks

  D. the best way of giving a lesson

  61. We can I earn from Paragraph 2 that the An family ______.

  A. started a business in 1975

  B. left Vietnam without much money

  C. bought a restaurant in San Francisco

  D. opened a sandwich shop in Los Angeles

  62. What can we infer about the An daughters?

  A. They did not finish their college education.

  B. They could not bear to work in the family business.

  C. They were influenced by what Helene taught them.

  D. They were troubled by disagreement among family members.

  63. Which of the following can be the best title for the passage?

  A. How to Run a Corporation

  B. Strength Comes from Peace

  C. How to Achieve a Big Dream

  D. Family Unity Builds Success

  9.【2015·新课标全国II】A

  My color television has given me nothing but a headache. I was able to buy it a little over a year ago because I had my relatives give me money for my birthday instead of clothes that wouldn’t fit. I let a salesclerk fool me into buying a discontinued model. I realized this a day later, when I saw newspaper advertisements for the set at seventy-five dollars less than I had paid. The set worked so beautiful when I first got it home that I would keep it on until stations signed off for the night. Fortunately, I didn’t get any channels showing all-night movies or I would never have gotten to bed.

  Then I started developing a problem with the set that involved static(静电) noise. For some reason, when certain shows switched into a commercial, a loud noise would sound for a few seconds. Gradually, this noise began to appear during a show, and to get rid of it, I had to change to another channel and then change it back. Sometimes this technique would not work, and I had to pick up the set and shake it to remove the sound. I actually began to build up my arm muscles(肌肉) shaking my set.

  When neither of these methods removed the static noise, I would sit helplessly and wait for the noise to go away. At last I ended up hitting the set with my first, and it stopped working altogether. My trip to the repair shop cost me $62, and the sit is working well now, but I keep expecting more trouble.

  21. Why did the author say he was fooled into buying the TV set?

  He got an older model than he had expected.

  He couldn’t return it when it was broken.

  He could have bought it at a lower price.

  He failed to find any movie shows on it.

  22.

  Which of the following an best replace the phrase “signed off” in paragraph 1?

  ended all their programs

  provided fewer channels

  changed to commercials

  showed all-night movies

  23.

  How did the author finally get this TV set working again?

  By shaking and hitting it

  By turning it on and off

  By switching channels

  By having it repaired

  24.

  How does the author sound when telling the story?

  Curious

  Anxious

  Cautious

  Humorous

  10.【2015·福建】B

  Papa, as a son of a dirt-poor farmer, left school early and went to work in a factory, for education was for the rich then. So, the world became his school. With great interest, he read everything he could lay his hands on, listened to the town elders and learned about the world beyond his tiny hometown. "There's so much to learn," he'd say. "Though we're born stupid, only the stupid remain that way. " He was determined that none of his children would be denied ( fll^fe) an education.

  Thus, Papa insisted that we learn at least one new thing each day. Though, as children, we thought this was crazy, it would never have occurred to us to deny Papa a request. And dinner time seemed perfect for sharing what we had learned. We would talk about the news of the day; no matter how insignificant, it was never taken lightly. Papa would listen carefully and was ready with some comment, always to the point. Then came the moment—the time to share the day's new learning.

  Papa, at the head of the table, would push back his chair and pour a glass of red wine, ready to listen. "Felice," he'd say, "tell me what you learned today. " "I learned that the population of Nepal is .... " Silence.

  Papa was thinking about what was said, as if the salvation of the world would depend upon it. "The population of Nepal. Hmm. Well . . . . " he'd say. "Get the map; let's see wrhere Nepal is. " And the wThole family

  went on a search for Nepal.

  This same experience was repeated until each family member had a turn. Dinner ended only after we had a clear understanding of at least half a dozen such facts.

  As children, we thought very little about these educational wonders. Our family, however, was growing together, sharing experiences and participating in one another's education. And by looking at us, listening to us, respecting our input, affirming our value, giving us a sense of dignity, Papa was unquestionably our most influential teacher.

  Later during my training as a future teacher /1 studied with some of the most famous educators. They were imparting what Papa had known all along—the value of continual learning. His technique has served me well all my life. Not a single day has been wasted, though I can never tell when knowing the population of Nepal might prove useful.

  60.What do we know from the first paragraph?

  A. The author's father was born in a worker's family.

  B. Those born stupid could not change their life.

  C. The town elders wanted to learn about the world.

  D. The poor could hardly afford school education.

  61.The underlined word "it" in the second paragraph refers to "

  A. one new thing B. a request C. the news D. some comment

  62.It can be learned from the passage that the author .

  A. enjoyed talking about news

  B. knew very well about Nepal

  C. felt regret about those wasted days

  D . appreciated his father's educational technique

  63What is the greatest value of "dinner time" to the author?

  A. Continual learning. B. Showing talents.

  C. Family get-together. D. Winning Papa's approval.

  64.The author's father can be best described as .

  A. an educator expert at training future teachers

  B. a parent insistent on his children's education

  C. a participant willing to share his knowledge

  D. a teacher strict about everything his students did

  【2015·新课标全国I】C

  Salvador Dali (1904-1989) was one of the most popular of modern artists. The Pompidou Centre in Paris is showing its respect and admiration for the artist and his powerful personality with an exhibition bringing together over 200 paintings, sculptures, drawings and more. Among the works and masterworks on exhibition the visitor will find the best pieces, most importantly The Persistence of Memory. There is also L’Enigme sans Fin from 1938, works on paper, objects, and projects for stage and screen and selected parts from television programmes reflecting the artist’s showman qualities.

  The visitor will enter the World of Dali through an egg and is met with the beginning, the world of birth. The exhibition follows a path of time and subject with the visitor exiting through the brain.

  The exhibition shows how Dali draws the viewer between two infinities (无限). “From the infinity small to the infinity large, contraction and expansion coming in and out of focus: amazing Flemish accuracy and the showy Baroque of old painting that he used in his museum-theatre in Figueras,” explains the Pompidou Centre.

  The fine selection of the major works was done in close collaboration (合作)with the Museo Nacional Reina Sofia in Madrid, Spain, and with contributions from other institutions like the Salvador Dali Museum in St. Petersburg, Florida.

  28.

  Which of the following best describe Dali according to Paragraph 1?

  A. Optimistic.

  B. Productive.

  C. Generous.

  D. Traditional.

  29.

  What is Dali’s The Persistence of Memory considered to be?

  A. One of his masterworks.

  B. A successful screen adaptation.

  C. An artistic creation for the stage.

  D. One of the best TV programmes.

  30.

  How are the exhibits arranged at the World of Dali?

  A. By popularity.

  B. By importance.

  C. By size and shape.

  D. By time and subject.

  31.

  What does the word “contributions” in the last paragraph refer to?

  A. Artworks.

  B. Projects.

  C. Donations.

  D. Documents.

  2017年高考试题

  人物故事

  【2017·四川卷】C

  A schoolgirl saved her father's life by kicking him in the chest after he suffered a serious allergic (过敏的) reaction which stopped his heart.

  Izzy, nine, restarted father Colm's heart by stamping (踩) on his chest after he fell down at home and stopped breathing.

  Izzy's mother, Debbie, immediately called 999 but Izzy knew doctors would never arrive in time to save her father, so decided to use CPR.

  However, she quickly discovered her arms weren't strong enough, so she stamped on her father's chest .Debbie then took over with some more conventional chest compressions (按压) until the ambulance

  Izzy, who has been given a bravery award by her school, said: "I just kicked him really hard. My mum taught me CPR but I knew I wasn't strong enough to use hands. I was quite scared. The doctor said I might as well be a doctor or a nurse. My mum said that Dad was going to hospital with a big footprint on his

  "She's a little star," said Debbie, "i was really upset but Izzy just took over. I just can't believe what she did. I really think all children should be taught first aid. Izzy did CPR then the doctor turned up. Colm had to have more treatment on the way to the hospital and we've got to see an expert."

  Truck driver Colm, 35, suffered a mystery allergic reaction on Saturday and was taken to hospital, but was sent home only for it to happen again the next day. The second attack was so serious that his airway swelled, preventing him from breathing, his blood pressure dropped suddenly, and his heart stopped for a moment.

  He has now made a full recovery from his suffering.

  . Izzy kicked her father in the chest ______ .

  A. to express her helplessness                             B. to practise CPR on him

  C. to keep him awake                                         D. to restart his heart

  . What's the right order of the events?

  1Izzy kicked Colm.

  2Debbie called 999.

  3Izzy learned CPR.

  4Colm's heart stopped.

  A. 3124                  B. 4231                C. 3421         D. 4312

  What does Paragraph 8 mainly talk about?

  A. What Colm suffered.                                     

  B. Colm's present condition.

  C. What caused Colm's allergy.                           

  D. Symptoms of Colm's allergic reaction.

  . Why does the author write the news?

  A. To describe a serious accident.                       

  B. To prove the importance of CPR.

  C. To report a 9-year-old girl's brave act.      

  D. To call people's attention to allergic reaction.

  【2017·四川卷】B

  In 1943, when I was 4, my parents moved from Coeur d'Alene, Idaho, to Fairbanks, Alaska, where adventure was never very far away.

  We arrived in the summer, just in time to enjoy the midnight sun. All that sunlight was fantastic for Mom's vegetable garden. Working in the garden at midnight tended to throw her timing off, so she didn't care much about my bedtime.

  Dad was a Railway Express agent and Mom was his clerk. That left me in a mess. I usually managed to find some trouble to get into. Once I had a little Are going in the dirt basement of a hotel. I had tried to light a barrel(桶) of paint but couldn't really get a good fire going. The smoke got pretty bad, though, and when 1 made my exit, a crowd and the police were there to greet me. The policemen took my matches and drove me

  Mom and Dad were occupied in the garden and Dad told the police to keep me, and they did! I had a tour of the prison before Mom rescued me. 1 hadn't turned 5 yet.

  As I entered kindergarten, the serious cold began to set in. Would it surprise you to know that I soon left part of my tongue on a metal handrail at school?

  As for Leonhard Seppala, famous as a dog sledder (驾雪橇者), I think I knew him well because I was taken for a ride with his white dog team one Sunday. At the time I didn't realize what a superstar he was, but I do remember the ride well. I was wrapped (包裹) heavily and well sheltered from the freezing and blowing weather.

  In 1950, we moved back to Coeur d'Alene, but we got one more Alaskan adventure when Leonhard invited us eight years later by paying a visit to Idaho to attend a gathering of former neighbors of Alaska.

  . What can be inferred about the author's family?

  A. His father was a cruel man.                      B. His parents didn't love him.

  C. His parents used to be very busy.                     D. His mother didn't have any jobs.

  . What happened when the author was 4?

  A. He learned to smoke.                                      B. He was locked in a basement.

  C. He was arrested by the police.                         D. He nearly caused a fire accident.

  . Which of the following is true?

  A. Leonhard was good at driving dog sleds.

  B. The author spent his whole childhood in Alaska.

  C. Leonhard often visited the author's family after 1950.

  D. The author suffered a lot while taking the dog sled in Alaska.

  . What is the author's purpose of writing the text?

  A. To look back on his childhood with adventures.

  B. To describe the extreme weather of Alaska.

  C. To express how much he misses Leonhard.

  D. To show off his pride in making trouble.

  【2017·全国新课标II】A

  Arriving in Sydney on his own from India, my husband ,Rashid, stayed in a hotel for a short time while looking for a house for me and our children.

  During the first week of his stay, he went out one day to do some shopping. He came back in the late afternoon to discover that his suitcase was gone. He was extremely worried as the suitcase had all his important papers, including his passport.

  He reported the case to the police and then sat there,lost and lonely in strange city, thinking of the terrible troubles of getting all the paperwork organized again from a distant country while trying to settle down in a new one.

  Late in the evening, the phone rang. It was a stranger. He was trying to pronounce my husband’s name and was asking him a lot of questions. Then he said they had found a pile of papers in their trash can(垃圾桶)that had been left out on the footpath.

  My husband rushed to their home to find a kind family holding all his papers and documents. Their young daughter had gone to the trash can and found a pile of unfamiliar papers. Her parents had carefully sorted them out, although they had found mainly foreign addresses on most of the documents. At last they had seen a half-written letter in the pile in which my husband had given his new telephone number to a friend.

  That family not only restored the important documents to us that day but also restored our faith and trust in people. We still remember their kindness and often send a warm wish their way.

  .What did Rashid plan to do after his arrival in Sydney?

  A. Go shopping

  B. Find a house

  C. Join his family

  D. Take his family

  .The girl’s parents got Rashid’s phone number from_______.

  A. a friend of his family

  B. a Sydney policeman

  C. a letter in his papers

  D. a stranger in Sydney

  .What does the underlined word “restored” in the last paragraph mean?

  A. Showed

  B. Sent out

  C. Delivered

  D. Gave back

  .Which of the following can be the best title for the text?

  A. From India to Australia.

  B. Living in

  a New Country.

  C. Turning Trash to Treasure.

  D. In Search of New Friends.

  【2017·陕西卷】B

  When I told my father that I was moving to Des Moines, Iowa, he told me about the only time he had been there. It was in the 1930s, when he was an editor if the literary magazine of Southern Methodist University(SMU)in Dallas, Texas. He also worked as a professor at SMU, and there was a girl student in his class who suffered from a serious back disease. She couldn;t afford the operation because her family was poor.

  Her mother ran a boardinghouse in Galveston, a seaside town near Houston, Texas. She was cleaning out the attic(阁楼)one day when she came across an old dusty manuscript(手稿). On its top page were the words, “By O. Henry”. It was a nice story, and she sent it to her daughter at SMU, who showed it to my father. My father had never read the story before, but it sounded like O. Henry, and he knew that O. Henry had once lived in Houston. So it was possible that the famous author had gone to the beach and stayed in the Gainestown boardinghouse, and had written the story there and left the manuscript behind by accident. My father visited an O. Henry expert at Columbia University in New York, who authenticated the story as O. Henry’s.

  My father then set out to sell it. Eventfully, he foud himself in Des Moines, meeting with Gardner Cowles, a top editor at the Des Moines Register. Cowles loves the story and bought it on the spot. My father took the money to the girl. It was just enough for her to have the operation she so desperately needed.

  My father never told me what the O. Henry story was about. But i doubt that it could have been better than his own story.

  Who found the O. Henry’s manuscript?

  A. The girl’s mother.

  B. The author’s father.

  C. The girl.

  D. The author.

  Which of the following might explain the fact that the manuscript was found in the attic?

  A. O. Henry once worked in Houston.

  B. O. Henry once stayed in Galveston.

  C. O. Henry once moved to Des Moines.

  D. O. Henry once taught at SMU.

  The underlined word “authenticated” in Paragraph 2 probably means __________.

  A. named

  B. treated

  C. proved

  D. described

  According to the text, why did the author’s father go to Des Moines?

  A. To sell the O. Henry story.

  B. To meet the author himself.

  C. To talk with the O. Henry expert.

  D. To give money to the girl.

  【2017·天津卷】C

  “Dad,” I say one day …..take a trip. Why don’t you fly and meet me?”

  My father had just retired……….. His job filled his day, his thought, his life. While he woke up and took a warm shower, I screamed under a freezing waterfall Peru. While he tied a tie and put on the same Swiss watch, I rowed a boat across Lake of the Ozarks.

  My father sees me drifting aimlessly, nothing to show for my 33 years but a passport full of funny stamps. He wants me to settle down, but now I want him to find an adventure.

  He agrees to travel with me through the national parks. We meet four weeks later in Rapid City.

  “ What is our first stop?” asks my father.

  “What time is it?”

  “Still don’t have a watch?”

  Less than an hour away is Mount Rushmore. As he stares up at the four Presidents carved in granite(), his mouth and eyes open slowly, like those of little boy.

  “Unbelievable,” he says, “How was this done?”

  A film in the information center shows sculptor Gutzon Borglum devoted 14 years to the sculpture and then left the final touches to his son.

  We stare up and I ask myself, Would I ever devote my life to anything?

  No directions, …… I always used to hear those words in my father’s voice. Now I hear them in my own.

  The next day we’re at Yellowstone National Park, where we have a picnic.

  “Did you ever travel with your dad? I ask.

  “Only once,” he says. “ I never spoke much with my father. We loved each other---but never said it. Whatever he could give me, he gave.”>

  The kast sebtebce----it’s probably the same thing I’s say about my father. And what I’d want my child to say about me.

  In Glacier National Park, my father says, “I’ve never seen water so blue.” I have, in several places of the world, I can keep traveling, I realize--- and maybe a regular job won’t be as dull as I feared.

  Weeks after our trip, I call my father.

  “The photos from the trip are wonderful,” he says.” We have got to take another trip like that sometime.

  I tell him I’ve learn decided to settle down, and I’m wearing a watch.

  We can learn from Paragraphs 2 and 3 that the father _________.

  A. followed the fashion

  B. got bored with his job

  C. was unhappy with……

  D. liked the author’s collection of stamps

  What does the author realize at Mount Rushmore?

  A. His father is interested in sculpture

  B. His father is as innocent as a little boy

  C. He should learn sculpture in the future

  D. He should pursue a specific aim in life.

  From the underlined paragraph, we can see that the author________.

  A. wants his children to learn from their grandfather

  B. comes to understand what parental love means

  C. learns how to communicate with his father

  D. hopes to give whatever he can to his father

  What could be inferred about the author and his father from the end of the story?

  A. The call solves their disagreements

  B. The Swiss watch has drawn them closer

  C. They decide to learn photography together.

  D. They begin to change their attitudes to life

  What could be the best title for the passage?

  A. Love Nature, Love Life

  B. A Son Lost in Adventure

  C. A Journey with Dad

  D.The Art of Travel

  【2017·湖南卷】B

  In the mid-1950s, I was a somewhat bored early-adolescent male student who believed that doing any more than necessary was wasted effort. One day, this approach threw me into embarrassment

  In Mrs. Totten’s eighth-grade math class at Central Avenue School in Anderson, Indiana, we were learning to add and subtract decimals (小数)

  Our teacher typically assigned daily homework, which would be recited in class the following day. On most days, our grades were based on our oral answer to homework questions.

  Mrs. Totten usually walked up and down the rows of desks requesting answers from student after student in the order the questions had appeared on our homework sheets. She would start either at the front or the back of the classroom and work toward the other end.

  Since I was seated near the middle of about 35 students, it was easy to figure out which questions I might have to answer. This particular time, I had completed my usual two or three problems according to my calculations.

  What I failed to expect was that several students were absent, which threw off my estimate. As Mrs. Totten made her way from the beginning of the class,I desperately tried to determine which math problem I would get. I tried to work it out before she got to me, but I had brain freeze and couldn’t function.

  When Mrs. Totten reached my desk,she asked what answer I’d got for problem No. 14. “I…I didn’t get anything,”

  I answered,and my face felt warm.

  “Correct,” she said.

  It turned out that the correct answer was zero.

  What did I learn that day? First, always do all your homework. Second, in real life it is’t always what you say but how you say it that matters. Third,I would never make it as a mathematician.

  If I could choose one school day that taught me the most, it would be that one.

  What does the underlined part in Paragraph 1 indicate?

  A.It is wise to value one’s time.

  B.It is important to make an effort

  C.It is right to stick to one’s belief.

  D.It is enough to do the necessary.

  Usually, Mrs. Totten asked her students to _______.

  A. recite their homework together

  B.grade their homework themselves

  C.answer their homework questions orally

  D.check the answers to their homework questions

  The author could work out which questions to answer since the teacher always _______.

  A.asked questions in a regular way

  B.walked up and down when asking questions

  C.chose two or three questions for the students

  D.requested her students to finish their usual questions

  The author failed to get the questions he had expected because _______.

  A.the class didn’t begin as usual

  B.several students didn’t come to school

  C.he didn’t try hard to make his estimate

  D.Mrs. Totten didn’t start from the back of the class

  Which of the following can be the best title for the passage?

  A.An Unforgettable Teacher

  B.A Future Mathematician

  C.An Effective Approach

  D.A Valuable Lesson

  【2017·浙江卷】D

       A city child’s summer is spent in the street in front of his home, and all through the long summer vacations I sat on the edge of the street and watched enviously the other boys on the block play baseball. I was never asked to take part even when one team had a member missing—not out of special cruelty, but because they took it for granted I would be no good at it. They were right, of course.

       I would never forget the wonderful evening when something changed. The baseball ended about eight or eight thirty when it grew dark. Then it was the custom of the boys to retire to a little stoop(门廊) that stuck out from the candy store on the corner and that somehow had become theirs. No grownup ever sat there or attempted to. There the boys would sit, mostly talking about the games played during the day and of the game to be played tomorrow. Then long silences would fall and the boys would wander off one by one. It was just after one of those long silences that my life as an outsider changed. I can no longer remember which boy it was that summer evening who broke the silence with a question: but whoever he was, I nod to him gratefully now. “What’s in those books you’re always reading?” he asked casually. “Stories,” I answered. “What kind?” asked somebody else without much interest.

       Nor do I know what drove me to behave as I did,for usually I just sat there in silence, glad enough to be allowed to reain among them; but instead of answering his question, I told them for two hours the story I was reading at the moment. The book was Sister Carrie. They listened bug-eyed and breathless. I must have told it well, but I think there was another and deeper reason that made them to keep an audience. Listening to a tale being told in the dark is one of the most ancient of man’s entertainments, but I was offering them as well, without being aware of doing it, a new and exciting experience.

       The books they themselves read were the Rover Boys or Tom Swift or G.A.Henty. I had read them too, but at thirteen I had long since left them behind. Since I was much alone I had become an enthusiastic reader and I had gone through the books-for-boys series. In those days there was no reading material between children’s and grownups’books or I could find none. I had gone right fromTome Swift and His Flying Machine to Theodore Dreiser and Sister Carrie. Dreiser had hit my young mind, and they listened to me tell the story with some of the wonder that I had had in reading it.

       The next night and many nights thereafter, a kind of unspoken ritual (仪式) took place. As it grew dark, I would take my place in the center of the stoop and begin the evening’s tale. Some nights, in order to taste my victory more completely, I cheated. I would stop at the most exciting part of a story by Jack London or Bret Harte, and without warning tell them that that was as far as I had gone in the book and it would have to be continued the following evening. It was not true, of course; but I had to make certain of my new-found power and position. I enjoyed the long summer evenings until school began in the fall. Other words of mine have been listened to by larger and more fashionable audiences, but for that tough and athletic one that sat close on the stoop outside the candy store, I have an unreasoning love that will last forever.

  Watching the boys playing baseball, the writer must have felt ________.

     A. bitter and lonely                            B. special and different

     C. pleased and excited                         D. disturbed and annoyed 

  The writer feels grateful even now to the boy who asked the question because the boy ________.

     A. invited him to join in their game       

     B. liked the book that he was reading

     C. broke the long silence of that summer evening

     D. offered him an opportunity that changed his life

  According to Paragraph 3, story-telling was popular among the boys basically because ________.

     A. the story was from a children’s book          B. listening to tales was an age-old practice

     C. the boys had few entertainments after dark     D. the boys didn’t read books by themselves

   The boys were attracted to Sister Carrie because ________.

     A. it was written by Theodore Dreiser            B. it was specifically targeted at boys

     C. it gave them a deeper feeling of pleasure        D. it talked about the wonders of the world

  Sometimes the writer stopped at the most exciting part of a story to _______.

     A. play a mean trick on the boys                B. experience more joy of achievement

     C. add his own imagination to the story          D. help the boys understand the story better

  What is the message conveyed in the story?

     A. One can find his position in life in his own way.

     B. Friendship is built upon respect for each other.

     C. Reading is more important than playing games.

     D. Adult habits are developed from childhood.

  【2017·浙江卷】A

  Wealth starts with a goal saving a dollar at a time. Call it the piggy bank strategy(策略). There are lessons in that time-honored coin-saving container.

     Any huge task seems easier when reduced to baby steps. I f you wished to climb a 12,000-foot mountain, and could do it a day at a time, you would only have to climb 33 feet daily to reach the top in a year. If you want to take a really nice trip in 10 years for a special occasion, to collect the $15,000 cost, you have to save $3.93 a day. If you drop that into a piggy bank and then once a year put $1,434 in a savings account at 1% interest rate after-tax, you will have your trip money.

     When I was a child, my parents gave me a piggy bank to teach me that, if I wanted something, I should save money to buy it. We associate piggy banks with children, but in many countries, the little containers are also popular with adults. Europeans see a piggy bank as a sign of good fortune and wealth. Around the world, many believe a gift of a piggy bank on New Year’s Day brings good luck and financial success. Ah, but you have to put something in it.

     Why is a pig used as a symbol of saving? Why not an elephant bank, which is bigger and holds more coins? In the Middle Ages, before modern banking and credit instruments, people saved money at home, a few coins at a time dropped into a jar or dish. Potters(制陶工) made these inexpensive containers from an orange-colored clay(黏土) called “pygg,” and folks saved coins in pygg jars.The Middle English word

  for pig was “pigge”. While the Saxons pronounced pygg, referring to the clay, as “pug”, eventually the two words changed into the same pronunciation, sounding the “i” as in pig or piggy. As the word became less associated with the orange clay and more with the animal, a clever potter fashioned a pygg jar in the shape of a pig, delighting children and adults. The piggy bank was born.

     Originally you had to break the bank to get to the money, bringing in a sense of seriousness into savings. While piggy banks teach children the wisdom of saving, adults often need to relearn childhood lessons. Think about the things in life that require large amounts of money--- college education, weddings, cars, medical care, starting a business, buying a home, and fun stuff like great trips. So when you have money, take off the top 10%, put it aside, save and invest wisely. 

  What is the piggy bank strategy?

     A. Paying 1% income tax at a time. 

     B. Setting a goal before making a travel plan.

     C. Aiming high even when doing small things.

     D. Putting aside a little money regularly for future use.

   Why did the writer’s parents give him a piggy bank as a gift?

  A. To delight him with the latest fashion.        B. To encourage him to climb mountains.

  C. To help him form the habit of saving.          D. To teach him English pronunciation.

  What does then underlined word “something”(Paragraph 3) most probably refer to?

  A. Money                                 B. Gifts

  C. Financial success                         D. Good luck

  The piggy ban originally was _________.

     A. a potter’s instrument                      B. a cheap clay container

     C. an animal-shaped dish                     D. a pig-like toy for children

  The last paragraph talks about ________.

     A. the seriousness of educating children                B. the enjoyment of taking a great trip

     C. the importance of managing money                 D. the difficulty of starting a business

  【2017·广东卷】C

  Like many new graduates, I left university full of hope for the future but with no real idea of what I wanted to do. My degree, with honors, in English literature had not really prepared me for anything practical. I knew I wanted to make a difference in the world somehow, but I had no idea how to do that. That’s when I learned about the Lighthouse Project.

  I started my journey as a Lighthouse Project volunteer by reading as much as I could about the experiences of previous volunteers. I knew it would be a lot of hard work, and that I would be away from my family and friends for a very long time. In short, I did not take my decision to apply for the Lighthouse Project lightly. Neither did my family.

  Eventually, however, I won the support of my family, and I sent in all the paperwork needed for the application. After countless interviews and presentations, I managed to stand out among the candidates and survive the test alone. Several months later, I finally received a call asking me to report for the duty. I would be going to a small village near Abuja, Nigeria. Where? What? Nigeria? I had no idea. But I was about to find out.

  After completing my training, I was sent to the village that was small and desperately in need of proper accommodation. Though the local villagers were poor, they offered their homes, hearts, and food as if I were their own family. I was asked to lead a small team of local people in building a new schoolhouse. For the next year or so, I taught in that same schoolhouse. But I sometimes think I learned more from my students than they did from me.

  Sometime during that period, I realized that all those things that had seemed so strange or unusual to me no longer did, though I did not get anywhere with the local language, and returned to the United States a different man. The Lighthouse Project had changed my life forever.

  What do we know about the author?

  A. His university education focused on the theoretical knowledge.

  B. His dream at university was to become a volunteer.

  C. He took pride in having contributed to the world.

  D. He felt honored to study English literature.

  According to the Paragraph 2, it is most likely that the author

  A. discussed his decision with his family.

  B. asked previous volunteers about voluntary work

  C. attended special training to perform difficult tasks

  D. felt sad about having to leave his family and friends

  In his application for the volunteer job, the author

  A. participated in many discussions

  B. went through challenging survival tests

  C. wrote quite a few paper on voluntary work

  D. faced strong competition from other candidates

  On arrival at the village, the author was

  A. asked to lead a farming team

  B. sent to teach in a schoolhouse

  C. received warmly by local villagers

  D. arranged to live in a separate house.

  What can we infer from the author’s experiences in Nigeria?

  A. He found some difficulty adapting to the local culture

  B. He had learned to communicate in the local language.

  C. He had overcome all his weaknesses before he left for home.

  D. He was chosen as the most respectable teacher by his students.

  【2017·广东卷】B

  It was a cold winter day. A woman drove up to the Rainbow Bridge tollbooth (收费站). “I’m paying for myself, and for the six cars behind me,” she said with a smile, handing over seven tickets. One after another, the next six drivers arriving at the tollbooth were informed, “Some lady up ahead already paid your fare.”

  It turned out that the woman, Natalie Smith, had read something on a friend’s refrigerator: “Practice random kindness and senseless acts of beauty.” The phrase impressed her so much that she copied it down.

  Judy Foreman spotted the same phrase on a warehouse wall far away from home. When it stayed on her mind for days, she gave up and drove all the way back to copy it down. “I thought it was beautiful,” she said, explaining why she’d taken to writing it at the bottom of all her letters, “like a message from above.” Her husband, Frank, liked the phrase so much that he put it up on the classroom wall for his students, one of whom was the daughter of Alice Johnson, a local news reporter. Alice put it in the newspaper, admitting that though she liked it, she didn’t know where it came from or what it really meant.

  Two days later, Alice got a call from Anne Herbert, a woman living in Marin. It was in a restaurant that Anne wrote the phrase down on a piece of paper, after turning it around in her mind for days.

  “Here’s the idea,” Anne says. “Anything you think there should be more of, do it randomly.” Her fantasies include painting the classrooms of shabby schools, leaving hot meals on kitchen tables in the poor part of town, and giving money secretly to a proud old lady. Anne says, “Kindness can build on itself as much as violence can.”

  The acts of random kindness spread. If you were one of those drivers who found your fare paid, who knows what you might have been inspired to do for someone else later. Like all great events, kindness begins slowly, with every single act. Let it be yours!

  Why did Natalie Smith pay for the six cars behind her?

  A. She knew the car drivers well.

  B. She wanted to show kindness.

  C. She hoped to please others.

  D. She had seven tickets.

  Judy Foreman copied down the phrase because she

  .

  A. thought it was beautifully written

  B. wanted to know what it really meant

  C. decided to write it on a warehouse wall

  D. wanted her husband to put it up in the classroom

  Who came up with the phrase according to the passage?

  A. Judy Foreman.

  B. Natalie Smith.

  C. Alice Johnson.

  D. Anne Herbert.

  Which of the following statements is closest in the meaning to the underlined sentence above?

  A. Kindness and violence can change the world.

  B. Kindness and violence can affect one’s behavior.

  C. Kindness and violence can reproduce themselves.

  D. Kindness and violence can shape one’s character.

  What can we infer from the last paragraph?

  A. People should practice random kindness to those in need.

  B. People who receive kindness are likely to offer it to others.

  C. People should practice random kindness to strangers they meet.

  D. People who receive kindness are likely to pay it back to the giver.

  【2017·福建卷】A

  It was Mother’s Day morning last year and I was doing my shopping at our local supermarket with my five-year-old son, Tenyson. As we were leaving, we found that only minutes earlier an elderly woman had fallen over at the entrance and had hit her head on the concrete. Her husband was with her, but there was blood everywhere and the woman was embarrassed and clearly in shock.

  Walking towards the scene, Tenyson became very upset about what had happened to the couple. He said to me, “Mum, it’s not much fun falling over in front of everyone.”

  At the front of the supermarket a charity(慈善) group had set up a stand selling cooked sausages and flowers to raise funds. Tenyson suggested that we should buy the lady a flower. “It will make her feel better,” he said. I was amazed that he’d come up with such a sweet idea. So we went over to the flower seller and asked her if we could buy a flower for the lady to cheer her up. “Just take it,” she replied. “I can’t take your money for such a wonderful gesture.”

  By now paramedics(救援人员)had arrived, and were attending the injured woman. As we walked up to her, my son became intimidated by all the blood and medical equipment. He said he was just too scared to go up to her.

  Instead I gave the flower to the woman’s husband and told him, “ My son was very upset for your wife and wanted to give her this flower to make her feel better.”

  At that, the old man started crying and said, “Thank you so much, you have a wonderful son. Happy Mother’s Day to you.”

  The man then bent down and gave his wife the flower, telling her who it was from. Though badly hurt and shaken, the old lady looked up at Tenyson with love in her eyes and gave him a little smile.

  56. What dose the author intend to tell us?

  A. One can never be too careful.

  B. Actions speak louder than words.

  C. Love begins with a little smile.

  D. A small act of kindness brings a great joy.

  57. Which of the following is TRUE according to the passage?

  A. The elderly woman was knocked down by Tenyson.

  B. Tenyson’s idea of buying a flower gained his father’s support.

  C. Tenyson’s care for the elderly woman puzzled the flower seller.

  D. The elderly woman was moved to tears by Tenyson’s gesture.

  58. The underlined word “intimidated” in the fourth paragraph probably means “___________”.

  A. astonished B. struck C. frightened D. excited

  59. What would be the best title for the passage?

  A. Flower Power

  B. Mother’s Day

  C. An Accidental Injury

  D. An Embarrassing Moment

  【2017·湖北卷】A

  Before I had my son, I spent two years working with children with disabilities. I learned that shouting and threats of punishment would result in a disaster. Coming up against their behaviour could only make the job harder and their behaviour more extreme. I found something that worked, though.

  There was a very naughty boy in the nursery and a teacher who was generally very confident with the children was asked to take charge of him. One day the boy joined a session in the room next to mine. His appearance created an atmosphere of tension. He spent the entire session running around, hitting and kicking, and destroying property.

  I was in the craft room working with some other children when my co-worker told me that this boy’s teacher was in tears, and could not get control of the situation. As we were talking,the boy ran in. I told my co-worker that I would take care of him.

  I closed the door. He was full of energy, throwing things around and making a huge mess. But I could see that he was doing all these to annoy me. He needed connection, and this was the only way he knew how to ask for it. So I sat back down and kept quiet. Then he slowed down and began making a rocket. I talked to him about it. We continued like this for a few minutes before I slipped into the conversation:

  “So what happened today?”

  It was purely a question, no blame or anger in my tone. I believe that if I had criticized him, the gate that was slowly opening would have shut firmly closed. He told me that the teacher didn’t let him do what he knew well due to safety but asked him to do what he disliked. He also admitted that he had enjoyed making her run around and saw it as a game. I explained that his teacher had not seen it as a game and was very upset. This again was stated simply as a fact. I suggested that next time he had a session, he talk about what he hoped to do at the start,which might be easier for everyone. He agreed and was quiet for a moment. Then he looked at me with tears in his eyes before quietly asking if he could go to find his teacher to apologize.

  .The boy made trouble for his teacher because he_____ .

  A. was accused of destroying property

  B. was told not to yell at other children

  C. was made to do things against his will

  D. was blamed for creating an air of tension

  .Why didn’t the author do anything about the boy’s bad behavior at first?

  A. She didn’t want to make it worse.

  B. She didn’t mind the huge mess at all.

  C. She was tired of shouting and threats.

  D. She hadn’t thought of a coping strategy.

  .The author managed to get the boy to talk to her by _____.

  A. playing games with him

  B. giving him a good suggestion

  C. describing his teacher’s feelings

  D. avoiding making critical remarks

  Why did the boy have tears in his eyes in the end?

  A. He was sorry about his reputation.

  B. He was regretful about his behavior.

  C. He was fearful of the author’s warning.

  D. He was sad for the author’s misunderstanding.

  【2017·江苏卷】D

  August 1990, Boston

  Dear Maya Shao­ming,

  June 6,1990 is a special day. My long­awaited dream came true the minute your father cried, “A girl!” You are more than just a second child, more than just a girl to match our boy. You, little daughter, are the link to our female line, the legacy of another woman's pain and sacrifice 31 years ago.

  Let me tell you about your Chinese grandmother. Somewhere in Hong Kong, in the late fifties, a young waitress found herself pregnant(怀孕) by a cook, probably a suffered to give it birth, and kept the little girl for the first three months of her life. I like to think that my mother—your grandmother—loved me and fought to raise me on her own, but that the daily struggle was too hard. Worn down by the demands of the new baby and perhaps the constant threat of starvation, she made the painful decision to give away her girl so that both of us might have a chance for a better life.

  More likely, I was dropped at the orphanage(孤儿院) steps or somewhere else. I will probably never know the truth. Having a baby in her unmarried state would have brought shame on the family in China, so she probably kept my existence a secret. Once I was out of her life, it was as if I had never been born. And so you and your brother and I are the missing leaves on a family tree.

  Do they ever wonder if we exist?

  Before I was two, I was adopted by an Anglo couple. Fed three square meals a day, I grew like a wild weed and grasped all the opportunities they had to offer—books, music, education, church life and community activities. In a family of blue­eyed blonds, though, I_stood_out_like_a_sore_thumb. Whether from jealousy or fear of someone who looked so different, my older brothers sometimes teased me about my unpleasing skin, or made fun of my clumsy walk. Moody and impatient, burdened by fears that none of us realized resulted from my early years of need, I was not an easy child to love. My mother and I conflicted countless times over the years, but gradually came to see one another as real human beings with faults and talents, and as women of strength in our own right. Lacking a mirror image in the mother who raised me, I had to seek my identity as a woman on my own. The Asian American community has helped me regain my double identity.

  But part of me will always be missing: my beginnings, my personal history, all the delicate details that give a person her origin. Nevertheless, someone gave me a lucky name “Siu Wai”. “Siu” means “little”, and “Wai” means “clever”. Therefore, my baby name was “Clever little one”. Who chose those words? Who cared enough to note my arrival in the world?

  I lost my Chinese name for 18 years. It was Americanized for convenience to “Sue”. But like an ill­fitting coat, it made me uncomfortable. I hated the name. But even more, I hated being Chinese. It took many years to become proud of my Asian origin and work up the courage to take back my birth­name. That, plus a little knowledge of classroom Cantonese, is all the Chinese culture I have to offer you. Not white, certainly, but not really Asian, I try to pave the way between the two worlds and bridge the gap for you. Your name, “Shao­ming”,

  is very much like mine—“Shao” means “little”. And “ming” is “bright”, as in a shining sun or moon. Whose lives will you brighten, little Maya? Your_past_is_more_complete_than_mine,_and each day I cradle you in your babyhood, generously giving you the loving care I lacked for my first two years. When I pat you, I comfort the lost baby inside me who still cries for her mother.

  Sweet Maya, it doesn't matter what you “become” later on. You have already fulfilled my wildest dreams.

  I love you,

  Why is June 6, 1990 a special day for Mommy?

  A. Her dream of being a mother came true.

  B. She found her origin from her Chinese mother.

  C. She wrote the letter to her daughter.

  D. Her female line was well linked.

  How does Mommy feel about her being given away?

  A. It is bitter and disappointing.

  B. It is painful but understandable.

  C. She feels sorry but sympathetic.

  D. She feels hurt and angry.

  What does “I stood out like a sore thumb” in Paragraph 5 mean?

  A. I walked clumsily out of pains.

  B. I was not easy to love due to jealousy.

  C. I was impatient out of fear.

  D. I looked different from others.

  What can be inferred from Mommy’s Anglo family life?

  A. She used to experience an identity crisis.

  B. She fought against her American identity.

  C. She forgot the pains of her early years.

  D. She kept her love for Asia from childhood.

  Why did Mommy name her daughter “Shao-ming?”

  A. To match her own birth-name.

  B. To brighten the lives of the family.

  C. To identify her with Chinese origin.

  D. To justify her pride in Chinese culture.

  By “Your past is more complete than mine,” Mommy means ________.

  A. her past was completed earlier than Shao-ming’s

  B. Shao-ming has got motherly care and a sense of roots

  C. her mother didn’t comfort her the way she did Shao-ming

  D. her past was spent brokenly, first in Asia, then in the US

  【2017·江西卷】A

  Larry was on another of his underwater expeditions(探险)but this time, it was different. He decided to take his daughter along with him. She was only ten years old. This would be her first trip with her father on what he had always been famous for.

  Larry first began diving when he was his daughter’s age. Similarly, his father had taken him along on one of his expeditions. Since then, he had never looked back. Larry started out by renting diving suits from the small diving shop just along the shore. He had hated them. They were either too big or too small. Then, there was the instructor. He gave him a short lesson before allowing him into the water with his father. He had made an exception. Larry would never have been able to go down without at least five hours of theory and another similar number of hours on practical lessons with a guide. Children his age were not even allowed to dive.

  After the first expedition, Larry’s later diving adventures only got better and better. There was never a dull moment. In his black and blue suit and with an oxygen tank fastened on his back, Larry dived from boats into the middle of the ocean. Dangerous areas did not prevent him from continuing his search. Sometimes, he was limited to a cage underwater but that did not bother him. At least, he was still able to take photographs of the underwater creatures.

  Larry’s first expedition without his father was in the Cayman Islands. There were numerous diving spots in the area and Larry was determined to visit all of them. Fortunately for him, a man offered to take him around the different Spots for free. Larry didn’t even know what the time was, how many spots he dived into or how many photographs he had taken. The diving spots afforded such a wide array of fish and sea creatures that Larry saw more than thirty varieties of creatures.

  Larry looked at his daughter. She looked as excited as he had been when he was her age. He hoped she would be able to continue the family tradition. Already, she looked like she was much braver than had been then. This was the key to a successful underwater expedition.

  56.In what way was this expedition different for Larry?

  A.His daughter had grown up.

  B.He had become a famous diver.

  C.His father would dive with him.

  D.His daughter would dive with him.

  57. What can be inferred from Paragraph2?

  A. Larry had some privileges.

  B. Larry liked the rented diving suits.

  C.Divers had to buy diving equipment.

  D.Ten-year-old children were permitted to dive.

  58. Why did Larry have to stay in a cage underwater sometimes?

  A. To protect himself from danger.

  B.To dive into the deep water.

  C.To admire the underwater view.

  D.To take photo more conveniently.

  59. What can be learned from the underlined sentence?

  A. Larry didn’t wear a watch.

  B. Larry was not good at math.

  C. Larry had a poor memory.

  D. Larry enjoyed the adventure.

  60. What did Larry expect his daughter to do?

  A. Become a successful diver.

  B. Make a good diving guide.

  C. Take a lot of photo underwater.

  D. Have longer hours of training.

  【2017·重庆卷】D

  One moment it was quiet and calm in the forest, the next, the air was charged with tension. The elephant had heard the distant alarm calls of animals and her mood suddenly changed. I urged the elephant deeper into the forest. We sounded like a forest fire --- cracking, snapping, trailblazing. But through all the noise came a sharp warning cry. The elephant stopped and we heard it again --- the tell-tale call of a spotted deer.

  I looked quickly around the shadows of the forest. Rays of sunlight shone through tree branches, beneath which the patchwork(交错)of green plants and shadows-within-shadows would make tiger stripes(条纹)look more attractive. Apart from an occasional noise from the elephant’s stomach, the forest was silent.

  Gradually, the tension slipped from our bodies. The elephant seized a nearby branch and put it into its mouth. I reached forward and gently moved my hand over the elephant’s neck; there was a soft part, free of wrinkles and hairs, behind her ear.

  This was my fourth time to sense the aura of the forest in Corbett, although I saw no tigers in the end. Located at the foot of the Himalayan mountains, Corbett is home to about 135 Bengal tigers, but the forest seemed to be guarding their whereabouts(出没处), a silent reminder of their secrecy and rarity. Still, I was happy enough touching the elephant behind the ear. If I had so desperately wanted to see a tiger, I could have gone to a zoo. After all, spotting tigers merely confirms their beauty; tracking them can make you aware of something more.

  . Which of the following was a clear signal of alarm?

  A. The elephant stopped.

  B. A spotted deer called.

  C. The elephant seized a branch.

  D. The forest was silent for a while.

  . The author begins his account of the tour in the forest mainly by _______.

  A. describing various sounds

  B. comparing different animals

  C. listing different activities

  D. introducing various plants

  . What does the underlined part “to sense the aura” most probably mean?

  A. To see the diversity.

  B. To enjoy the scenery.

  C. To feel the atmosphere.

  D. To experience the freedom.

  . How does the author feel after several visits to Corbett?

  A. Seeing a Bengal tiger is quite thrilling.

  B. It is very time-consuming to travel in Corbett.

  C. It is really worthwhile to study the animals in Corbett.

  D. The process of finding Bengal tigers is most appealing.

  【2017·北京卷】B

  The Brown Bear

  My wife Laura and I were on the beach, with three of our children, taking pictures of shore birds near our home in Alaska when we spotted a bear. The bear was thin and small, moving aimlessly.

  Just a few minutes later, I heard my daughter shouting, “Dad! The bear is right behind us!” An agreesive bear will usually rush forward to frighten away its enemy but would suddenly stop at the last minute. This one was silent and its ears pinned back---- the sign (迹象) of an animal that is going in for the kill. And it was a cold April day. The bear behaved abnormally, probably because of hunger.

  I held my camera tripod (三脚架) in both hands to form a barrier as the bear rushed into me. Its huge head was level with my chest and shoulders, and the tripod stuck across its mouth. It bit down and I found myself supporting its weight. I knew I would not be able to hold it for long.

  Even so, this was a fight I had to win: I was all that stood between the bear and my family, who would stand little chance of running faster than a brown bear.

  The bear hit at the camera, cutting it off the tripod. I raised my left arm to protect my face; the beast held tightly on the tripod and pressed it into my side. My arm could not move, and I sensed that my bones were going to break.

  Drawing back my free hand, I struck the bear as hard as I could for five to six times. The bear opened its mouth and I grasped its fur, trying to push it away. I was actually wrestling (扭打) with the bear at this point. Then, as suddenly as it had begun, the fight ended. The bear moved back toward the forest, before returning for another attack----- The first time I felf panic.

  Apparently satisfied that we caused no further threat, the bear moved off, destroying a fence as it went. My arm was injured, but the outcome for us could hardly have been better. I’m proud that my family reminded clear-headed when panic could have led to a very different outcome.

  60. The brown bear approached the family in order to _______.

  A. catch shore birds

  B. start an attack

  C. protect the children

  D. set up a barrier for itself

  61. The bear finally went away after it _______.

  A. felt safe

  B. got injured

  C. found some food

  D. took away the camera

  62. The writer and his family survived mainly due to their ______ .

  A. pride

  B. patience

  C. calmness

  D. cautiousness

  【2017·重庆卷】A

  I was never very neat, while my roommate Kate was extremely organized. Each of her objects had its place, but mine always hid somewhere. She even labeled(贴标签)everything. I always looked for everything. Over time, Kate got neater and I got messier. She would push my dirty clothing over, and I would lay my books on her tidy desk. We both got tired of each other.

  War broke out one evening. Kate came into the room. Soon, I heard her screaming. “Take your shoes away! Why under my bed!” Deafened, I saw my shoes flying at me. I jumped to my feet and started yelling. She yelled back louder.

  The room was filled with anger. We could not have stayed together for a single minute but for a phone call. Kate answered it. From her end of the conversation, I could tell right away her grandma was seriously ill. When she hung up, she quickly crawled(爬)under her covers, sobbing.

  Obviously, that was not something she should not go through alone. All of a sudden, a warm feeling of sympathy rose up in my heart.

  Slowly, I collected the pencils, took back the books, made my bed, cleaned the socks and swept the floor, even on her side. I got so into my work that I even didn’t notice Kate had sat up. She was watching, her tears dried and her expression one of disbelief. Then, she reached out her hands to grasp mine. I looked up into her eyes. She smiled at me, “Thanks.”

  Kate and I stayed roommates for the rest of the year. We didn’t always agree, but we learned the key to living together: giving in, cleaning up and holding on.

  . What made Kate so angry one evening?

  A. She couldn’t find her books.

  B. She heard the author shouting loud.

  C. She got the news that her grandma was ill.

  D. She saw the author’s shoes beneath her bed.

  . The author tidied up the room most probably because _______.

  A. she was scared by Kate’s anger

  B. she hated herself for being so messy

  C. she wanted to show her care

  D. she was asked by Kate to do so

  . How is Paragraph 1 mainly developed?

  A. By analyzing causes

  B. By showing differences

  C. By describing a process

  D. By following time order

  . What might be the best title for the story?

  A. My Friend Kate

  B. Hard Work Pays Off

  C. How to Be Organized

  D. Learning to Be Roommates

  【2017·山东卷】A

  One morning, Ann’s neighbor Tracy found a lost dog wandering around the local elementary school. She asked Ann if she could keep an eye on the dog. Ann said that she could watch it only for the day.

  Tracy took photos of the dog and printed off 400 FOUND fliers(传单), and put them in mailboxes. Meanwhile, Ann went to the dollar store and bought some pet supplies, warning her two sons not to fall in love with the dog. At the time, Ann’s son Thomas was 10 years old, and Jack, who was recovering from a heart operation, was 21 years old.

  Four days later Ann was still looking after the dog, whom they had started to call Riley. When she arrived home from work, the dog threw itself against the screen door and barked madly at her. As soon as she opened the door, Riley dashed into the boys’ room where Ann found Jack suffering from a heart attack. Riley ran over to Jack, but as soon as Ann bent over to help him the dog went silent.

  “If it hadn’t come to get me, the doctor said Jack would have died,” Ann reported to a local newspaper. At this point, no one had called to claim the dog, so Ann decided to keep it.

  The next morning Tracy got a call. A man named Peter recognized his lost dog and called the number on the flier. Tracy started crying, and told him, “That dog saved my friend’s son.”

  Peter drove to Ann’s house to pick up his dog, and saw Thomas and Jack crying in the window. After a few moments Peter said, “Maybe Odie was supposed to find you, maybe you should keep it.”

  41. What did Tracy do after finding the dog?

  A. She looked for its owner

  B. She gave it to Ann as a gift.

  C. She sold it to the dollar store.

  D. She bought some food for it.

  42. How did the dog help save Jack?

  A. By breaking the door for Ann.

  B. By leading Ann to Jack’s room.

  C. By dragging Jack out of the room.

  D. By attending Jack when Ann was out.

  43. What was Ann’s attitude to the dog according to Paragraph 4?

  A. Sympathetic

  B. Doubtful

  C. Tolerant

  D. Grateful

  44. For what purpose did Peter call Tracy?

  A. To help her friend’s son.

  B. To interview Tracy

  C. To take back his dog.

  D. To return the flier to her.

  45. What can we infer about the dog from the last paragraph?

  A. It would be given to Odie.

  B. It would be kept by Ann’ family.

  C. It would be returned to Peter.

  D. It would be taken away by Tracy.

  【2017·山东卷】B

  It was one of those terribly hot days in Baltimore. Needless to say, it was too hot to do anything outside. But it was also scorching in our apartment. This was 1962, and I would not live in a place with an air conditioner for another ten years. So my brother and I decided to leave the apartment to find someplace indoors. He suggested we could see a movie. It was a brilliant plan.

  Movie theaters were one of the few places you could sit all day and—most important —sit in air conditioning. In those days, you could buy one ticket and sit through two movies. Then, the theater would show the same two movies again. If you wanted to, you could sit through them twice. Most people did not do that, but the manager at our theater. Mr. Bellow did not mind if you did.

  That particular day, my brother and I sat through both movies twice, trying to escape the heat. We bought three bags of popcorn and three sodas each. Then, we sat and watched The Music Man followed by The Man Who Shot Liberty Valance. We’d already seen the second movie once before. It had been at the theater since January, because Mr. Bellow loved anything with John Wayne in it.

  We left the theater around 8, just before the evening shows began. But we returned the next day and saw the same two movies again, twice more. And we did it the next day too. Finally, on the fourth day, the heat wave broke.

  Still, to this day I can sing half the songs in The Music Man and recite half of John Wayne and Jimmy Stewart’s dialogue from The Man Who Shot Liberty Valance! Those memories are some of the few I have of the heat wave of 1962. They’re really memories of the screen, not memories of my life.

  46.In which year did the author first live in a place with an air conditioner?

  A. 1952

  B. 1962

  C. 1972

  D. 1982

  47.What does the underlined word”It” in Paragraph 3 refer to?

  A. The heat

  B. The theater.

  C. The Music Man

  D. The Man Who Shot Liberty Valance

  48.What do we know about Mr. Bellow?

  A. He loved children very much.

  B. He was a fan of John Wayne.

  C. He sold air conditioners.

  D. He was a movie star.

  49.Why did the author and his/her brother see the same movies several times?

  A. The two movies were really wonderful.

  B. They wanted to avoid the heat outside.

  C. The manager of the theater was friendly.

  D. They liked the popcorn and the soda at the theater.

  50.What can we learn from the last paragraph?

  A. The author turned out to be a great singer.

  B. The author enjoyed the heat wave of 1962.

  C. The author’s life has been changed by the two movies.

  D. The author considers the experience at the theater unforgettable.

  人物传记

  【2017·山东卷】 C

  Elizabeth Freeman was born about 1742 to African American parents who were slaves. At the age of six months she was acquired, along with her sister, by John Ashley, a wealthy Massachusetts slaveholders. She became known as “Mumbet” or “Mum Bett.”

  For nearly 30 years Mumbet served the Ashley family. One day, Ashley’s wife tried to strike Mumbet’s sister with a spade. Mumbet protected her sister and took the blow instead. Furious, she left the house and refused to come back. When the Ashleys tried to make her return, Mumbet consulted a lawyer, Theodore Sedgewick. With his help, Mumbet sued(起诉) for her freedom.

  While serving the Ashleys, Mumbet had listened to many discussions of the new Massachusetts constitution. If the constitution said that all people were free and equal, then she thought it should apply to her. Eventually, Mumbet won her freedom---- the first slave in Massachusetts to do so under the new constitution.

  Strangely enough, after the trial, the Ashleys asked Mumbet to come back and work for them as a paid employee. She declined and instead went to work for Segdewick. Mumbet died in 1829, but her legacy lived on in her many descendants(后裔). One of her great-grandchildren was W.E.B. Du Bois, one of the founder of the NAACP, and an important writer and spokesperson for African American civil rights.

  Mumbet’s tombstone still stands in the Massachusetts cemetery where she was buried. It reads, in part: “She was born a slave and remained a slave and remained a slave for nearly thirty years. She could neither read nor write, yet in her own sphere she had no superior or equal.”

  51. What do we know about Mumbet according to Paragraph 1?

  A. She was born a slave

  B. She was a slaveholder

  C. She had a famous sister

  D. She was born into a rich family

  52. Why did Mumbet run away from the Ashleys?

  A. She found an employer

  B. She wanted to be a lawyer

  C. She was hit and got angry

  D. She had to take care of her sister

  53. What did Mumbet learn from discussions about the new consititution?

  A. She should always obey her owners’ orders B. She should be as free and equal as whites

  C. How to be a good servant

  D. How to apply for a job

  54. What did Mumbet do after the trial?

  A. She chose to work for a lawyer

  B. She found the NAACP

  C. She continued to serve the Ashleys

  D. She went to live with her grandchildren

  55. What is the test mainly about?

  A. A story of a famous writer and spokesperson B. The friendship between a lawyer and a slave

  C. The life of a brave African American woman D. A trial that shocked the whole world

  【2017·广东卷】A

  Samuel Osmond is a 19-year-old law student from Cornwall, England. He never studied the piano. However, he can play very difficult musical pieces by musicians such as Chopin and Beethoven just a few minutes after he hears them. He learns a piece of music by listening to it in parts. Then he thinks about the notes in his head. Two years ago, he played his first piece Moonlight Sonata(奏鸣曲)by Beethoven. He surprised everyone around him.

  Amazed that he remembered this long and difficult piece of music and played it perfectly, his teachers say Samuel is unbelievable .They say his ability is very rare, but Samuel doesn’t even realize that what he can do is special. Samuel wanted to become a lawyer as it was the wish of his parents, but music teachers told him he should study music instead. Now, he studies law and music.

  Samuel can’t understand why everyone is so surprised. “I grew up with music. My mother played the piano and my father played the guitar. About two years ago, I suddenly decided to start playing the piano, without being able to read music and without having any lessons. It comes easily to me ---I hear the notes and can bear them in mind---each and every note,” says Samuel.

  Recently, Samuel performed a piece during a special event at his college. The piece had more than a thousand notes. The audience was impressed by his amazing performance. He is now learning a piece that is so difficult that many professional pianists can’t play it. Samuel says confidently,” It’s all about super memory---I guess I have that gift.”

  However, Samuel’s ability to remember things doesn’t stop with music. His family says that even when he was a young boy, Samuel heard someone read a story, and then he could retell the story word for word.

  Samuel is still only a teenager. He doesn’t know what he wants to do in the future. For now, he is just happy to play beautiful music and continue his studies.

  What is special about Samuel Osmond?

  A. He has a gift for writing music.

  B. He can write down the note he hears.

  C. He is a top student at the law school.

  D. He can play the musical piece he hears.

  What can we learn from Paragraph 2?

  Samuel chose law against the wish of his parents.

  B. Samuel planned to be a lawyer rather than a musician.

  C. Samuel thinks of himself as a man of great musical ability.

  D. Samuel studies law and music on the advice of his teachers.

  Everyone around Samuel was surprised because he _________.

  received a good early education in music

  played the guitar and the piano perfectly

  could play the piano without reading music

  could play the guitar better than his father

  What can we infer about Samuel in Paragraph 4?

  He became famous during a special event at his college.

  He is proud of his ability to remember things accurately.

  He plays the piano better than many professional pianists.

  He impressed the audience by playing all the musical pieces.

  Which of the following is the best title of the passage?

  The Qualities of a Musician

  The Story of a Musical Talent

  The Importance of Early Education

  The Relationship between Memory and Music.

  【两年模拟】2015、201名校模拟题及其答案解析

  1.【河南省重点中学协作体2016届高三第二次适应性考试】A

  This is a true story from Guyana.One day, a boy took a piece of paper from a box.He made a paper ball and pushed it into his nose.He couldn’t get it out.He ran crying to his mother.His mother couldn’t get the paper out, either.A week later, the paper was still in the boy’s nose.His nose began to have a bad smell.

  So his mother took the boy to a hospital.The doctor looked up at the child’s nose, but she couldn’t get the paper out.She said she had to cut the boy’s nose to get the paper out.

  The boy’s mother came home looking sad.She didn’t want her child to have his nose cut.The next day she took the boy to her friend Sidney who lived in a house with an old lady called May.May wanted to see the child, so the child let her look up his nose.

  “Yes, I can see it,” May said.“It will be out soon.”

  As she spoke, she shook some black pepper (胡椒粉)on the child’s nose.The child gave a mighty sneeze and the paper flew out.His mother was surprised.May told his mother to take the boy to the seaside for a swim, for the salt water would go up his nose and stop the bad smell.So the lucky boy didn’t have to go to the hospital to have his nose cut.

  21.After the boy pushed a paper ball into his nose, ____.

  A.he took it out                      

  B.his mother took it out

  C.he tried to take it out but failed             D.he did nothing but cry

  22.Which of the following is TRUE?

  A.The doctor helped to take the paper ball out of the boy’s nose.

  B.The boy had to have his nose cut at last.

  C.The boy’s mother found some black pepper to solve the problem.

  D.May succeeded in taking the paper out.

  23.The boy should be taken to the seaside for a swim because ____.

  A.he needed to learn to swim.

  B.the sea water would wash out the paper ball.

  C.the sea water would stop the bad smell of his nose.

  D.he needed a rest.

  A

  Malala Yousafzai, the teen activist from Pakistan, has just become the youngest person ever to win a Nobel Prize. In 2017, Malala survived from being shot by terrorists for speaking out for a girl’s right to be educated in her native country. She is being recognized today for her global activism on education for girls. Winning this award is a huge honor, especially for such a young person.

  Malala’s journey to the Nobel Prize has not been easy. She grew up in a dangerous district of Pakistan called the Swat Valley. This area was taken over by the Taliban. The Taliban ruled the district with an iron fist. They were especially cruel to women, not allowing them to go shopping, have jobs, or go to school.

  In 2009, Malala began speaking out publicly for every girl's right to an education. She blogged about how the Taliban didn’t want her to go to school. She also continued to attend her local school, despite the danger involved.

  Then, in 2017, the Taliban tried to kill Malala to silence her. Two Taliban gunmen shot her and two other students while they were on a school bus. Malala was flown to the United Kingdom for immediate medical treatment.

  Malala eventually recovered from her injuries.

  She now lives in Birmingham, England, and continues to work hard to support girls' and women’s rights around the world.

  “The terrorists thought that they would change my aims and stop my ambitions,” she said, “but nothing changed in my life except this. Weakness, fear, and hopelessness died. Strength, power, and courage were born.”

  The 17-year-old girl from Pakistan is sharing the 2017 Nobel Peace Prize with a children’s activist Kailash Satyarthi, who is from India Satyarthi leads many campaigns dedicated to removing child labor.

  21. Which of the following is the correct order of the events relevant to Malala?

  a. She recovered from the serious injuries.

  b. She won the Nobel Peace Prize.

  c. She began speaking out publicly for girls' education.

  d. She was shot by the terrorists on a school bus.

  A. c, d, a, b

  B. a, d, c, b

  C. a, c. d, b

  D. c, a, d, b

  22. What can we mainly infer from the second paragraph?

  A. People in Pakistan were brave.

  B. Women in Pakistan were not allowed to go shopping.

  C. The Taliban’s cruelty was severe.

  D. Malala’s journey contributed to her Nobel Prize.

  23. After recovering from being shot, Malala ________.

  A. felt depressed

  B. was devoted to removing child labor

  C. decided to stop her ambitions

  D. continued to fight for women’s right

  24. Which words can best describe Malala according to the passage?

  A. Sensitive and stubborn.

  B. Devoted and fearless.

  C. Proud and optimistic.

  D. Creative and outgoing.

  3. 【湖南省师大附中、长沙一中、长郡中学、雅礼中学2016届高三四校联考】 B

  Climbing the corporate ladder (晋升) during my 32 years at Southern Bell (now AT&T) never appealed much to me. But climbing telephone poles? Now that's a whole different story.

  In 1978, I became the first female lineman in Columbus, Georgia.

  I really enjoyed the work, and most of the men accepted me because I worked hard at it.

  I think that at first, they didn't think I could do it. But I got right in there and proved myself.

  I was a lineman for about three years. Then l went on to work at other outdoor jobs at the company before l retired in 1997.

  I decided to become a lineman because I wanted to work outdoors. I was a tomboy(假小子) as a child and even built my own tree house, so you could say I had a little experience. My husband, William, encouraged me to give the job a try. After work, I liked to drive around town and show him the wires I'd spliced (加固) that day.

  People were surprised to see a female lineman. They'd say, “Look, that's a woman up that pole." One day I heard a man say, “Oh, look, there's a telephone man-woman!" Safety belts and hooks minimized the risk of falling, but concentration was essential. Sometimes I felt a little nervous,

  but the guys told me I was less likely to fall if I

  wasn't too confident. It helped that I was strong, because the cables and equipment are heavy. I'm not sure every woman could have done what I did. 1 took a lot of pride in it.

  25. When the author first became a lineman,

  .

  A. she was sure to be promoted

  B. she loved it and went all out for it

  C. she tried her best to convince her husband

  D. she received immediate recognition from colleagues

  26. What is needed to be a lineman according to the author?

  A. Concentration and strength.

  B. Confidence and patience.

  C. Bravery and devotion.

  D. Efficiency and talent.

  27. Which of the following best describe the author's husband?

  A. Outgoing and considerate.

  B. Proud and demanding.

  C. Adventurous and humorous.

  D. Open-minded and supportive.

  4.D【】

  When Luca first heard of the Island of Inventions, he was still very young. But its wonders sounded so incredible that they were forever burned in his memory. From that moment, he never stopped searching for clues which might lead him to the island. He read hundreds of adventure books, histories, volumes of physics and chemistry, even music.

  Over time, he pieced together his idea of what the Island of Inventions was like. It was a secret place, where all the great wise men of the world would meet to learn and invent together. Access to the island was totally restricted. To join, you had to have created some great invention for humanity. Only then could you receive the special invitation — which came with directions to the island.

  So Luca spent his youth studying and inventing. Every new idea he got, he made it into an invention, and if there were something he didn't understand, he’d seek others to help him. Soon he met other young inventors and he told them about the Island of Inventions. They too dreamed of one day receiving an invitation letter.

  As years passed, the disappointment of not receiving their invitation made Luca and his friends work harder and co-operate more. They would meet in Luca's house, share their ideas and build new machines. Their inventions became known throughout the world, and improved the lives of millions.

  But still, no invitation came.

  They didn’t lose heart. They continued learning and inventing every day, trying to come up with more and better ideas. Fresh young talent joined their group, as more inventors dreamed of getting to the island.

  One day many years later, Luca, already very old, was speaking with a brilliant young man named Roberts, who had written to him, asking him to join the group. Luca started telling the man of the wonderful Island of Inventions, and of how he was sure that some day they would receive an invitation. Surprised, the young inventor interrupted: “You mean this place isn’t the Island of Inventions? Isn't the letter you sent me the real invitation?”

  It was only then that Luca realised that his dream had become true in his very own house. No island could exist which would be better than where he was now. No place of invention would be better than what he and his friends had created. Luca felt happy to know that he had always been on the island, and that his life of invention and study had been a truly happy one.

  13. Why did Luca and his colleagues probably never receive an invitation to the island?

  A. Their ideas were not considered worthy enough.

  B. They were too concerned with acquiring fame.

  C. The organization had already broken up.

  D. The island was not a real place.

  14. What did Luca learn at the end of the story?

  A. The island of inventions did not exist.

  B. He had wasted most of his life.

  C. He had unknowingly built his own dreamland.

  D. He finally would receive his invitation.

  15. Which of the following best describes Luca?

  A. Aggressive.

  B. Trusting.

  C. Creative.

  D. Easy-going.

  5.B【】

  I was blind, but I was ashamed of it if it was known. I refused to use a white stick and hated asking for help. After all, I was a teenage girl, and I couldn’t bear people to look at me and think I was not like them. I must have been a terrible danger on the roads. Coming across me wandering through the traffic, motorists probably would have to stop rapidly on their brakes. Apart from that, there were all sorts of disasters that used to occur on the way to and from work.

  One evening, I got off the bus about halfway home where I had to change buses, and as usual I ran into something. “I’m awfully sorry,” I said and stepped forward only to run into it again. When it happened a third time, I realized I had been apologizing to a lamppost. This was just one of the stupid things that constantly happened to me. So I carried on and found the bus stop, which was a request stop, where the bus wouldn’t stop unless passengers wanted to get on or off. No one else was there and I had to guess if the bus had arrived.

  Generally in this situation, because I hated showing I was blind by asking for help, I tried to guess at the sound. Sometimes I would stop a big lorry and stand there feeling stupid as it drew away. In the end, I usually managed to swallow my pride and ask someone at the stop for help.

  But at this particular evening no one joined me at the stop; it seemed that everyone had suddenly decided not to travel by bus. Of course I heard plenty of buses pass, or I thought I did. But because I had given up stopping them for fear of making a fool of myself, I let them all go by. I stood there alone for half an hour without stopping one. Then I gave up. I decided to walk on to the next stop.

  24. The girl refused to ask for help because she thought_________.

  A. she might be recognized

  B. asking for help looked silly

  C. she was normal and independent

  D. being found blind was embarrassing

  25. After the girl got off the bus that evening, she_________.

  A. began to run

  B. hit a person as usual

  C. hit a lamppost by accident

  D. was caught by something

  26. What was the problem with guessing at the sound to stop a bus?

  A. Other vehicles also stopped there.

  B. It was unreliable for making judgments.

  C. More lorries than buses responded to the girl.

  D. It took too much time for the girl to catch the bus.

  27. Finally the girl decided to walk to the next stop, hoping__________.

  A. to find people there

  B. to find more buses there

  C. to find the bus by herself there

  D. to find people more helpful there

  6.B 【】On May 28th, 1998 I was driving my 13-year-old daughter and 14-year-old son to school. Unfortunately my car crashed with a truck. All three of us were critically injured. Stacey’s brain injury was the most severe. Despite all attempts to save her, she died two days later. When I became conscious, I heard my husband talking to the organ donor coordinator(器官捐助协调员).

  We knew that Stacey would want that because that’s the kind of person she was. She always wanted to assist people. She was always smiling and brought so much joy to others. She was friendly and enjoyed writing stories and poetry.

  It has been a comfort to know that Stacey lives on in the lives of others. I have even met the family of one of Stacey’s kidney recipients (接受人). This little girl was 6 years old at the time of the transplant. She was suffering from Nephrotic Syndrome. I saw pictures showing the effects of the Nephrotic Syndrome and the pictures of her after the transplant. They already showed an amazing difference. We correspond every year through letters, cards and pictures. The girl is doing really well at school and will be graduating from high school next year. I am so grateful that we have this relationship. I also feel relieved that my daughter has made so great a difference to a person’s life.

  Over the years, I have had some kind of contact with all of the recipients except the heart recipient. I’m very sorry for that. I would love to hear something from the heart recipient’s family. There are so many things that I wonder about her. I wonder if she is anything like Stacey. Does she smile all the time? Does she enjoy writing stories and poetry? I do hope that they can contact us!

  I feel guilty because I was the one driving in the accident which caused Stacey’s death. I also feel proud that my daughter has helped so many people.

  4. When did Stacey die from the traffic accident?

  A. On May 28th, 1998

  B. On May 29th, 1998

  C. On May 31st, 1998

  D. On May 30th, 1998

  5. What does the underlined word “They” refers to?

  A. The pictures taken after the transplant.

  B. The people who received the organs of Stacey.

  C. The people who cared for the girl in the hospital.

  D. The pictures showing the effects of the Nephrotic Syndrome.

  6.

  What is the writer’s attitude towards her daughter’s organ donation?

  A. Grateful

  B. Dissatisfied

  C. Proud

  D. Guilty

  7. What has the writer been looking forward to doing?

  A. Contacting the kidney recipient’s family.

  B. Visiting kidney recipient’s family.

  C. Getting paid from the heart family.

  D. Seeing the heart recipient.

  7.B【山东省日照市第一中学2016届高三上学期期末考试】Seated in a convertible (敞篷车) with the top down at 60 miles an hour, Mary looked at the flies that sat on top of Mark’s hair. How could they stay there? She wondered.

  Mary had not seen Mark in years, until the other day when he came into the café where she worked. His appearance was different from when he was in high school. Now, he was a bit fat and thick glasses covered his eyes. However, that didn’t change Mary’s feelings for him at all. He had just bought a brand new red sports car and asked her if she would like to go for a ride and then have dinner with him. Her heart beat with excitement as he opened the door for her. She noticed something very strange as Mark got into the car and started to drive away. On the hair was a group of flies just sitting there.

  Mary stared in amazement. It seemed the faster they drove, the more determined the flies were to stick to his hair.

  Mary remained silent. She leaned forward and turned the radio on, to try to divert her attention away from the flies, but she couldn’t help thinking about them. She looked up at Mark, who was completely devoted to what was going on and continued to sing to the music while driving merrily along.

  Finally, Mark pulled the vehicle over to the side of the road beside a diner and looked in the mirror and said, “Mary, I would like to comb my hair before we get something to eat. Could you reach under the seat and give me my hair cream?”

  Mary reached under the seat and pulled out a camping backpack, which contained a round jar and started to laugh wildly as she pointed to the label (商标).

  “That’s not hair cream on your hair, Mark! It says it’s Fly Trap Glue!”

  24. What can we learn from the text?

  A. Mark had bought a new sports car for Mary.

  B. Mark used to be a thin, handsome boy.

  C. Mark and Mary had been in love for years.

  D. Mary felt strange when she saw the red car.

  25. What does the underlined word “divert” in Paragraph 4 probably mean?

  A. Replace.

  B. Absorb.

  C. Take.

  D. Recover.

  26. Why did Mary burst into laughter?

  A. Because Mary played a trick on Mark.

  B. Because their ride wasn’t well prepared.

  C. Because Mark had prepared a jar of hair cream for her.

  D. Because Mark had mistaken Fly Trap Glue for hair cream.

  27. What can be the best title for this passage?

  A. Stuck on you. B. Unchanged love.

  C. A hair trick.

  D. A wonderful ride.

  8.A【黑龙江省哈尔滨师大附中、东北师大附中、辽宁省实验中学2016届三高第一次三校联合模拟】When I was young, my father had one of the first telephones in our neighborhood. I remember the shiny box on the wall. But I was too little to reach it, I could only listen to my mother talk on it.

  I discovered that inside the wonderful box lived an amazing person whose name was Information Please. There was nothing she did not know.

  One day my mother went out. I accidentally hurt my finger when I was playing with a hammer. I walked around the house sucking my hurting finger, finally seeing the telephone. Quickly I dragged a chair over and climbed up.

  “Information Please,” I spoke into the receiver. A click or two and a small clear voice spoke into my ear, “Information.”

  “I hurt my finger…” I cried into the phone.

  “Isn’t your mother home?” came the question.

  “Nobody’s home but me.” I sobbed.

  “Are you bleeding?” the voice asked.

  “No,” I replied. “I hit my finger with the hammer and it hurts.”

  “Can you open your icebox?” she asked. I said I could.

  “Then take a little piece of ice and hold it to your finger.” Said the voice.

  After that, I called “Information Please” for everything. I asked her for help with my geography and she told me where Philadelphia was. She helped me with my math. She told me her name was Sally.

  Then there was the time my pet canary (金丝雀) died. I called “Information Please” and told Sally the sad story. “Why is it that birds sing so beautifully and bring joy to all families, only to end up as a pile of feathers?” She listened, then said quietly, “Paul, always remember that there are other worlds to sing in.” Somehow I felt better.

  One day when I called “Information Please” again, a different voice answered “Information.”

  I asked for Sally. “Are you a friend?” she said.

  “Yes.” I answered

  “I’m sorry to have to tell you this,” she said. “She died five days ago.” Before I could hang up she said, “Wait a minute. Did you say your name was Paul?”

  “Yes!”

  “Well, Sally left a message for you. The note said, “Tell him I still say there are other worlds to sing in. He’ll know what I mean.” I thanked her and hung up. I knew what Sally meant.

  21. When the author hurt his finger, he rang because______.

  A. he believed that the telephone knew everything.

  B. the telephone could tell him a doctor’s number

  C. his mother was on the other end of the telephone

  D. he was once told to ask for help in this way

  22.

  Through the help from Sally, it can be inferred that______.

  A. Sally was a geography teacher

  B. Sally was the author’s friend

  C. Sally was a considerate person

  D. Sally was an imaginary person

  23. What did Sally really mean by her message?

  A. She was sure that she could sing in other worlds.

  B. She didn’t want Paul to be sad about her death.

  C. Singing helped Paul to face death more easily.

  D. There was no need for Paul to call her anymore.

  24. What is the best title for this passage?

  A. My Pet’s Death

  B. An Amazing Woman

  C. A Healing Message

  D. My Magic Box

  9.B【黑龙江省哈尔滨师大附中、东北师大附中、辽宁省实验中学2016届三高第一次三校联合模拟】Tu Youyou has become the first Chinese woman to win a Nobel Prize, for her work in helping to create an anti-malaria(疟疾) medicine. The 84-year-old’s route to the honour has been anything but traditional. In China, she is being called the "three nos" winner: no medical degree, no doctorate, and she’s never worked overseas.

  In 1967, malaria, a then deadly disease, spread by mosquitoes was decimating Chinese soldiers fighting Americans in the jungles of northern Vietnam. A secret research unit “Mission 523 “was formed to find a cure for the illness. Two years later, Tu Youyou was instructed to become the new head of “Mission 523”.

  “Mission523” read ancient books carefully for a long time to find historical methods of fighting malaria. When she started her search for an anti-malarial drug, more than 240,000 compounds(化合物) around the world had already been tested, without any success. Finally, the team found a brief reference to one substance, sweet wormwood(青蒿), which had been used to treat malaria in China around 400 AD. The team took out one active compound in wormwood, and then tested it. But nothing was effective until Tu Youyou returned to the original ancient text. After another careful reading, she changed the drug recipe one final time, heating the compound without allowing it to reach boiling point.

  After the drug showed promising results in mice and monkeys, Tu Youyou volunteered to be the first human recipient of the new drug. “

  In any case, Tu Youyou is consistently praised for her drive and passion. One former colleague. Lianda Li, says Ms Tu is “unsociable and quite straightforward”, adding that “if she disagrees with something, she will say it.”

      Another colleague, Fuming Liao, who has worked with Tu Youyou for more than 40 years, describes her as a “tough and stubborn woman”. Stubborn enough to spend decades piecing together ancient texts, she applies them to modern scientific practices. The result has saved millions of lives.

  25. According to Para. I, we can learn that_______.

  A. Tu is the first woman to win a Nobel Prize

  B. Tu has a medical degree

  C. Tu’s road to success is not traditional

  D. Tu discovered a cure for malaria

  26.

  The underlined word “decimating” can be replaced by_______.

  A. encouraging

  B. killing

  C. annoying

  D. benefiting

  27.

  Which of the following statements is true?

  A. Tu first invented the idea of using sweet wormwood as a cure.

  B. Tu was inspired by medical textbooks published in northern vietnem.

  C. The compound needs to be heated to the boiling point to be effective.

  D. Over 240,000 compounds were proved ineffective before Tu’s seareh.

  28. Tu Youyou can be best described as a _______ person.

  A. devoted and stubborn

  B. straightforward and mean

  C. considerate and tough

  D. sociable and generous

  10.A 【海南省文昌中学2016届高三上学期期末】 After spending a year in Brazil on a student exchange program, her mother recalled(回忆), Marie Colvin returned home to find that her classmates had narrowed down their college choices. “Everyone else was already admitted to college,” her mother, Rosemarie Colvin, said from the family home. “So she took our car and drove up to Yale and said , ‘You have to let me in .’ ”

  “Impressed--she was a National Merit (全国英才) finalist who had picked up Portuguese in Brazil--Yale did, admitting her to the class of 1978, where she started writing for the Yale Daily News and decided to be a journalist,” her mother said.

  On Wednesday, Marie Colvin, 56, an experienced journalist for The Sunday Times of London, was killed as Syrian forces shelled(炮击) the city of Homs. She was working in a temporary media center that was destroyed in the attack.

  “She was supposed to leave Syria on Wednesday”, Mrs. Colvin said. “Her editor told me he called her yesterday and said it was getting too dangerous and they wanted to take her out. She said she was doing a story and she wanted to finish it.”

  Mrs. Colvin said it was pointless to try to prevent her daughter from going to conflict zones. “If you knew my daughter,” she said, “it would have been such a waste of words. She was determined, she was enthusiastic about what she did, it was her life. There was no saying ‘Don’t do this.’ This is who she was , absolutely who she was and what she believed in :cover the story, not just have pictures of it, but bring it to life in the deepest way you could.” “So it was not a surprise when she took an interest in journalism,” her mother said.

  From the underlined sentence in Paragraph 1, we can infer that ________ .

  A. Yale was her last choice

  B. Marie Colvin was confident of herself

  C. Yale must keep its promise to Marie Colvin

  D. Marie Colvin was good at persuading

  Which of the following is the correct order to describe Marie Colvin’s life ?

  a. She was doing a story in Syria and got killed.

  b. She was admitted to Yale University.

  c. She studied in Brazil as an excellent student.

  d. She was hired by The Sunday Times of London.

  e. She began to take an interest in journalism.

  A. d→e→c→a→b

  B. b→c→d→e→a

  C. e→d→c→b→a

  D. c→b→e→d→a

  23. From the last paragraph, we can know that Mrs. Colvin ________.

  A. dislikes the choice of her daughter. B. cares little about her daughter.

  C. knows her daughter very well.

  D. doesn’t fully appreciate her daughter.

  What can be the best title of the text ?

  A. Covering Stories in a Dangerous Conflict Area.

  B. Applying for Top Universities, a Successful Case

  C. Choosing Lifelong Careers Based on Your Own Interest.

  D. Recalling Her Daughter, a Journalist Killed in Syria.

  11.A 【云南省玉溪市一中学2016届高三下学期第一次月考】Twelve years ago, I arrived in Central Florida from Puerto Rico. I had heard of a job opportunity and decided to pursue it. But it never became a reality. I quickly learned that being alone without resources in an unfamiliar city was not a comfortable situation to be in.

  Once my limited funds ran out I became homeless and spent a year and a half living on the streets of Orlando. Apart from feeling not being noticed and missing my family, I had to face another challenge—hunger.

  For the first time in my life, I, as a foreign man, who had lived my life in the relative comfort of the middle class, understood the desperation someone feels when they don’t get enough to eat. I clearly remembered having run a distance of more than 7 miles on many occasions just to make it to a local feeding program before they closed at 7 pm.

  Once the need for food was met, the next challenge would arise—where to find a place to sleep for the night. Fortunately, local programs like the Coalition for the Homeless, Second Harvest Food Bank and the Wayne Densch Center were an important part of my ability to survive my painful experience.

  With the assistance of these programs, I was accepted by a college scholarship program through the Coalition and obtained two degrees from Valencia.

  My experience has helped me understand that devoting my life to helping others offers lasting rewards. Today, I am employed as Childhood Hunger Programs Manager at Second Harvest Food Bank and oversee the summer feeding, Hi-Five Kids Pack, and Kids Café programs. I am so proud to be able to distribute food resources to those wonderful programs and help hundreds more like me.

  21. What can be inferred from Paragraph 1?

  A. It was impossible for the author to get a job then.

  B. Many local people were out of work in Florida.

  C. Florida was not a good place for people living there.

  D. The author rejected many chances to get a job in Florida.

  22. Why did the author run a distance of over 7 miles on many occasions?

  A. Because he didn’t want to be late for work there.

  B. Because he desired to get some food to eat.

  C. Because he went there to help the poor.

  D. Because he helped distribute food resources to the poor.

  23. Before being accepted by a college scholarship program, the author_____.

  A. had lived with his relatives happily.

  B. had won his degree in a university.

  C. had been struggling financially.

  D. had lost hope of facing the future.

  24.

  From his experience, the author learns that ______.

  A. misfortune may be an actual blessing.

  B. a friend in need is a friend indeed.

  C. where there is a will there is a way.

  D. one good turn deserves another.

  2015年模拟题

  【2015届浙江省温州市十校联合体(温州中学等)高三联考】A

  Katharine Meyer Graham was once described as “the most powerful woman in America.” She was not a government official or elected representative. She owned and published The Washington Post. Under her leadership, it became one of the most important newspapers in the country.

  Katharine Meyer was born in New York City in 1917. Her father was a successful investment banker and became an important financial official. Her family was very rich. Katharine grew up in large houses in New York and Washington. Her parents were often away from home, traveling and working, Katharine was often lonely. Katherine Meyer graduated from the University of Chicago in Illinois in 1938. In 1933, her father bought a failing newspaper, The Washington Post. It was the least successful one of five newspapers in Washington.

  Katharine Graham returned to Washington and got a job editing letters to the editor of her father's newspaper. She married Philip Graham. He was a clerk for Supreme Court Justice Felix Frankfurter but soon accepted a job at his wife's father's newspaper. Mr. Graham improved The Washington Post. He bought Newsweek and several television stations. He also established close ties with important political leaders. However, Mr. Graham treated his wife badly. He had an affair with a young reporter. For many years, Mr. Graham suffered from mental illness. He killed himself in 1963.

  Katharine Graham had four children to raise and a newspaper to operate. At first, she was only concerned about finding a way to keep control of The Washington Post until her sons were old enough. She did not think she had the ability to do an important job. She had no training in business or experience in operating a large company. In those days, it was unusual for a woman to be the head of a business. Women were expected to look after their homes and children.

  Katharine Graham met with officials of The Washington Post. She was elected president of The Washington Post Company. She had no idea about how to operate a newspaper. So she decided to learn. She began by hiring Benjamin C.Bradlee. He later became chief editor. Mr. Bradlee improved the newspaper. He hired excellent reporters and editors. They began doing important investigative reporting. In 1969, Mrs. Graham became publisher as well as president of The Washington Post Company. In the 1970s, The Washington Post became famous around the world because of two major successes.

  41. Katharine Graham is considered powerful because ____________.

  A. she was born of a very rich family

  B. she had the most important newspaper

  C. she was in charge of the The Washington Post

  D. she gained much money from newspapers

  42. We can infer from the second paragraph that Katharine Graham _____________.

  A. was one of the top students at university

  B. helped her father in purchasing the paper

  C. didn’t take interest in managing a bank

  D. lived a rich life when she was young

  43. After Katharine Graham got married ___________.

  A. her husband gave up his previous job

  B. she got a job editing letters for the editor

  C. she made The Washington Post improved

  D. her husband wanted to be a political leader

  44. Faced with the death of Mr. Graham ____________.

  A. Katharine Graham had confidence to operate the newspaper well.

  B. Katharine Graham made up her mind to raise the kids.

  C. KatharineGraham thought more about gaining experience.

  D. Katharine Graham seemed to be trapped in the dark world.

  45. What do you think of Benjamin C. Bradlee?

  A. He was an expert when it came to earning money.

  B. He brought The Washington Post back to life.

  C. He intended to buy The Washington Post.

  D. He should have given more help to Katharine Graham.

  【英语卷(解析)·2015届浙江省绍兴一中高三】A 

  Fat and shy, Ben Saunders was the last kid in his class picked for any sports team. "Football, tennis, cricket--- anything with a round ball, I was useless," he says now with a laugh. But back then he was the one always made fun of in school gym classes in Devonshire, England.

  It was a mountain bike he received for his 15th birthday that changed him. At first he went biking alone in a nearby forest. Then he began to ride the bike along with a runner friend. Gradually, Saunders set up his mind on building up his body, increasing his speed and strength. At the age of 18, he ran his first marathon.

  The following year he met John Ridgway and was hired as an instructor at Ridgway's school of adventure in Scotland, where he learnt about Ridgway's cold-water exploits. Greatly interested, Saunders read all he could about North Pole explorers and adventures, the decided that this would be his future.

  In 2001, after becoming a skillful skier, Saunders started his first long-distance expedition towards the North Pole. It took unbelievable energy. He suffered frostbite, ran into a polar bear and pushed his body to the limit, pulling his supply-loaded sled up and over rocky rice.

  Saunders has since become the youngest person to ski alone to the North Pole, and he's skied more of the North Pole by himself than any other British man. His old playmates would not believe the change.

  Next October, Saunders, 27, heads south from the coast of Antarctica to the South Pole and back, a 2900-kilometer journey that has never been completed on skis. 56. What change happened to Saunders after he was 15 years old?  

  A. He became good at most sports.  

  B. He began to build up his body.  

  C. He joined a sports team.     

  D. He made friends with a runner. 57. The underlined word “exploits” (paragraph 3) is closest in meaning to ______.  

  A. journeys   B. researches   C. adventures   D. operations. W 58. What does the story mainly tell us abut Saunders?  

  A. He is a success in sports.    

  B. He is the best British skier.  

  C. He is Ridgway's best student. `        D. He is a good instructor at school. 【英语卷(解析)·2015届宁夏银川一中高三上学期】C

  The drug store was closing for the night. Young Alfred Higgins, the shop-assistant, was ready to go home. Mr. Carr, the boss, stared at him and said “Hold on, Alfred. Maybe you'd be good enough to take the things out of your pockets and leave them here before you go.” Alfred's face got red. After a little hesitation, he took out what he had stolen. Mr. Carr said, “Maybe I should call your mother and let her know I'm going to have to put you in prison.”

  Alfred thought his mother would come rushing in, eyes burning with anger. But, to his surprise, she arrived wearing a smile. “Hello, I'm Alfred's mother. Is he in trouble?” she said. Mr. Carr was surprised, too. He had expected Alfred's mother to come in nervously, shaking with fear, asking with wet eyes for a mercy for her son. But no, she was most calm, quiet and pleasant and was making Mr. Carr feel guilty…. Soon Mr. Carr was shaking his head in agreement with what she was saying. “Of course”, he said, “I don't want to be cruel. You are right. Sometimes, a little good advice is the best thing for a boy at certain times in his life and it often takes the youths long time to get sense into their heads.” And he warmly shook Mrs. Higgins's hand.

  Back home, without even looking at Alfred, she said, “You are a bad luck. It is one thing after another, always has been. Why do you stand there so stupidly? Go to bed.” In his bedroom, Alfred heard his mother in the kitchen. There was no shame in him, just pride in his mother's strength. He felt he must tell her how great she was. As he got to the kitchen, he saw his mother drinking a cup of tea. He was shocked by what he saw. His mother's face was a frightened, broken one. It was not the same cool, bright face he saw earlier in the drug store. As she lifted the tea cup, her hand shook. And some of the tea splashed on the table. Her lips moved nervously. She looked very old. He watched his mother without making a sound. The picture of his mother made him want to cry. He felt his youth coming to an end. He saw all the troubles he brought his mother in her shaking hand and the deep lines of worry in her grey face. It seemed to him for the first time he had ever really seen his mother.

  44. Which of the following is probably said by Mrs. Higgins while talking to Mr. Carr?

  A. “Please, for God's sake, you know, he is just a kid.”

  B. “I can't believe it! You are treating my son like that!”

  C. “You know, it takes time for a youth to truly grow up.”

  D. “Punishment makes sense because it teaches about the law.”

  45. What does the underlined phrase "his mother's strength"(Para. 3) refer to?

  A. Mrs. Higgins's calmness and communicating skills.

  B. Mrs. Higgins's attitude towards parental sufferings.

  C. Mrs. Higgins's love and care for her son Alfred.

  D. Mrs. Higgins's greatness as a woman.

  46. After the incident, young Alfred would probably ________.

  A. feel ashamed of her mother

  B. change his attitudes towards life

  C. learn to live an independent life

  D. hate Mr. Carr for hurting her mother

  【英语卷·2015届河南省顶级名校高三定位考试】B

  In the fall of 1985, I was a bright-eyed girl heading off to Howard University, aiming at a legal career and dreaming of sitting on a Supreme Court bench somewhere. Twenty-one years later I am still a bright-eyed dreamer and one with quite a different tale to tell.

  My grandma, an amazing woman, graduated from college at the age of 65. She was the first in our family to reach that goal. But one year after I started college, she developed cancer. I made the choice to withdraw from college to care for her. It meant that school and my personal dream would have to wait.

  Then I got married with another dream: building my family with a combination of adopted and biological children. In 1999, we adopted our first son. To lay eyes on him was fantastic---and very emotional. A year later came our second adopted boy. Then followed son No. 3. In 2003, I gave birth to another boy.

  You can imagine how fully occupied I became, raising four boys under the age of 8. Our home was a complete zoo---a joyous zoo. Not surprising, I never did make it back to college full-time. But I never gave up on the dream either. I had only one choice: to find a way. That meant taking as few as one class each semester.

  The hardest part was feeling guilty about the time I spent away from the boys. They often wanted me to stay home with them. There certainly were times I wanted to quit, but I knew I should set an example for them to follow through the rest of their lives.

  In 2007, I graduated from the University of North Carolina. It took me over 21 years to get my college degree!

  I am not special, just single-minded. It always struck me that when you’re looking at a big challenge from the outside it looks huge, but when you’re in the midst of it, it just seems normal. Everything you want won’t arrive in your life on one day. It’s a process. Remember: little steps add up to big dreams.

  6. When the author went to Howard University, her dream was to be __________.

  A. a writer

  B. a teacher

  C. a judge

  D. a doctor

  7. Why did the author quit school in her second year of college?

  A. She wanted to study by herself.

  B. She fell in love and got married.

  C. She suffered from a serious illness.

  D. She decided to look after her grandma.

  8. What can we learn about the author from Paragraphs 4 and 5?

  A. She was busy yet happy with her family life.

  B. She ignored her guilty feeling for her sons.

  C. She wanted to remain a full-time housewife.

  D. She was too confused to make a correct choice.

  9. What does the author mostly want to tell us in the last paragraph?

  A. Failure is the mother of success.

  B. Little by little, one goes far.

  C. Every coin has two sides.

  D. Well begun, half done.

  10. Which of the following can best describe the author?

  A. Caring and determined.

  B. Honest and responsible.

  C. Ambitious and sensitive.

  D. Innocent and single-minded.

  【英语卷·2015届河北省唐山市高三模拟考试】A

  My 16-year-old son, Anton, had gone to the local swimming hole. Most of the kids swim there, and there are plenty of rocks for them to use as safe harbors, so I had no fears for his safety.

  Still, the firefighter's first words "You need to come up here to the Stillwater River" made me catch my breath, and his follow-up words gave me relief: “ Your son is OK.”

  When I got to the river, I immediately saw the firetruck, ambulance and Anton, wrapped with a towel about his shoulders, sitting quietly on a low platform of the fire engine.

  I hurried over to him. "You OK?"I asked.

  ”Yeah," was all he said. But my eyes begged for an explanation, I didn't get it from my son, however, who tends to play his cards close to his vest.

  The story was this: A woman was being swept under water. Hearing the cries, Anton and his friend Tyler, without hesitation, swam out to her, and brought her safely to shore.

  In an age in which the word "hero" is broadcast with abandon and seemingly applied to anyone who make it through the day, I realized the real thing in my son. The teens are stubborn and self-centred, but that didn't mean they have no desire to do good.

  Still shocked by my son's daring, I drove him home. Along the way, I tried to dig out some more information from him - but he had precious little to say. The only words he said were,

  ”What's for supper?"

  I spent some time alone that evening, thinking about the tragedy that might have been. The next morning, when Anton got up, I half expected him to tell me the story. But all he did was toast some bread, pull himself together, and head for the door to start a new day. Watching from the window,1 was reminded that still water often runs deep.

  21.Why did the mother allow her son to swim there?

  A. He was an excellent swimmer.

  B.The water of the river is shallow.

  C. He was old enough to swim.

  D.The rocks can be of help if there's danger.

  22.The underlined part "who tends to play his cards close to his vest" probably means

  A. Anton is a boy fond of swimming with other kids

  B. Anton is unwilling to tell others what he thinks

  C. Anton always has a desire to help others

  D. Anton seldom changes his mind

  23.In the mother's eyes, what her son did was

  A. dangerous but interesting

  C. unexpected and courageous

  B. meaningful but difficult

  D. awful and absurd

  24. What might be the best title for the passage?

  A. My Son, My Hero

  B. Anton, A Silent Boy

  C. A Good Deed

  D.A Proud Mother

  A

  I left university with a good degree in English Literature, but no sense of what I wanted to do. Over the next six years, I was treading water, just trying to earn an income. I tried journalism, but I didn’t think I was any good, then finance, which I hated. Finally, I got a job as a rights assistant at a famous publisher. I loved working with books, although the job that I did was dull.

  I had enough savings to take a year off work, and I decided to try to satisfy a deep down wish to write a novel. Attending a Novel Writing MA course gave me the structure I needed to write my first 55,000 words. It takes confidence to make a new start

  there’s a dark period in-between where you’re neither one thing nor the other. You’re out for dinner and people ask what you do, and you’re too ashamed to say, Well, I’m writing a novel, but I’m not quite sure if I’m going to get there. My confidence dived. Believing my novel could not be published, I put it aside.

  Then I met an agent(代理商)who said I should send my novel out to agents. So, I did and, to my surprise, got some wonderful feedback. I felt a little hope that I might actually become a published writer and, after signing with an agent, I finished the second half of the novel.

  The next problem was finding a publisher. After two-and-a-half years of no income, just waiting and wondering, a publisher offered me a book deal — that publisher turned out to be the one I once worked for.

  It feels like an unbelievable stroke of luck — of fate, really. When you set out to do something different, there’s no end in sight, so to find myself in a position where I now have my own name on a contract(合同)of the publisher — to be a published writer — is unbelievably rewarding.

  21. The author decided to write a novel ______.

  A. to finish the writing course

  B. to realize her own dream

  C. to satisfy readers’ wish

  D. to earn more money

  22. How did the writer feel halfway with the novel?

  A. Disturbed.

  B. Ashamed.

  C. Confident

  D. Uncertain.

  23. What does the author mainly want to tell readers in the last paragraph?

  A. It pays to stick to one’s goal.

  B. Hard work can lead to success.

  C. She feels like being unexpectedly lucky.

  D. There is no end in sight when starting to do something.

  【英语卷·2015届河北省邯郸市馆陶县一中高三调研考试(201707)】

  His first successful fight was for the equal rights of black people in South Africa. Then, as the first black president, he fought to unite the country and organize the government. Now Nelson Mandela has set his sights on a new enemy, AIDS.

  On March 19 the former president, hosted his second AIDS­awareness concert. He warned that 25 million people in Africa were already infected with the fatal disease.

  Mandela was born in a small village in South Africa in 1918. He was adopted by the chief of his tribe and could have been a chief himself and lived a happy country life.

  But he refused to be a chief when his people lived under racial discrimination(歧视). He decided to fight for equal rights for all the people in South Africa. Before 1990, under the country's Racial Segregation Law, coloured people and white people lived separately. Black people were treated unfairly even when taking a bus. Blacks had to stand at the back of the bus to make room for white people even when there were only a few of them on board.

  For his opposition to the system, Mandela was arrested and spent 27 years in prison. He was freed in 1990 and became the president of the country after the first election was held in which everyone could vote.

  Mandela was not only a political fighter who attacked with speeches. He was also a trained boxer and fought in the ring when he was young.

  “Although I did not enjoy the violence of boxing, I was

  interested in how one moved one's body to protect oneself, how one used a strategy both to attack and retreat,”he wrote in his autobiography.

  As a skillful fighter, he chose music as his weapon against AIDS. He hopes to win another victory against AIDS.

  33.When was Mandela arrested?

  A.In 1963.

  B.In 1990.

  C.When he refused to be a chief.

  D.When he became the president.

  34.Nelson Mandela succeeded in doing the following EXCEPT ________.

  A.winning the equal rights for the black people in South

  Africa

  B.uniting South Africa

  C.organizing a government in South Africa

  D.controlling the spread of AIDS

  35.Which of the following statements can best describe the life of Nelson Mandela?

  A.Struggle is his life.

  B.Sports make his fame.

  C.Fight for equal rights.

  D.A great fighter against the government.

  B【北京市东城区2015届高三上学期期末教学统一检测】

  Rescue

  The girl was hanging by her hands from the railings of a balcony(阳台的栏杆). The balcony was on the twelfth floor of the high-rise block next to his. His flat was on the ninth floor and he had to look up to see her. It was half-past six in the morning. He had been awakened by the sound of an aircraft flying dangerously low overhead, and had got out of bed to look. His sleepy eyes, moved from the blue sky which was empty of cloud, empty of anything but the bright disappearing arrow of the aircraft, and then rested on the hanging figure.

  He really thought he must be dreaming, for this sunrise time was the hour for dreams. Then, when he knew he wasn’t, he decided it must be a scene in a film. There were cameramen down there, a whole film unit, and all the correct safety precautions had been taken. Probably the girl wasn’t even a real girl, but a dummy(假人). He opened the window and looked down. The car park, paved courts, grass spaces between the blocks, all were deserted. On the balcony rail one of the dummy’s hands moved, desperately. He had to believe then what was obviously happening. The girl was trying to kill herself. She had lost her courage and now was trying to stay alive. All these thoughts and conclusions of his occupied about thirty seconds. Then he acted. He picked up the phone and dialed the emergency number for the police.

  The arrival of the police cars and the rescue of the girl became the focus of talk for the people of the two blocks. Someone found out that it was he who had called the police and he became an unwilling hero. He was a modest, quiet young man, and was in relief when the talk began to die away. Again he was able to enter and leave his flat without being pointed at as a kind of St George and sometimes even congratulated.

  About a fortnight after that morning, he was getting ready to go to the theatre, just putting on his overcoat, when the doorbell rang. He didn’t recognize the girl who stood outside. He had never seen her face. She said, “I’m Lydia Simpson. You saved my life. I’ve come to thank you.”

  60. What did the man do first after he got up?

  A. He looked down from the window.

  B. He went to see the noisy aircraft.

  C. He called the police to save the girl.

  D. He hurried to check who was outside.

  61. The moment the man saw the girl hanging there, he felt it was _________.

  A.

  scaring

  B. desperate

  C. unbelievable

  D. dangerous

  62. We can learn from the passage that __________.

  A. the man disliked to be talked about

  B. the girl was unwilling to be rescued

  C. the police arrived quickly on the scene

  D. the girl was actually an actress in a film

  A【河北省正定中学2015届高三1月月考】

  In 1939 two brothers, Mac and Dick McDonald, started a drive-in restaurant in San Bernadino, California. They carefully chose a busy corner for their location. They had run their own businesses for years, first a theater, then a barbecue(烤肉) restaurant, then another drive-in. But in their new operation, they offered a new, shortened menu: French fries, hamburgers, and sodas. To this small selection they added one new idea: quick service, no waiters or waitresses, and no tips.

  Their hamburgers sold for fifteen cents. Cheese was another four cents. Their French fries and hamburgers had a remarkable uniformity(一致性), for the brothers had developed a strict routine(程序) for the preparation of their food, and they insisted on their cooks sticking to their routine. Their new drive-in became surprisingly popular, particularly for lunch. People drove up by the hundreds during the busy noontime. The self-service restaurant was so popular that the brothers had allowed ten copies of their restaurant to be opened. They were content with this modest success until they met Ray Kroc.

  Kroc was a salesman who met the McDonald brothers in 1954, when he was selling milkshake-mixing machines. He quickly saw the special attraction of the brothers’ fast-food restaurants and bought the right to franchise(特许经营) other copies of their restaurants. The agreement included the right to duplicate(复制) the menu, the equipment, even their red and white buildings with the golden arches(拱门).

  Today McDonald’s is really a household name. In 1976, McDonald’s had over $ l billion in total sales. Its first twenty-two years is one of the most surprising success stories in modern American business history.

  .56. This passage mainly talks about _______.

  A. the development of fast food services

  B. how McDonald’s became a billion-dollar business

  C. the business careers of Mac and Dick McDonald

  D. Ray Kroc’s business talent

  .57. Mac and Dick managed all of the following businesses except _______.

  A. a drive-in

  B. a theater

  C. a cinema D. a barbecue restaurant

  .58. We may infer from this passage that _______.

  A. Mac and Dick McDonald never became wealthy for they sold their idea to Kroc.

  B. the place the McDonalds chose was the only source of the great popularity of their drive-in

  C. forty years ago there were lots of fast-food restaurants

  D. Ray Kroc was a good businessman

  .59. The passage suggests that _______.

  A. creativity is an important element of business success

  B. Ray Kroc was the close partner of the McDonald brothers

  C. Mac and Dick McDonald became broken after they sold their ideas to Ray Kroc

  D. California is the best place to go into business

  B【河北省邯郸市2015届高三上学期1月份教学质量检测】

  A British shopper, Emma Mumford, who was named “the Coupon(优惠券) Queen” has spent just £350 on a luxury(奢侈) Christmas for her and her familybut got £2,500 worth of food, drink and presents thanks to coupons and offers.

  Ms Mumford is a so-called “extreme couponer” and spends hours hunting out the best deals and bargains. She has been preparing for this Christmas since the end of the last one. Ms Mumford made the huge savings by checking price comparison websites and as a result collected more presents than she needed for Christmas. After sorting out gifts for all of her family, she has even been able to donate £1,000 worth of presents to her local hospital.

  “It has been so nice to have Christmas sorted and not having to worry about breaking the bank. I wanted to get high quality presents for my family and show that you can do it without spending a fortune. There is a misconception that everyone who does couponing only gets value products but through my careful shopping I have managed to purchase designer perfumes(香水), a high-end coffee machine.” she explained.

  She said “What started as a hobby to help me save money has turned into my life and I am able to run a business helping people save money. I know what it feels like to struggle as my ex-partner had a lot of debt which I took on for him. We were in a poor situation where every penny counted. That’s what really inspired me.”

  She has now turned her bargain-hunting addiction into a business where she searches for the best deals and offer tips to the general public through video blogging and her Facebook Page.

  “I try and show people that it is just a little bit of work in the beginning but it will eventually pay off. One of the best things to do is just to be a bit forward thinking and save throughout the year.”

  How did Emma make the huge savings?

  She compared prices through websites.

  She hunted out bargains in real shops.

  She used coupons and offers worth £2,500.

  She collected more Christmas presents than needed.

  What does “breaking the bank” in the third paragraph mean?

  A. Buying presents of low quality.

  B. Spending more money than you have.

  C. Breaking into the bank.

  D. Taking out money from the bank.

  What really inspired Emma to help people save money?

  A. Her addiction to bargain-hunting.

  B. The financial difficulties she shared with her ex-partner.

  C. The poor situation people around her were going through.

  D. Her experience of running an unsuccessful business.

  Which of the following can best describe Emma?

  A. Ambitious and creative.

  B. Sensitive and determined.

  C. Brave and lucky.

  D. Careful and caring.

  A【江西省吉安市第一中学2015届高三上学期第二次阶段考】

  Noah Webster, born on October 16, 1758, is known for The American Dictionary of the English Language. He has been called the" Father of American Scholarship and Education. " His " Blue-Backed Speller" books were used to teach spelling and reading to five generations of American children. But how much do you know about him beyond that?

  At the age of 16, Noah Webster began attending Yale College, Unfortunately, he spent his four years at Yale during the American Revolutionary War, and, because of food shortages, many of his college classes were held in Glastonbury, Connecticut. Later, he served in the army. Having graduated from Yale in 1778, Webster wanted to continue his education in order to earn his law degree. He had to teach school in order to pay for his education. He set up many small schools that didn’t survive, but he was a good teacher because instead of forcing his students to learn, like what most teachers did, he rewarded them.

  He earned his law degree in, 1781, but did not practice law until 1789. Once he started he found the law was not to his liking. Webster did not have much money.

  In 1793, Alexander Hamilton lent him $ 1500 to move to New York City to edit a newspaper. In December, he founded New York’s first daily newspaper, American Minerva, and edited it for four years. For decades, he published textbooks, political essays, a report on some diseases, and newspaper articles for his party. He wrote so much that a modern list of his published works required 655 pages.

  Noah Webster died on May 28, 1843 and was buried in the Grove Street Cemetery.

  . Which of the following best describes Webster’s life at Yale?

  A. Tough.

  B. Smooth.

  C. Normal.

  D. Tiring.

  . What did Noah want to be after graduation from Yale?

  A. A teacher.

  B. A lawyer.

  C. A headmaster.

  D. A scholar.

  . What can we infer from Paragraph 4?

  A. Webster was a most productive author.

  B. Webster led a miserable life in New York City.

  C. Webster’s books added up to 655 pages.

  D. Webster didn’t write any political Works.

  D【山西省大同一中、同煤一中2015届高三联考】

  When the swim season began, my 11-year-old daughter, Elizabeth, and I cut a deal. She would go to practice three times a week, and I wouldn’t make her compete in swim meets.

  Elizabeth does not like swim meets, she gets horribly nervous because she is afraid that she will do something wrong and let everyone down. she started to talk about quitting swimming, which broke my heart because she loves swimming. So I came up with the deal.

  Recently, Elizabeth’s team announced a T-shirt relay, which works like this: One person from each relay team puts on a T-shirt and a pair of socks and swims 50 meters. She takes off the clothes and put them on the next person, who then swims 50 meters. This continues until everyone on the team has completed a lap.

  It wasn’t exactly a meet, because it would involve only team members. But Elizabeth thought it was. I told Elizabeth I really wanted her to go. She fought back angrily but finally agreed.

  When the day for the T-shirt relay arrived, Elizabeth was nervous. She was chosen to swim the anchor leg (最后一棒).By the last leg, Elizabeth’s team had built up a narrow lead. Then it was Elizabeth’s turn to swim.

  Approaching the halfway mark, she was still in the lead. Then somebody noticed that one of

  Elizabeth’s socks had fallen off and was floating in the pool. “She has to get that sock on before the end of the race,” a swimming official told Elizabeth’s team,“ or you will be disqualified.”

  Everybody on her team started shouting, “Elizabeth! Get the sock!” But she couldn’t hear them. Meanwhile, a girl in lane two was gaining on Elizabeth. Just then, a girl on my daughter’s team jumped in the pool, grabbed the sock, swam after Elizabeth and put the sock on Elizabeth.

  With the sock finally on, Elizabeth swam her heart out for the last 15 meters and won! There was much celebration. And, for a few minutes, Elizabeth was the hero.

  On the ride home, she relived her moment of glory again and again. She told me that if the T-shirt relay was an Olympic event, her team would win the gold medal, I told her that in my professional opinion, she Was absolutely right.

  What do we know about the T-shirt relay?

  A. Elizabeth was eager to attend it.

  B. Elizabeth made full preparations for it.

  C. Elizabeth thought she was sure to fail the relay.

  D. Elizabeth agreed to attend it after a lot of persuasion.

  What happened to Elizabeth when she was swimming the anchor leg?

  A. The girl on the other team swam faster than her.

  B. She was disqualified for breaking the rule.

  C. She was too nervous to swim.

  D. One of her socks fell off.

  We can infer from the last paragraph that Elizabeth_____.

  A. believed she was the best of her team.

  B. Hoped to take part in the Olympics.

  C. Overcame her fear of swim meets.

  D. Was grateful for the girl’s help.

  What would be the best title for the text?

  A. Born to be a swimmer

  B. Swimming in socks

  C. The swim season

  D. Never give up!

  C【山东省实验中学第三次诊断考试】

  Kenji Croman has broken his bones,his shoulder and elbow, and nearly drowned several times.He’s also had three close encounters with sharks over the years.A 10-foot wave landed directly on top of him five years ago,bending his body to the point that he actually kicked himself in the head.”I literally heard every bone in my body crack.”he said.

  Croman has been photographing ocean waves since 2008,sometimes risking his life to get that perfect shot at the surf break.Despite the risks and certain injuries that come with wave photography, the Hawaii resident,body surfer and former competitive swimmer wouldn’t do anything else.The 36-year-old photographer loves the thrill of meeting breaking waves head on,shooting waves as his interest and shooting surfers to pay the bills.

  The process of photographing waves requires more than just swimming out to where the surf breaks and waiting,he says.The night before a photo shoot Croman checks Surfline.com,a website and the coastal camera network that provides live and predicted ocean weather information,to see what the winds,waves and tide will be like at certain times.Croman has both a primary camera and a backup for emergencies.He goes without a wetsuit and uses heavy duty underwater housing to protect his camera.There is no great way for him to protect himself.He admits there are probably protective clothes he should be wearing,but there is little he could have done to decrease the impact of the 10-foot wave that crashed on top of him.

  The hardest part of Croman’s job is staying calm in the face of breaking waves when his initial reaction is to panic and swim away.At most.Croman will take two to three shots of a single wave.”I've shot waves for so many years now that I see the wave in slow movement,”he said.

  What do we know about Kenji Croman’s job from the first paragraph?

  A.It is a game of death.

  B.It is full of mysteries.

  C.It requires patience.

  D.It calls for courage.

  How does Kenji Croman make a living?

  A. By taking swimming races.

  B.By teaching photography.

  C.By shooting surfers.

  D.By surfing waves.

  Why does Kenji Croman check the websites like Surfline.com?

  A.To get better prepared for shooting.

  B.To predict something about weather.

  C.To learn more about surfing waves.

  D.To buy better equipment for safety.

  What would be the best title of the text?

  A.Taking adventures around the ocean

  B.Risking his life for the perfect wave

  C.Making preparations for life risks

  D.Appreciating breaking waves in danger

  A

  My passion started decades ago as a kid on a flight to India. I was attracted by how such a large airplane was able to glide(滑翔)smoothly above the clouds. I made it a goal to become a pilot. I am now a pilot for American airlines. Though being a pilot consumes my life,it is a job worth doing.

  My job is slightly different compared to that of a typical pilot. Most pilots have their routes they fly. They then familiarize(使…熟悉)themselves with the routes. However, in my case,I participate in any job where a pilot is immediately required. It is true that some days I might have a workday from nine in the morning to five in the afternoon. In fact,some days,I might even begin my work at three in the morning when I am needed to fly. Sometimes, as for the typical day,I do not have one.

  Having been on the job for over two decades,I have learned some important life lessons. As I greet passengers boarding the airplane,I realize I,along with other airline pilots, have enhanced(提高)the lives of those who we are serving. In fact, there is one particular experience that I will never forget. A family was walking in from the gate, explaining that their beloved one was ill and had to be taken to a big hospital in another city immediately. It was at this point that I realized just how important my job was.

  Therefore, my life as an airline pilot is definitely not for everyone. However,my job is something I enjoy thoroughly and will continue to do till I am old enough. As a pilot, I have been able to experience so many different types of situations in different places throughout the world. For anyone interested in piloting, I would recommend going for it.

  The author dreamed of being a pilot when _______.

  A. he was travelling in India

  B. it was about 30 years ago.

  C. he was flying to India as a kid

  D. he was a kid living in India

  What is the main difference between the author’s job and a typical pilot’s?

  A. The author’s working time is not regular.

  B. The author has his regular route he flies.

  C. The author has to get up very early every day.

  D. The daily working time of the author is longer.

  The author realized how important his job was because he _______.

  A. could serve everyone as he does his daily work

  B. could make his guests’ life more convenient

  C. could take the ill patient to hospital in short time

  D. had been to many advanced countries in the world

  What would be the best title for this passage?

  A. My pleasure in serving people

  B. Difference in my job as a pilot

  C. My interest in being a pilot as a boy

  D. My job as an American airline’s pilot

  A【辽宁省锦州市2015届高三上学期考试英语】

  Jimmy is an automotive mechanic, but he lost his job a few months ago. He has a good heart,but always afraid to apply for a new job.

  One day, he gathered up all his strength and decided to attend a job interview. His appoint⁃ment was at 10 a.m. and it was already 8:30. While waiting for a bus, he saw an elderly man wildly kicking the tyre of his car. Obviously there was something wrong with the car. Jimmy immediately went up to lend him a hand. When Jimmy finished working on the car, the old man asked him how much he should pay for the service. Jimmy said there was no need to pay him; he just helped someone in need, and he had to rush for an interview. Then the old man said,“Well, I could take you to the office for your interview. It’s the least I could do. Please, I insist.”Jimmy agreed.

  Upon arrival, Jimmy found a long line of applicants waiting to be interviewed. Finally his name was called. The interviewer was sitting on a large chair facing the office window. Rocking the chair back and forth, he asked,“Do you really need to be interviewed?”Jimmy’s heart sank.“With the way I look now, how could I possibly pass this interview?”He thought to himself.

  Then the interviewer turned the chair and to Jimmy’s surprise, it was the old man he helped earlier in the morning.

  “Sorry I had to keep you waiting, but I was pretty sure I made the right decision to have you as part of our workforce before you even stepped into this office. I just know you’d be a trustworthy worker. Congratulations!”Jimmy sat down and they shared a cup of well-deserved coffee .

  Why did Jimmy apply for a new job?

  A. He was out of work.

  B. He was bored with his job.

  C. He wanted a higher position.

  D. he hoped to find a better boss.

  What did Jimmy see on the way to the interview?

  A. A friend’s car had a flat tyre.

  B. A wild man was pushing a car.

  C. A terrible car accident happened.

  D. An old man’s car broke down.

  Why did the old man offer Jimmy a ride?

  A. He was also to be interviewed.

  B. He needed a traveling companion.

  C. He always helped people in need.

  D. He was thankful to Jimmy .

  How did Jimmy feel on hearing the interviewer’s question?

  A. He was sorry for the other applicants.

  B. There was no hope for him to get the job.

  C. He regretted helping the old man.

  D. The interviewer was very rude.

  C【辽宁省锦州市2015届高三上学期考试英语】

  Driving to a friend’s house on a recent evening, I was attracted by the sight of the full moon rising just above my friend’s roof-tops. I stopped to watch it for a few moments, thinking about what a pity it was that most citizens, myself included, usually miss sights like this because we spend most of our lives indoors.

  My friend had also seen it. He grew up living in a forest in Europe, and the moon meant a lot to him then. It had touched much of his life.

  I know the feeling. Last December I took my seven-year-old daughter to the mountainous jungle of northern India with some friends. We stayed in a forest rest-house with no electricity or running hot water. Our group had campfires outside every night, and indoors when it was too cold outside. The moon grew to its fullest during our trip. Between me and the high mountains lay three or four valleys. Not a light shone in them and not a sound could be heard. It was one of the quietest places I have ever known, a bottomless well of silence. And above me was the full moon,which struck me deeply.

  Today our lives are filled with glass, metal, plastic and fibre-glass. We have televisions, cellphones, papers, electricity, heaters and ovens and air-conditioners, cars, computers.

  Struggling through traffic that evening at the end of a tiring day, most of which was spent in⁃doors, I thought that before long I would like to live in a small cottage. There I will grow vegetables and read books and walk in the mountains, and perhaps write, but not in anger. I may be come an old man there, and wear the bottoms of my trousers rolled and measure out my life in coffee spoons. But I will be able to walk outside on a cold silent night and touch the moon.

  The best title for the passage would be ________.

  A. The pleasures of modern life

  B. Touched by the moon

  C. A bottomless well of silence

  D. Break away from modern life

  What impressed the writer most in the mountainous jungle of northern India?

  A. No modern equipment.

  B. Complete silence.

  C. The nice moonlight.

  D. The high mountains.

  Modern things (Paragragh 4) are mentioned mainly to ________.

  A. show that the writer likes city life very much

  B. tell us that people greatly benefit from modern life

  C. explain that people have less chances to enjoy nature

  D. show that we can also enjoy nature at home through them

  The author wrote the passage to ________.

  A. express the feeling of returning to nature

  B. show the love for the moonlight

  C. advise modern people to learn to live

  D. want to communicate longing for modern life

  A【山东省青岛市2015届高三上学期考试】

  One day a student was walking with his teacher in the countryside.As they went

  along,they saw a pair of old shoes lying in the path.They were a pool farmer’s,who was working in the nearby field.

  The student turned to the teacher saying:“We will hide his shoes,and hide ourselves behind those trees,and wait to see what he will do”

  “My young friend.”answered the teacher,“we should never make fun of the poor. Why not put a coin in each shoe,and then we will hide ourselves and watch?”The student did so and they both hid themselves behind the trees. The poor man soon finished his work,and came across the field to the path where he had left his coat and shoes.

  After putting on his coat,he put his foot into one of his shoes,and felt something hard.Then he bent (弯腰) down to feel what it was,and found the coin. Surprised,he looked at the coin,turned it around and looked at it again.He then looked around,but no person was seen.He put the money into his pocket, and continued to put on the other shoe. His surprise was doubled on finding the other coin.

  He couldn’t control his feelings and fell to his knees,looked up to the sky and expressed his thanks. Then he spoke of his wife,sick and helpless,and his children without bread.He said the help would save them from dying.

  The student stood there deeply moved,and his eyes filled with tears.“Now,”said the teacher, “Are you not much happier than if you had hidden the shoes?”

  When the student saw the shoes,he wanted to_______

  A. steal them

  B. play a joke on the owner

  C. find their owner

  D. give the owner some money

  According to the passage,the teacher is______.

  A.quiet and honest

  B. kind and friendly

  C. patient and clever

  D.strict and careful

  When the farmer saw the second coin, he______

  A. was surprised and decided to find the owner

  B. was worried and looked up at the sky

  C.was very excited and grateful

  D.spoke of his difficulties and asked for more help

  What can we learn from the passage?

  A.True happiness comes from helping others.

  B .All good things come to an end

  C. Where there is a will,there is away

  D. It is never too late to learn

  B

  I work in a nursing home and my job is to take care of the elderly.

  This year, a very sweet old lady that I cared for,Alice, had gone through a difficult time .She got dementia and so she had been in the hospital twice .In November I was finally able to get her back to her“home”.

  Alice had thought her daughter was coming to visit her on Christmas day and that they were going to have the whole family together like the old days. When she finally realized that that happy moment was not going to happen,she was very sad.Knowing that her daughter was coming after Christmas was not enough to make her feel happy,I hated the idea of her being alone on the holiday!

  On Christmas Eve,I gave her a surprise by asking if she could like to go to a candlelight service at church that night

  Instead of taking her to my church,I took her to her old neighborhood church where all her friends were. We got there early and I got her a seat where her friends could see her as they came in.Then soon some of her friends came to the church and they rushed over to greet her and sit with her.

  The candlelight service was beautiful and Alice got a lot of love from her old and new friends there.

  The truth of the story is that I am the one who got the best gift: the smile on Alice’s face

  The underlined word“dementia”in Paragraph 2 probably is_______

  A.a house

  B.a letter

  C.an illness

  D.a plan

  Where did Alice spend her Christmas Eve according to the story?

  A. In her old house

  B. In her old neighborhood church.

  C.In the nursing home. D. In the hospital.

  What was the best gift that the author got on Christmas Eve?

  A.The candlelight service.

  B.Praise from Alice’s friends.

  C.Alice’s happiness

  D.Thanks from Alice’s daughter.

  What is the best title for this passage?

  A.A special candlelight Christmas Eve

  B.The dream of all old lonely lady

  C.My special job

  D . The true love

  E

  In New York, Ma witnessed the Alibaba Group opening on the New York stock exchange with the largest initial public offering (IPO,新股发行) in American history. “Alibaba, the world’s largest Internet commerce company makes China equal to the US in the rapidly increasing global competition for technological innovation (创新) and economic transformation ” commented the South China Morning Post.

  It has also made Ma China’s richest man with a fortune of around $25 billion (153 billion yuan), reported Reuters.

  It’s reported that Ma’s rags-to-riches journey is just as spectacular as his Internet empire. Ma failed the national college entrance exams twice before he was finally in Hangzhou Normal University on his third attempt and failed in finding a suitable job several times. Aslo, Mr Ma is a big believer in perseverance, which was proved in his English study experience.

  However, it was his vision and goals that launched his career.

  During a short trip to the US as an interpreter in 1995, Ma first experienced the Internet. He believed in the Internet’s business potential when few other Chinese people did. He started Alibaba in his Hangzhou apartment, with 17 friends and $60,000 of funds. At the time, when e-commerce was unheard of in China, “I called myself a blind man riding on the back of blind tigers,” he once said, according to The Guardian. His unusual ideas earned him the nickname “Crazy Jack Ma” .

  Indeed, Ma is different. Although he is very thin and about 160cm tall, “Ma is yet animated and forceful, said The Guardian. “He is funny, creative, and a compelling (引人注目的) speaker.

  Ma told Time magazine that he was “old for the Internet”. He started to slow down and looked around. According to The Wall Street Journal, Ma “plans to open a tai chi club with the actor Jet Li, build an entrepreneur (企业家) university in Hangzhou and continue to work on several environmental projects.”

  .What made him into e-commerce industry?

  A. His English learning experience.

  B. His foresight and ambitions.

  C. His belief in perseverance.

  D. His 17 friends and $60,000 funds.

  .What does the underlined sentence in Paragraph 5 mean?

  A. Ma is a blind man riding on a blind tiger.

  B. Ma had no confidence in his business future.

  C. Ma knew little about e-commerce.

  D. Ma had not enough money at that time.

  .What’s the personality of Ma Yun according to the passage?

  A. Funny and competitive.

  B. Determined and creative.

  C. Rich and different.

  D. Attractive and believable.

  .What’s the best title of the passage?

  A. The Development of Alibaba Group

  B. Ma Yun’s Personal Life

  C. Ma Yun’s Crazy Success

  D. E-commerce in China

  D【浙江省重点中学协作体2015届高三第二次适应性测试】

  Hilary Smith belonged to a good family. But by the age of twenty, he had spent all the money the good old family had. He then had some trouble with the bank and was put in prison. He escaped from the prison and ran to Australia without delay.

  Hilary did not like Australia and Australia did not like Hilary. What he could do seemed to be one of two things: die or work. Then he remembered that he was not alone in the world. He had an aunt.

  She was his father's only sister, but his father used to say she brought no glory to the family. Hilary, of course, tried to discover what she had done. It seemed that she had failed to marry a nobleman. Instead, she had chosen a husband who was connected with "trade". Of course as soon as she became "Mrs. Parks", her brother considered her dead. Later on, Mr. Parks died and left her a lot of money; but that did not bring her back to life in her brother's opinion.

  Hilary discovered his aunt's address. Fortunately she remained faithful and honest to him even after she fell ill. So Hilary's star shone again, and soon he moved into her house and lived as comfortably as a sailor who had just reached harbor. He had only about a sixpence in his pocket.

  One thing was soon clear: his aunt was seriously ill, and nothing could cure her illness. Hilary was very worried. Fate had found a home for him, and was now going to throw him out of it. There was only one thing that could save him: her will.

  "Will?" she said, "yes, I have made one. That was when I was a girl and had not much money. I left all my money to some religious people."

  "Didn't you make another will when you were married?" Hilary asked.

  His aunt shook her head. "No," she said in a low voice, "There was no need. When I finally had a lot of money I found 1 had no relations."

  On the next day he went to the public library and examined a book of law. It told him what he already believed. When a woman is married, an earlier will loses its value. A new will must be made. If no new will is made, the money goes to the nearest relation. Hilary knew that he was his aunt's only relation. His future was safe.

  After a few months had passed, Hilary's problems became serious. He badly needed money. He had expensive tastes, and owed a lot of money to shopkeepers. They trusted him because his aunt was rich; but the debt was terrible.

  Unfortunately his aunt did not want to discuss money matters at all. In the end they had a quarrel about the small amount of ten pounds. Hilary was not very angry. He began to wonder about a new problem. Was it kind to want his aunt to live any longer? Was it not better for her to die now? While he was considering what to do, his aunt told him that she was going to send for her lawyer. So she was going to make a new will, Hilary thought. She might leave all her money to someone else. Soon he reached a clear decision. He must do a great kindness to the poor old woman.

  One night when the old servant who had been nursing his aunt went off, he doubled the amount of some medicine. The total amount was too great and it could just put her to sleep forever.

  "Thank you," his aunt took the glass from his hand with a grateful look." I want, more than anything, to sleep, and never to wake up again. Is that what you wish, Hilary? Don't blame me if I have some doubts about what you intended to do. Sick people get these ideas, you know. One thing I ought to explain to you. Mr. Parks never married me. He already had a wife and couldn't marry again. That made your foolish father very angry with me...Well, if I am alive tomorrow I shall make another will in your favor. If I die tonight, you'll get nothing...No, Hilary, don't try to take the glass away. If you do that, I shall know; and I don't want to know. Good night, Hilary."

  Then, very carefully, she raised the glass to her mouth and drank.

  . From the story ,we learn that Hilary's aunt was ______.

  A. bad-tempered and lonely

  B. kind-hearted and wise

  C. careless about money

  D. cruel to her niece

  . Why did Hilary's father consider his aunt dead?

  A. Because they lost contact with each other after her marriage.

  B. Because she married a businessman with a wife against his will.

  C. Because she broke away from the family for a better life.

  D. Because he thought she was a shame to the family

  . The underlined sentence "He must do a great kindness to the poor old woman" in paragraph 11 suggests that Hilary Smith ______.

  A. was determined to put his aunt's life to an end

  B. decided to do his aunt a favor to call in a lawyer

  C. made up his mind to take good care of his aunt

  D. would help his aunt to donate all her money to religious people

  . Which of the following is the focus of the story?

  A. Hilary's aunt's money.

  B. Hilary Smith's debts.

  C. The intended murder.

  D. Hilary's aunt's marriage.

  . According to the law book,_____.

  A. Hilary's aunt is not allowed to make a new will.

  B. the money will go to Hilary after his aunt dies without a new will.

  C. Hilary's aunt must leave the money to the religious people after she dies.

  D. Hilary must look after his aunt till her death because he is the only relation.

  . Which detail from the story is the climax of the story?

  A. Hilary escaped from the prison.

  B. Hilary quarreled with is aunt about ten pounds.

  C. Hilary's aunt drank the medicine her nephew prepared for her.

  D. Hilary's aunt made a will to leave all her money to someone else.

  A【湖南省株洲市2015届高三教学质量统一检测(一)】

  Karl Fleming joined the military because he needed a change in his life. He had a successful career with a shipping company but he wanted to do something more. He found that something in the U.S. army. Fleming began his service in 2009 and never looked back.

  A few years later, Fleming volunteered to go to Afghanistan. There, he worked as a bodyguard for the U.S. Army Corps of Engineers. He enjoyed it, except for the almost-nightly rocket attacks. Karl was never hit directly by a rocket, but he didn’t need to be to feel its effects. The rockets caused severe shaking, shaking so bad that Fleming was left with many injuries. He was also diagnosed with concussions(脑震荡) and Post-Traumatic Stress Disorder(PTSD).

  Fleming said he was down but not out. Once he returned from Afghanistan, Fleming underwent one test after another. At first, Fleming said he thought he could recover or be able to return to duty and realize his dream of becoming an officer. But then came the news he had never imagined: Fleming would never be an officer because he was too injured to continue.

  Fleming said he was depressed after learning his military career was over. Add that to the memory loss, extreme anxiety and the many painful medical procedures he was already experiencing. He rarely ventured outside on the weekend. Instead, he preferred to sleep in. All that changed, however, with Fleming’s service dog, Kuchar.

  Fleming said he had heard dogs could help people suffering from PTSD, so he started doing research. Karl eventually selected K9s for Warriors, which is where he met Kuchar, a yellow lab. Fleming and Kuchar trained together for weeks, before returning to Fort Benning. K9s for Warriors provided Fleming with Kuchar and the training for free.

  Life with Kuchar has been life-changing. Fleming doesn’t sleep in any more because Kuchar won’t let him. Instead, they venture out into a world Karl was once afraid of — a world for Fleming that now seems impossible to imagine without Kuchar by his side.

  Karl Fleming joined the army because ___________.

  A. he had a successful career

  B. he loved to be a soldier

  C. he wanted to have a changeD. he expected to work in a shipping company

  From Paragraph 2, we can learn Karl Fleming was __________.

  A. forced to go to AfghanistanB. struck directly by a rocket one day

  C. satisfied with everything in the army

  D. injured because of the shaking from the rockets

  Fleming suffered from the following illnesses except ___________.

  A. concussions

  B. PTSD

  C. memory loss

  D. a heart attack

  Karl Fleming recovered with the help of ___________.

  A. an experienced bodyguard

  B. a well-trained dog

  C. a laboratory engineer

  D. a military officer

  The passage is mainly about Fleming’s__________.

  A. change of life

  B. service in the military

  C. medical procedures

  D. experiences in Afghanistan

  【一年原创】

  原创试题及其解析

  1

  In the fall of 1985, I was a bright­eyed girl heading off to Howard University, aiming at a legal career and dreaming of sitting on a Supreme Court bench somewhere. Twenty­one years later I am still a bright­eyed dreamer and one with quite a different tale to tell.

  My grandma, an amazing woman, graduated from college at the age of 65. She was the first in our family to reach that goal. But one year after I started college, she developed cancer. I made the choice to withdraw from college to care for her. It meant that school and my personal dream would have to wait.

  Then I got married with another dream: building my family with a combination of adopted and biological children. In 1999, we adopted our first son. To lay eyes on him was fantastic—and very emotional. A year later came our second adopted boy. Then followed son No.3.

  In 2003, I gave birth to another boy.

  You can imagine how fully occupied I became, raising four boys under the age of 8! Our home was a complete zoo—a joyous zoo. Not surprising, I never did make it back to college full­time. But I never gave up on the dream either. I had only one choice: to find a way. That meant taking as few as one class each semester.

  The hardest part was feeling guilty about the time I spent away from the boys. They often wanted me to stay home with them. There certainly were times I wanted to quit, but I knew I should set an example for them to follow through the rest of their lives.

  In 2007, I graduated from the University of North Carolina. It took me over 21 years to get my college degree!

  Everything you want won't arrive in your life in one day. It's a process. Remember: little steps add up to big dreams.

  21. What is the meaning of the underlined word “withdraw” in Paragraph 2?

  A. Continue.

  B. Drop.

  C. Graduate.

  D. Learn.

  22. How did the author feel when she had to leave home to study?

  A. Excited.

  B. Happy.

  C. Guilty.

  D. Disappointed.

  23. How did the writer develop the passage?

  A. By giving examples.

  B. By comparison.

  C. By giving explanations.

  D. By time order.

  24. What can we learn from this story?

  A. Well begun is half done.

  B. God helps those who help themselves.

  C. Nothing is impossible to a willing heart.

  D. Pride hurts, modesty benefits.

  2

  In my mind, there is a moving story about a skinny young boy who loved football with all his heart.

  Practice after practice, he eagerly gave everything he had. But being half the size of the other boys, he got absolutely nowhere. At all the games, this hopeful athlete sat on the bench and hardly ever played.

  This teenager lived alone with his father, and the two of them had a very special relationship. Even though the son was always on the bench, his father was always in the stands cheering. He never missed a game. This young man was still the smallest of the class when he entered high school. But his father continued to encourage him.

  The young man loved football and was determined to try his best at every practice, and perhaps he’d get to play when he became a senior. All through high school he never missed a practice nor a game but remained a bench-warmer. Besides, his faithful father was always in the stands, always with words of encouragement for him. When the young man went to college, he decided to try out for the football team as a “walk-on”. Everyone was sure he could never make the cut, but he did.

  The news that he had survived the cut thrilled him so much that he rushed to the nearest phone and called his father. His father shared his excitement and was sent season tickets for all the college games. This persistent young athlete never missed practice during his four years at college, but he never got to play in a game.

  It was at the end of his senior football season that the coach met him with a telegram. The young man read the telegram and he became totally silent. Swallowing hard, he mumbled to the coach, “My father died this morning. Is it all right if I miss practice today?” The coach put his arm gently around his shoulder and said, “Take the rest of the week off, son. And don’t even plan to come back to the game on Saturday.”

  Saturday arrived, and the game was not going well. In the third quarter, when the team was ten points behind, a silent young man quietly slipped into the empty locker room and put on his football gear. As he ran onto the sidelines(边线), the coach and his players were surprised to see their faithful teammate back so soon. “Coach, please let me play. I’ve just got to play today,” said the young man. The coach pretended not to hear him. There was no way he wanted his worst player in this close playoff game. But the young man insisted, and finally feeling sorry for the kid, the coach gave in. “All right,” he said. “You can go in." Before long, the coach, the players and everyone in the stands could not believe their eyes. This little unknown, who had never played before was doing everything right. The opposing team could not stop him. He ran, he passed, blocked, and tackled like a star. His team began to succeed. The score was soon tied. In the closing seconds of the game, this kid got a pass and ran all the way for the winning touchdown.

  Finally, after the stands had emptied, the coach noticed that this young man was sitting quietly in the corner all alone. The coach came to him and said, “Kid, you were fantastic! Tell me what got into you? How did you do it?”

  He looked at the coach, with tears in his eyes, and said, “Well, you knew my dad died, but did you know that my dad was blind?” The young man swallowed hard and forced a smile, “Dad came to all my games, but today was the first time he could see me play, and I wanted to show him I could make it!”

  55. What’s the writer’s attitude toward the young boy in the passage?

  A. Appreciative.       

  B. Critical.

  C. Grateful.

  D. Negative.

  56. What does the underlined sentence “he got absolutely nowhere” mean?

  A. He could never make his father see his performance clearly.        

  B. There was no chance of his becoming a top athlete in this sport.

  C. There was no coach to train him well enough to be a good player.

  D. He had no idea at all what kind of person he wanted to be in future.

  57. Who felt very pleased when the young man was chosen to be a member of his college team?

  A. His teachers and classmates. 

  B. His coach and teammates.

  C. His parents and relatives.

  D. He himself and his father.

  58. From the passage, we can infer that _________.

  A. the young man’s coach played an important role in his growth          

  B. the young man’s father had a great positive influence on him

  C. not until his father passed away did the young man train hard

  D. the young man’s road to success in football was always smooth

  59. Why did the young man return to the match on Saturday?

  A. He hoped his dead father could be proud of his performance.

  B. He thought he was much better than other players in his team.

  C. He wanted to defeat the opposing team in this significant game.

  D. He planned to make money to cure his father of his blindness.

  60. What’s the best title of the passage?

  A. A thrilling football match

  B. Determined father and son

  C. The power of dreams

  D. The encouragement of a coach

  3

  Magic is the performance of tricks. It has been a part of almost every culture in the world. Magic shows might include a disappearing act, card tricks, or pulling a rabbit out of a hat. But what could that have to do with health?

  Kevin Spencer, an American magician goes beyond just entertaining crowds. He also makes magic to improve the lives of people with disabilities.

  What made him change the focus of his work was an accident. It took him almost a year to regain the skills he’d lost as a result of the accident and this made him think about using magic tricks as a tool for healing. So Spencer started conducting workshops all over the world. He teaches magic tricks to children and adults with disabilities.

  Mr. Spencer says magic therapy(疗法) may seem non-traditional. But many skills needed to perform a good magic trick are used in traditional forms of therapy — physical movement, thinking, understanding and social skills are all there. And that social connection with other people can also help people feel better about themselves and increase their confidence.

  Liam Shannon is an example. Liam has a brain disorder that can make learning and connecting with people difficult. People with severe brain disorder may also have trouble understanding complex emotions. The 10-year-old boy said after he learned a few simple tricks, he felt many different emotions. "It made me feel happy and proud. It was great!" Liam said

  Kevin Spencer says seeing kids like Liam come alive is better than all the applause in the world. "We can be on a stage and get the applause of thousands of people, but that is nothing compared to the smile that comes across a kid’s face and when they say ‘Look! I did it!’ and it’s like, ‘yeah, you did!’"

  He says he plans to spend more time working with people with disabilities.

  21.What do you think of Kevin Spencer?

  A. Kind.

  B. Humorous.

  C. Brave.

  D. Honest.

  22.Magic therapy is similar to traditional forms of therapy in_________.

  A. the use of tools

  B. the use of skills

  C. the length of history

  D. the variety of forms

  23.The author mentioned Liam Shannon in the text in order to_________.

  A. show Liam Shannon likes magic very much

  B. tell us Liam Shannon has a talent for magic

  C. prove the success of Kevin Spencer’s magic therapy

  D. explain the connection between magic and personality

  24.What is mainly talked about in the text?

  A. Magic is an interesting performance.

  B. Kevin Spencer is popular with children.

  C. Kevin Spencer has a deep love for magic.

  D. Kevin Spencer use magic to help the disabled.

  4

  When someone walks into your life, a story begins. Pay attention. Who knows where it will go or how it will end?

  I was a transplanted Southerner trying to take root in California, married only six months when my new husband got his first job teaching and coaching at Monterey High.

  It would take almost 10 years for him to move from basic math courses and lower-division sports to teaching physics and coaching varsity basketball. But he was never in a hurry, never in a rush to do anything, really, but his best. All that mattered, he said, was not the status of a job, but how well and how diligently he performed it. While he was busy doing his “best,” I was busy having babies (three in five years) and going to basketball games. That’s where I met David, at the games. I was 21. He was 16. It’s not like you could miss him, really. 

  David was a special education student at Monterey High, a big lumbering(步态笨拙的) bear(急性子) who loved basketball with all his being, but lacked the coordination(协调) to play.

  Instead, starting in high school and continuing some 40 years, he showed up at every game as “team manager,” taking care of the balls and towels, slapping players on the backs if they did well, and giving grief, like it or not, if they missed shots or made the team look bad. He took special umbrage at lazy defense. “You’ve got to box out! Even I can do that! Want me to teach you?” He said. They would pretend to ignore him, but win or lose, they’d at least try a little harder to box out.

  More than anything — maybe even more than basketball — David loved hugs. I’m not sure David ever knew my name. He called me “Mrs. Coach.” At every game, home or away, he’d spot me in the stands and come clomping up(聚拢) the bleachers, parting the crowd like the Red Sea to get a hug. When David hugged you, you knew you’d been hugged. 

  Later, in the years the coach was battling cancer, David never said much, just did his job as always, diligently and well. He hated sweeping the gym floor. Sweeping was a janitor’s job, not the team manager’s, he said. But if he caught the coach sweeping, as he sometimes did, he’d grab the broom and finish the job without a word.

   The morning of the coach’s memorial service, someone took photos of preparations in the gym. Bleachers(露天看台) were pulled out, chairs were set up to seat the crowd. A thousand paper cranes(鹤) folded by his students flew in a “V” from the goal. And there was David, sweeping the floor.

  That was long ago. I had not seen David in years. I heard he’d retired as team manager. I had remarried, moved away, was back in town for a visit, when I ran into him at Toasties, my favorite breakfast place.

   He was sitting at the counter, older, grayer, big as ever, with his sweet wife, Stephanie. “David,” I said, tapping his shoulder, “remember me?” His eyes welled up as he rose from the counter and slowly engulfed(包围)

  me in a big bear hug. “Mrs. Coach,” he said, lifting me off my feet, “Mrs. Coach.”

  When David hugs you, you still know you’ve been hugged.

  55. The underlined word in Paragraph 1 refers to __________ in this passage.

  A. the author’s new husband        

  B. the author’s late(已故的) husband

  C. David

  D. Stephanie

  56. As to the writer’s former husband, we know that he ____________.

  A. loved teaching special education students         

  B. was in a rush to do what he enjoyed

  C. died as team manager

  D. was devoted to his job

  57. The underlined part in Paragraph 5 means “_________”?

  A. was very angry at           

  B. was indifferent to

  C. was reluctant to accept

  D. was sorry to see

  58. What do David and the author’s late husband share in common?

  A. They were both excellent coaches.            

  B. They were both very hardworking.

  C. They both retired with great honor.

  D. They both loved cleaning the gym floor.

  59. Which of the following words can NOT be used to describe David?

  A. Clumsy.

  B. Athletic.

  C. Loving.

  D. Diligent.

  60. The author wrote the passage to _________.

  A. remember her late husband

  B. express her deep love for basketball

  C. bring back happy memories in high school

  D. remember an ordinary but respectable old friend

  5

  Ernest Hemingway was born in Oak Park, Illinois, on July 21st, 1899. Influenced by his father, he enjoyed camping, fishing and hunting at his early age. In 1917, after graduation from high school, Hemingway began his writing career with The Kansas City Star. And then, after being rejected for army service in World War I because of poor vision, he volunteered to serve as a driver for an American ambulance unit in France. In 1918, he transferred to duty on the Italian front, where he was seriously wounded in an explosion. After his recovery, he returned home. He worked for The Toronto star, covered the Greco-Turkish war as a foreign reporter, and then returned to Paris, which was a city full of intellectual life, creativity, and genius after the war. In Paris, where he accomplished a revolution in literary style and language, his first book, Three Stories and Ten Poems, appeared in 1923, and was followed by a short story collection In Our Time, which marked his entry to American literature in 1925.

  Hemingway’s status as a remarkable writer of his time was confirmed with the publication of A Farewell to Arms in 1929. The novel represented a farewell both for war and for love. In 1937, he became a foreign reporter covering the Spanish Civil war. Three years later, he published For Whom the Bell Tolls. Set in Spain during the Civil war, the novel restated his view of love found and lost and described the tough spirit of the common people. In 1912, the same judgment was reflected in his portrait of fisherman, Santiago, with an indomitable spirit in defeat, in The Old Man and the Sea, which was awarded the Pulitzer Prize in 1953. Hemingway won the Nobel Prize in Literature in 1954. With one of the most important influences on the development of the American short story and novel, Hemingway has seized the imagination of the American public like no other twentieth-century author. He died by suicide, in Ketchum, Idaho, in 1961.

  1. Hemingway was turned down for army service in World war I because ______ .

  A. he was unlearned

  B. he was in poor condition

  C. he was inexperienced

  D. he had bad eyesight

  2. The publication of _______ proved Hemingway one of the greatest literary lights of the 20th century.

  A. Three Stories and Ten Poems

  B. A Farewell to Arms

  C. The Old Man and the Sea

  D. In Our Time

  3. Which is the correct order of the following events given in this passage?

  a. Hemingway’s work For Whom the Bell tolls came out.

  b. Hemingway won the Nobel Prize in Literature.

  c. Hemingway’s first book was published in Paris.

  d. Hemingway got wounded on the Italian front.

  e. Hemingway covered the Greco-Turkish war as a foreign reporter.

  A. d, e, c, a, b

  B.e, c, d, a, b

  C.a, c, e, b, d

  D. c, e, a, b, d

  4. The underlined word “indomitable” in the last paragraph probably means ______.

  A. unending

  B. unselfish

  C. unbending

  D. Unchanging

  6

  Breathe, wave and smile. Along with more than 300 other seniors, I marched into the stadium on the afternoon of May 10. The audience burst into deafening cheers. The huge stadium shook with all the whistling and clapping.

  It was as if a Hollywood superstar had walked on stage. And indeed, every single senior that day was a star of the moment. Each deserved it. The seniors had been preparing for four years for this once-in-a-lifetime moment—the commencement (graduation ceremony).

  Seated, I waited anxiously for the opening address. As a foreign exchange student, I was not able to receive a diploma. However, I still had the wonderful feeling of being part of things. Like the other graduates, I was dressed in marron cap and gown.

  Our principal, Mr. Glover, delivered a short, warm greeting. The US national anthem followed and then, hands on chests, a solemn Pledge of Allegiance(美国的爱国誓言).

  Students who had excelled academically gave farewell speeches. The tears in some eyes convinced me that many had deep feelings about the occasion. It was as the class motto says, “Life brings us tears, smiles and memories. The tears dry; the smiles fade; but the memories last forever.”

  Then came the core (the most important part) of the commencement. Hundreds of names were announced. Each graduate walked across the stage to receive his or her diploma from the principal. From the different cheers each graduate got, we had the funny sense that it was a kind of competition of who could cheer the loudest.

  To be honest, the presentation of diplomas got boring. A girl sitting next to me even started yawning. But it wasn’t boring for those receiving the diploma: they would treasure the moment the principal placed the sacred brown document in their hands for the rest of their lives.

  A new page in the book of that person’s life had turned. They were glimpsing their futures: futures of challenge, hardship, perhaps loneliness too, which would take all of their courage.

  1. What’s the article mainly about?

  A. An American high school’s graduation ceremony.

  B. The opening ceremony of a sports meeting.

  C. A presentation of college diplomas.

  D. A US college’s farewell party.

  2. Which of the following statements about the author is FALSE?

  A. She was one of the graduates.

  B. She was a foreign exchange student.

  C. She was excited to receive her diploma.

  D. She felt it great to be at the ceremony.

  3. The right order of the following events is______.

  a. Diplomas were presented to graduates.

  b. Students gave farewell speeches.

  c. The US national anthem was played.

  d. The principal gave a short, warm speech.

  e. Seniors went into the stadium.

  A. b,c,a,d,e

  B. c,d,b,e,a

  C. d,b,e,a,c

  D. e,d,c,b,a

  4. From the text, we can conclude that_______.

  A. a diploma ensures a good job

  B. a diploma guarantees a bright future

  C. the commencement symbolizes the beginning of a new stage in life

  D. everyone felt excited at the presentation of the diploma

  7

  My day began on a definitely sour note when I saw my six-year-old wrestling with a limb of my azalea(杜鹃花)bush. By the time I got outside, he’d broken it. “Can I take this to school today?” he asked. With a wave of my hand, I sent him off. I turned my back so he wouldn’t see the tears gathering in my eyes.

  The washing machine had leaked on my brand-new linoleum. If only my husband had just taken the time to fix it the night before when I asked him instead of playing checkers with Jonathan.

  It was days like this that made me want to quit. I just wanted to drive up to the mountains, hide in a cave, and never come out.

  Somehow I spent most of the day washing and drying clothes and thinking how love had disappeared from my life. As I finished hanging up the last of my husband’s shirts, I looked at the clock. 2:30. I was late. Jonathan’s class let out at 2:15 and I hurriedly drove to the school.

  I was out of breath by the time I knocked on the teacher’s door and peered through the glass. She rustled through the door and took me aside. “I want to talk to you about Jonathan,” she said.

  I prepared myself for the worst. Nothing would have surprised me. “Did you know Jonathan brought flowers to school today?” she asked. I nodded, thinking about my favorite bush and trying to hide the hurt in my eyes. “Let me tell you about yesterday,” the teacher insisted. “See that little girl?” I watched the bright-eyed child laugh and point to a colorful picture taped to the wall. I nodded.

  “Well, yesterday she was almost hysterical. Her mother and father are going through a nasty divorce. She told me she didn’t want to live, she wished she could die. I watched that little girl bury her face in her hands and say loud enough for the class to hear, ‘Nobody loves me.’ I did all I could to comfort her, but it only seemed to make matters worse.” “I thought you wanted to talk to me about Jonathan,” I said.

  “I do,” she said, touching the sleeve of my blouse. “Today your son walked straight over to that child. I watched him hand her some pretty pink flowers and whisper, ‘I love you.’“

  I felt my heart swell with pride for what my son had done. I smiled at the teacher. “Thank you,” I said, reaching for Jonathan’s hand, “you’ve made my day.”

  Later that evening, I began pulling weeds from around my azalea bush. As my mind wandered back to the love Jonathan showed the little girl, a biblical verse came to me: “...these three remain: faith, hope and love. But the greatest of these is love.” While my son had put love into practice, I had only felt anger.

  I heard the familiar squeak of my husband’s brakes as he pulled into the drive. I snapped a small limb bristling with hot pink azaleas off the bush. I felt the seed of love that God planted in my family beginning to bloom once again in me. My husband’s eyes widened in surprise as I handed him the flowers. “I love you,” I said.

  25.Why did the woman cry when seeing her son had broken the azalea bush?

  A. Because she could not tolerate the harm to it.

  B. Because she wanted to hand it to her husband.

  C. Because her son did not ask her for permission.

  D. Because it made her bad mood even worse.

  26.The writer wanted to hide in the mountain cave probably for the reason of

  .

  A. boring daily routine with a feeling of lack of love

  B. her husband’s failing to fix the machine in time

  C. feeling fed up with her endless daily housework

  D. her hoping to seek happiness in a brand new place

  27.We can infer from the passage that the writer expressed love to her husband in that

  .

  A. she felt guilty that she misunderstood her husband and wanted to apologize

  B. she was inspired by her son that love was supposed to be felt and practiced

  C. she felt it necessary to have a complete family for the happiness of herself

  D. she wanted to prove her love and expected the same words from her husband

  8

  Tony Hawken, 57, is divorcing his wife Xiu Li, 51, Britain’s wealthiest woman entrepreneur(企业家), because he says he doesn’t like being rich and is ‘not in the habit’ of spending lots of money.

  The pair traded up their semi-detached home in South Norwood, London, and bought a £1.5million house in Surrey.

  Li, who is now worth $1.2billion (£700million) according to Forbes, quickly settled into a life which included sipping a £900 bottle of wine on a luxurious yacht.

  However, Mr Hawken says he felt more comfortable getting lunch in his local Wetherspoon’s.

  Despite his sudden wealth he continued to buy books from charity shops, and shunned dear clothes.

  In an interview with The Times, he said: ‘I think it made me uncomfortable because I’m not in the habit, I don’t like spending lots of money — I’ve been brought up that way.

  ‘Until recently I was never a wealthy person. I’ve been moderately comfortable because I have been careful with my money.’

  Now the couple have decided to part, Mr Hawken will walk away with just £1million, but says it will be enough for him.

  He added: ‘I have got a settlement which is not great, but it’s enough for me because I don’t have an extravagant lifestyle. I won’t have to work if I’m careful.’

  On a recent trip to China, Mr Hawken said his wife took him on a yacht and treated him to a £900 bottle of wine, but he prefers his local Wetherspoon pub.

  ‘I’m getting a little pay when you consider her potential wealth, but I don’t really want to fight it.’

  Mr Hawken met Li on a blind date while he was still a teacher and she was studying English.

  The couple married, but as Li’s business took off the couple spent more and more time apart. Mr Hawken says the couple have spent most of the relationship apart.

  Far from driving them apart, Mr Hawken believes the distance kept them together, and says they would have divorced a long time ago if they were under the same roof.

  Mr Hawken says his only regret is not getting a divorce sooner, but he didn’t push for it over fears it would affect the couple’s teenage son William, now 17.

  Mr Hawken no longer teaches full-time, but instead gives free tuition to under-privileged children.

  46.Tony Hawken is divorcing his wife Xiu Li because________.

  A.he doesn’t affect his son.

  B.they spent less and less time together.

  C.he is no longer a teacher.

  D. he isn’t used to spending a lot of money.

  47.what does the underlined word “shunned”mean in the fourth paragraph?

  A.enjoy

  B.prefer

  C.avoid

  D. expect

  .From the passage, we inferred that ________.

  A. Tony Hawken gave up teaching, because of his sudden wealth.

  B. Tony Hawken wanted to divorce sooner.

  C. Tony Hawken lived a rich life.

  D. Tony Hawken didn’t care about his son.

  . From the passage,we know that Xiu Li________.

  A.doesn’t like Mr Hawken.

  B.is richest woman in China.

  C.is wealthiest woman entrepreneur in Britain.

  D.is studying English now.

  5.which of the following statements is true according to the passage?

  A.Mr Hawken wants to get a lot of money by divorcing

  B.When Mr Hawken dated Xiu Li,she was an English teacher.

  C.Mr Hawken believes it was the distance that kept them apart.

  D.As many passed, Mr Hawken still kept his own lifestyle.

  9

  I have a friend named John Roberts who owns a horse ranch (牧马场)in San Ysidro.The last time I was there he shared a story with me. “It is a story about a young man who was the son of an itinerant horse trainer. When he was a senior,he was asked to write a paper about what he wanted to be and do when he grew up.”

  That night he wrote a seven-page paper describing his goal of someday owning a horse ranch. He wrote about his dream in great detail and he even drew a diagram of a 200-acre ranch, showing the location of all the buildings,the stables and the track.

  “he next day he handed it in to his teacher. Two days later he received his paper back. On the front page was a large red F with a note that read, ‘See me after class.’”

  “The boy went to see the teacher after class and asked, ‘Why did I receive an F?’”

  “The teacher said‘This is an unrealistic dream for a young boy like you. You have no money. You come from an itinerant family. You have no resources. Owning a horse ranch requires a lot of money. You have to buy the land. You have to pay for the original breeding stock and later you’ll have to pay large stud fees. There’s no way you could ever do it.’ Then the teacher added, ‘If you will rewrite this paper with a more realistic goal, I will reconsider your grade.’”

  “The boy went home and asked his father what he should do. His father said, ‘Look, son, you have to make up your own mind on this. HoweverI think it is a very important decision for you.’”

  “Finally,after sitting with it for a weekthe boy turned in the same paper, making no changes at all. He stated, ‘You can keep the F and I’ll keep my dream.‘”

  John continued, “I tell you this story because you are sitting in the middle of my 200-acre horse ranch. I still have that school paper framed over the fireplace.” He added, “If I had changed my paper at that time, maybe we would not have the chance to be here talking about the dreams .?

  41. What was the boy’s goal described in his seven-page paper?

  A.to own a horse ranch

  B. to be a senior trainer

  C.to build many buildings

  D.to buy many horses.

  42.The teacher give the young boy an F because____________.

  A. he thought he didn’t have money.

  B. he thought he came from an itinerant family.

  C. he thought he didn’t have land.

  D. he thought it was an unrealistic dream for a young boy.

  43.John’s father respond to his son’s puzzlement by__________.

  A. By criticizing his decision.

  B. By helping him make the decision.

  C. By encouraging him to make decision himself.

  D. By praising him brave.

  44.What can we learn from the article?

  A.Make decision by yourself.

  B. Never give up dreams easily.

  C.Praise makes a difference.

  D.Success needs other’s help.

  45.Which of the following can be the best title of the passage?

  A.Keep My Deam

  B.My Horse Rnch

  C. Father’s Lve

  D.My Goal

  10

  My grandma whom we all called Nanny was a solid, first-generation, Italian immigrant. She came to this country with her family on a tiny ship during World War I when she was only 8 years old. During the trip she survived stormy seas,little food, and an attack by a German U-boat. After getting here, her large family worked hard to build a new life in America. They never had much money but were rich in love.

  One day, when I was about 5 or 6 years old, my parents and brothers went on a trip. It was just me and Nanny in the house for the weekend. Nanny seemed so happy to be taking care of me all by herself. She made me a special breakfast that first morning. However, all I could do was to complain about how the food was not how Mom always made it. Nanny quietly put down the plate and went into the living room. I followed a minute later and saw that she had tears in her eyes. It was the first time I had ever seen my strong and proud grandmother cry and I was

  the one who had caused it

  I walked over to Nanny, climbed on her lap, and for the first time in my short life I did something else too. I apologized without being told to and asked Nanny to forgive me. She smiled, rubbed my head, and told me I was a good boy even though I didn't feel like one then.

  That memory just like my Nanny's love will stay in my heart forever. It is a sign of both strength and wisdom. Asking for forgiveness helps us to learn, to grow, and to love.

  56.When did Nanny come to this country?

  A.During World War II,Nanny come to America.

  B.When she was 5,Nanny come to America.

  C.When she was 8,Nanny come to America.

  D.When she was 6,Nanny come to America.

  57.What did Nanny survived during the trip?

  A.stormy seas and an attack.

  B.stormy seas and little food.

  C.little food and an attack.

  D.stormy seas, little food and an attack.

  58.According to the passage, what kind of person is Nanny?

  A.She is narrow-minded and lovely.

  B.She is strong and proud.

  C.She is kind-hearted and fragail.

  D.She is rich and proud.

  59.My grandma cried one day because___________.

  A.my parents didn’t have her travel with them.

  B.I didn’t have the special breakfast that my grandma made.

  C.I complained about her breakfast not as good as my mother’s.

  D.she wasn’t happy to take care of me all by herself.

  60.What did I learn from the experience?

  A.My Nanny's love will stay in my heart forever.

  B.We should learn to apologize to the old.

  C.Life is not easy for grandma.

  D.Asking for forgiveness helps us to learn, to grow and to love.

  11

  San Diego resident Rob Greenfield just reached his goal of going a year without showering. And, surprisingly, he says it wasn’t that hard.

  The founder of an environmentally friendly marketing company, Greenfield started his project with a bike ride across America.

  Greenfield set a few rules for himself on his ride, meant to promote sustainability and eco-friendly living: He could only harvest water from natural sources like rivers and rain, or from wasted sources like leaky faucets. He also had to keep track of exactly how much he used, hoping to show just how little he needed to get by.

  Over his 100-day bike trip, Greenfield was able to use less than two gallons of water a day, or eight Nalgene bottles.

  After making it through the 100-day bike ride without a shower, Greenfield decided to try to go six months. Once that passed, he figured he might as well go a full year.

  All he used was organic soap, toothpaste, and essential oils.?Nobody thought that I smelled at all,” he wrote on his blog. “And I surprised myself at how clean I was, just like everyone else.” Greenfield says he had no lack of friends, and sometimes they’d even join him.

  He even had a few romances, which he says were the times he felt the most challenged.

  ?The hardest part were the times when a beautiful girl wanted me to get in the shower with her and I had to say no,” he told BuzzFeed. “A few times I almost got in and then remembered I was aiming for a year without showers.”

  He said it was also difficult when the temperature dropped in San Diego, and he didn’t feel like having to go to the Pacific Ocean for a dip. But he still did it.

  ?I realized that water doesn’t have to come from a shower head to get me clean,” he wrote.

  In Brooklyn, for example, he found a bath in a leaky fire hydrant.

  For people who don’t live near bodies of water but want to be more eco-friendly, Greenfield recommends trying to be conscious of water usage, taking shorter or fewer showers, and turning off your faucet.

  The world traveler told BuzzFeed his biggest takeaway from the project has been abolishing stereotypes.

  ?A short time ago I would have thought it crazy to forgo conventional showers,” he said. “Turns out it’s not really a big deal at all. We have a tendency to make instant character judgments based on clothing, style, race, sexual preference, political affiliation, and even bathing habits, and most of the time we’re just absolutely wrong.”

  So is he done showering forever? He’s not sure, but it sounds like he’s not ready to throw in the towel (pun intended) just yet.

  “Maybe I never will again or maybe I’ll shower tomorrow,” he said. “All I know is that I feel really darn good right now and I’m having a positive impact on our society.”495

  41.From the paragraphs, we can infer that__________.

  A. Greenfield had lack of friends because he didn’t showered.

  B. Rob Greenfield hasn’t bathed in the past year.

  C. Greenfield was able to use less than two gallons of water a day.

  D. Greenfield has showered using water from natural sources in the past year.

  42. Greenfield didn’t get in the shower with a beautiful girl because________.

  A. he didn’t like girls.

  B. there is no enough water to use.

  C.he wanted to save some water.

  D.he meant what he said.

  43. What does the underlined word ?faucet.”mean?

  A.light

  B.tap

  C.gas

  D.water

  44.According to the passage, which of the following statements is NOT true?

  A. Greenfield is the founder of an environmentally friendly marketing company.

  B. Greenfield has a positive impact on the society.

  C. Greenfield made a full year plan of no showering at the beginning.

  D. Over his 100-day bike trip, Greenfield was able to use less than eight Nalgene bottles of water a day.

  45.The author writes this story to tell us that_________.

  A. Greenfield is a determined man.

  B.we should save water as much as possible.

  C.we should mean what we said.

  D. Greenfield hasn’t showered for a full year.

  12

  In the middle of a hot July afternoon, when the stifling air came with rippling waves of heat, I became a thief of some sort—a thief of music.

  For the first time, I had created an original piano arrangement of one of my favorite songs. Not once had I looked for the help of premade sheet music or video tutorials on YouTube. Using only my ears and iPod, I had transformed a mix of intermingling sounds and intricate melodies into the tones of a single instrument; I had created complex harmonies and voices into something I could perform with only two hands. No help, no guide: I had done it on my own.

  I’ve been a pianist since before my hands were big enough to reach an octave: with a musician and composer for a father, I was all but born on the piano bench. For many years, my musical identity was defined by the notes that others had written in centuries past.

  This was why, when I added the finishing touches to my piano version of a modern rock song, I was proud of myself : this arrangement was mine. What I’d done seemed magical: an ability to take what had already existed—to “steal” a song from my favorite band—and to change it into something different and all my own. I was a thief, but I was also an artist.

  In music, as in other aspects of life, I believe that true originality rarely exists. Almost everything has, in one form or another, been done before. The most romantic novel may very well be a slightly changed version of a play by Shakespeare, which is likely to be borrowed from the works of Ancient Greece: same themes, different characters, different circumstances. But, the novel is no less deserving of praise just because its uniqueness is compromised. Adaptation is not a symbol for failure.

  The gift of creativity is the ability to do what I did on the piano: to find something beautiful, to analyze and twist it and lose yourself in the mystery of its composition, and then to make it new. Such an act is not copying; it is finding inspiration and having the strength and the innovation to use it as fuel for your own masterpiece. The world is nothing more than collections of preexisting parts—scattered and often lost in the chaos of everyday life. I believe it is my job, as an artist, to rearrange this world into what I imagine it to be.

  I refuse to live as if I were trapped within the walls of a museum: looking but never touching, afraid to ruin the so-called perfection of the artifacts inside. Therefore, I will embrace my ability to be a thief, because if I don’t steal what the world has to offer, I’ll never have the tools to share with others a creation of my own.

  My life is my own arrangement, and because of that, anything is possible.

  56.Which of the following can be the best title ?

  A. The passion to create music

  B. Changes from a thief to an artist

  C.A music thief

  D. My piano arrangement

  57.According to the passage , which statement is true ?

  A. Actually ,there was originality only in music

  B. The author created the original piano arrangement all by himself .

  C. He was born on the piano bench .

  D .The author was once a thief who stole music from his favorite band.

  58.Which word has the closest meaning with the underlined word “embrace”?

  A. accept

  B.

  describe

  C. adopt

  D. quit

  59.The reason why the author felt proud of himself was that ___.

  A. he became a real artist without others’ help.

  B. he could play the piano before he was shorter than the piano bench.

  C .he has a father who was a musician and composer.

  D. he changed one of his favorite songs into a piano arrangement on his own..

  60. From the passage we can learn that the writer______

  A. is more of a follower than a leader.

  B. likes to recreate songs whenever it is possible .

  C. would like to lead his life the way he likes

  D. is a stubborn person .

  13

  Today we remember a man who was considered one of the best teachers in the United States. Jaime Alfonso Escalante was born in nineteen thirty in La Paz, Bolivia. Both his parents were teachers. He taught math and science in his home country for twelve years.

  In the nineteen sixties, Jaime Escalante and his wife Fabiola came to the United States. During the next several years, he learned to speak English and repeated his college education and teacher training.

  In nineteen seventy-four, Mr. Escalante was hired at Garfield High School to teach mathematics.

  The school was in a poor area of Los Angeles, California. Many of the students had serious problems with illegal drugs, gangs and violence. The school was in danger of losing its officialapproval to operate.

  Mr. Escalante's students were mostly Mexican-Americans from a Spanish-speaking area of the city. Many were the worst students at the school and thought to be "unteachable." But Mr. Escalante pushed the group to work hard. He told them basic math was too easy and that they had the ability to do harder work.

  He told his students that he believed in them, but they must have the desire to be successful.

  Jaime Escalante started an advanced math program with a small group of students. Advanced Placement is a program of college-level classes and tests.? In nineteen eighty-two, eighteen of his students took the Advanced Placement test in calculus. They all passed.

  Officials at the testing company suspected the students of cheating. Mr. Escalante protested. He said the students were being rejected because they were Hispanic and from a poor school. He urged his students to take the test again.

  Twelve of the students repeated the test. And again, all of them passed. Five earned top scores. Mr. Escalante's students went on to become engineers, scientists and university professors. The advanced math program at Garfield became extremely successful over the next several years.

  Jaime Escalante received many teaching awards including the Presidential Medal of Excellence in Education. A movie about his success, "Stand and Deliver," was released in nineteen eighty-eight. It influenced other teachers to use his methods.

  Jaime Escalante retired from teaching in nineteen ninety-eight and later moved back to Bolivia with his wife. He died earlier this year. He was seventy-nine years old.

  1.From the paragraph, we know that_________.

  A. Jaime Alfonso Escalante was born in the United States.

  B. Jaime Alfonso Escalante taught math and science in a poor area of Los Angeles for twelve years.

  C. Jaime Alfonso Escalante was considered one of the best teachers in Bolivia.

  D. Jaime Escalante received the Presidential Medal of Excellence in Education.

  2.How many years did Jaime Alfonso Escalante teach in the United States?

  A.12years

  B.24years

  C.38years

  D.36years

  3.His students were rejected because ___________.

  A. the students had serious problems with illegal drugs.

  B. the students cheated in the exam.

  C. they were Hispanic and from a poor school.

  D. they were unteachable.

  4.According to theparagraph,we infer that__________.

  A.he died in Bolivia.

  B.his students all passed the Advanced Placement test in calculus.

  C.only five of his students passed the Advanced Placement test in calculus.

  D.12 students were rejected because they were Hispanic and from a poor school.

  5.which is the best title of the paragraph?

  A.The Best Math Teacher

  B. How to Teach Math Well

  C. Stand and Deliver

  D. Jaime Escalante and his wife Fabiola

  14

  It had been a very disheartening day. The doctors had given us the worst of news. Our daughter, who had just completed her first brain surgery to remove a tumor and was going through radiation treatment, was now officially given a two percent chance of survival as this type of cancer had no cure.

  My wife and I decided to take our daughter to lunch before continuing our afternoon conversation. We went to a local restaurant where we sat in silence waiting for the waitress. I noticed a very elderly couple sitting a few booths away, who were also in silence and did not speak a word. I couldn’t help but wonder what challenges they had faced in their life and if they ever faced such terrible news about a child of theirs.

  We eventually ordered our lunch and still sitting in silence we ate what we could. At some point I became intrigued (好奇的) by the old couple. I thought to myself that they hadn’t yet spoken to each other and I wondered if it was the peace they were enjoying or the food or maybe both. However, at some point I lost interest and put my focus back on my lunch.

  Molly was still talking away and enjoying her meal. Her mom and I both listened and tried to be happy in her presence but it wasn’t going very well. All of a sudden I saw a hand come out of nowhere. It was huge and I could tell that it had been afflicted with arthritis (关节炎). I couldn’t take my eyes off that hand. It landed on my daughter’s tiny six-year-old hand and as it did I looked up; it was the old woman who had been sitting with the old man in silence eating their lunch.

  I looked into her eyes and she spoke, but not to me. She looked at my daughter and simply whispered, “If I could do more for you I would…” and then she smiled and moved away to join her husband, who had moved towards the door.

  “Look, a whole dollar,” Molly spoke with excitement as she discovered a crumpled one dollar bill left behind by the old lady. I looked up to thank her, but she was gone. I sat stunned, not sure what had just happened and then I looked over at my wife. In almost unison, we broke out into a smile. The sadness of the day had been wiped out by the crippled hand and generous touch of the old lady. The dollar, although exciting to Molly, was not what made us smile; it was the offer from the old lady, who felt our suffering. The crippled hand symbolized a healing touch and made us realize that we did not have to fight this battle alone.

  41. According to the doctor, the author’s daughter _______.【原创】

  A. suffered from serious heart disease.

  B. didn’t need any more operations.

  C. was likely to die soon.

  D. had been treated by radiation.

  42. The author was ____, when he first caught sight of the old couple in the restaurant. 【原创】

  A. curious

  B. confused

  C. upset

  D. indifferent

  43. By saying “If I could do more for you I would…”, the old woman means_______.【原创】

  A. she had the ability to cure the disease of Molly.

  B. she had the responsibility for the safety of Molly.

  C. she was an old friend of the author and can lend some money to them.

  D. she would like to help Molly as much as possible.

  44. What can we inferred from the passage? 【原创】

  A. The author and his wife were not satisfied with the food in the restaurant.

  B. The old couple had once gone through the same trouble as the author did.

  C. The one dollar bill made the author feel touched by the warmth from others.

  D. The old woman sympathized with Molly because she suffered from serious arthritis herself.

  45. What would be the best title of the text? 【原创】

  A. A girl with cancer

  B. A touch of heaven

  C. A crumpled dollar bill

  D. A kind-hearted couple

  15

  In 1982 Steven Callahan was crossing the Atlantic alone in his sailboat when it struck something and sank. He was out of the shipping lanes and floating in a life raft, alone. His supplies were few. His chances were small. Yet when three fishermen found him seventy-six days later, he was alive.

  His account of how he survived is fascinating. But the thing that caught my eye was how he managed to keep himself going when all hope seemed lost. Giving up would have seemed the only wise option.

  When people survive these kinds of circumstances, they do something with their minds that gives them the courage to keep going. Many people in similarly desperate circumstances give in or go mad.

  “I tell myself I can handle it," wrote Callahan. "Compared to what others have been through, I'm fortunate. I tell myself these things over and over, building up fortitude(坚强意志)….”

  It struck me as something important. And I've told myself the same thing when my own goals seemed far off or when my problems seemed too overwhelming. And every time I've said it, I have always come back to my senses.

  The truth is, our circumstances are only bad compared to something better. But others have been through much worse. So here, coming to us from the extreme edge of survival, are words that can give us strength. Whatever you're going through, tell yourself you can handle it. Tell this to yourself over and over, and it will help you get through the rough spots with a little more fortitude.

  (

  ) 52. The writer seemed more interested in ______.【原创】

  something that made Steven Callahan want to cross the Atlantic

  What made Steven Callahan live through the circumstances

  what Steven Callahan thought of in the ban circumstances

  what was wrong with his boat on the half way

  (

  ) 53. From the third paragraph we can infer that ______.【原创】

  the state of mind means one’s life or death

  keeping up one’s spirit is especially easy for one

  most people have good physiological quality

  bad conditions can strengthen one’s will

  (

  ) 54. How did Steven Callahan build up his confidence in difficulty? 【原创】

  By comparing himself with others.

  By trying over and over again to call help.

  By encouraging himself once and again.

  By trying out different ways of how to survive.

  (

  ) 55. What did the writer get from the experience of Steven Callahan? 【原创】

  One shouldn’t lose heart when in difficulty.

  Encouragement comes from one’s behaviors.

  The strength of example is endless.

  To experience the difficulty does good to one person.

  16

  Isabella Stewart was born in New York City in 1840.Her father made a great deal of money in the trade.During school,her parents took her to Italy to explore the country’s many cultural treasures.   

      One of the private art collections Isabella visited in Milan had a deep influence on her.She wrote to her friends about her dream of owning a house one day with an art collection like the one she had seen in Italy.

      In Paris,Isabella became close friends with one of her classmates,Julia Gardner,whose family was from Boston.Julia would later introduce Isabella to her brother,Jack.In 1860,Isabella Stewart married Jack Gardner.

      The couple had too much art to fit inside their home.So they decided to start planning a museum.Mrs.Gardner didn’t like the cold and empty.spaces of many museums during her time.She wanted a warm museum filled with light.She once said that she decided years ago that the greatest need in her country was art.America was a young country developing quickly in other areas.But the country needed more chances for people to See beautiful examples of art.

      After her husband’s death in 1898.Isabella knew she had no time to lose in building her museum.She bought land,hired a building designer,and supervised(监督)every detail of her museum’s construction.

      Mrs.Gardner opened her museum on January 1,1903,which was then called Fenway  Court.She invited her friends that night for a special musical performance.The next month,she opened the museum to the public.At first,visits were limited to twenty days out of the  year.Visitors paid one dollar to enter.

      Isabella Stewart Gardner died in 1924 in Boston. In her will,she left the museum a million dollars and a series of requirements about how it should be managed, one that the permanent collection cannot be changed.

  (1)The text is mainly about ________.

  A.how Isabella Gardner realized her dream of being an artist

  B.how Mr.and Mrs.Gardner added to their collections of art

  C.how Isabella’s museum was opened

  D.Isabella Stewart Gardner and her museum

  (2)By saying“the greatest need in her country was art”in Paragraph 4.Mrs.Gardner meant that  ________.

  A.America was still a young developing country

  B.there were no museums in America at that time

  C.art fell far behind other fields of the American society

  D.Americans had no chance to enjoy the beautiful art

  (3)In order to achieve her dream of owning an arts museum,Mrs Gardner ________.

  A.raised money from her friends and relatives

  B.sold out all her land and houses

  C.held special musical performances for money

  D.watched over the process of building the museum

  (4)Which statement is NOT true according to the text?      

  A.Isabella-and Jack were classmates when studying in Paris.

  B.The museums at that time was not comfortable。

  C.Isabella opened the museum about five years after Jack’s death.

  D.Great as the museum was,visitors had to wait and pay to enter.

  (5)From the passage,we can learn that the museum ________.

  A.helps earn much money for its collections of art

  B.is called Fenway Court by the visitors

  C.was opened to the public on January 1st,1903

  D.is still affected by Isabella Gardner in management now

查看全部
推荐文章
猜你喜欢
附近的人在看
推荐阅读
拓展阅读
大家都在看

分类
  • 年级
  • 类别
  • 版本
  • 上下册
年级
不限
类别
英语教案
英语课件
英语试题
不限
版本
不限
上下册
上册
下册
不限